Sie sind auf Seite 1von 255

OSCE guide

Third edition

Table of contents
Table of contents ............................................................................................................................. 2
History taking Medicine ............................................................................................................... 6
General review:............................................................................................................................ 7
Notes .......................................................................................................................................... 12
Chest pain ACUTE ................................................................................................................. 14
Chest pain CHRONIC ............................................................................................................ 16
Headache.................................................................................................................................... 18
Blood results / Macrocytic Anemia / B12 Deficiency................................................................. 21
Difficulty swallowing ................................................................................................................ 23
Elevated liver enzymes .............................................................................................................. 24
Drinking / Alcohol ..................................................................................................................... 26
Fever / Tired .............................................................................................................................. 28
Diarrhea ACUTE .................................................................................................................... 29
Diarrhea CHRONIC ............................................................................................................... 30
ASTHMA .................................................................................................................................. 32
COPD management ................................................................................................................... 35
Ankle swelling Bilateral ......................................................................................................... 36
Ankle swelling Unilateral ....................................................................................................... 37
Congestive heart failure CHF ................................................................................................. 38
Heart racing ............................................................................................................................... 40
Fall ............................................................................................................................................. 42
Peripheral vascular disease: ....................................................................................................... 44
Urinary symptoms: .................................................................................................................... 45
Anuria ........................................................................................................................................ 46
Hematuria .................................................................................................................................. 47
Renal stones ............................................................................................................................... 48
Incontinence............................................................................................................................... 50
Lump Neck Swelling .............................................................................................................. 51
Lump Breast............................................................................................................................ 52
Dizziness.................................................................................................................................... 55
INR Counselling ..................................................................................................................... 57
Patient is receiving blood counsel for adverse reactions ........................................................ 59
Counselling Ventilator............................................................................................................ 61
Ethical questions ........................................................................................................................ 62
HIV post-test counselling .......................................................................................................... 63
Lung Nodule .............................................................................................................................. 64
High Creatinine.......................................................................................................................... 66
Impotence / Erectile Dysfunction .............................................................................................. 67
Rheumatology History Taking................................................................................................ 68
Multiple Sclerosis ...................................................................................................................... 69
Obesity....................................................................................................................................... 70
Epilepsy Counselling ................................................................................................................. 71
Medical note .............................................................................................................................. 73
Pre-diabetes Counselling ........................................................................................................ 74
Emergency Medicine..................................................................................................................... 75
Emergency Room ...................................................................................................................... 76
Trauma ....................................................................................................................................... 77

OSCE-guide-III.doc

Page 2 of 255

Unconscious Patient Neuro..................................................................................................... 81


Unconscious Patient Diabetic ................................................................................................. 82
Unconscious Seizing Patient DT / Epilepsy / Brain tumour / ............................................ 84
Heart Attack Chest Pain (MI or Heart Block) ........................................................................ 86
Case 1: Chest pain with initial normal ECG.............................................................................. 87
Case 2: Chest pain with STEMI ................................................................................................ 89
Case 3: Chest pain v fibrillation / v tachy............................................................................... 90
Case 4: Chest pain v fibrillation intoxicated patient ............................................................ 90
Heart Block................................................................................................................................ 91
Headache.................................................................................................................................... 92
Acute Abdominal Pain............................................................................................................... 93
Upper GIT bleeding ................................................................................................................... 98
Lower GIT bleeding................................................................................................................... 99
ECG ......................................................................................................................................... 100
Phone calls ............................................................................................................................... 103
Physical Examination .................................................................................................................. 106
Medical Physical Exam ........................................................................................................... 107
Abdominal examination:.......................................................................................................... 109
Liver Examination ................................................................................................................... 111
Nasal bleeding  Hematological Examination....................................................................... 112
Chest Examination................................................................................................................... 114
Pneumonia ............................................................................................................................... 115
Cardiac Examination Essential HTN case ............................................................................ 117
Secondary Hypertension .......................................................................................................... 119
Hypertension............................................................................................................................ 120
SOB shortness of breathe...................................................................................................... 121
DVT ......................................................................................................................................... 122
Peripheral Arterial Disease Examination................................................................................. 125
Diabetic Foot ........................................................................................................................... 126
Neurological Examination ....................................................................................................... 127
Cranial Nerves Examination.................................................................................................... 129
Tremors.................................................................................................................................... 131
Thyroid Exam .......................................................................................................................... 132
Dermatomes ............................................................................................................................. 133
Neck Examination.................................................................................................................... 134
Carpal Tunnel Syndrome ......................................................................................................... 135
Hand Laceration / Wrist Laceration......................................................................................... 136
Back Pain ................................................................................................................................. 137
Acute Back Pain....................................................................................................................... 138
Chronic Back Pain ................................................................................................................... 139
Back Joint Examination ........................................................................................................... 140
Ankle Twist ............................................................................................................................. 141
Shoulder Joint .......................................................................................................................... 142
Elbow....................................................................................................................................... 143
Hip Joint................................................................................................................................... 144
Knee Joint ................................................................................................................................ 145
Obstetrics and Gynecology.......................................................................................................... 146
History taking OB-GYN ....................................................................................................... 147
OB/GYN cases......................................................................................................................... 147
MGOS history questions:......................................................................................................... 148
History of pregnant lady third trimester................................................................................ 149

OSCE-guide-III.doc

Page 3 of 255

Vaginal Discharge.................................................................................................................... 150


Vaginal Bleeding Non-Pregnant / Not-Known Pregnant...................................................... 151
Vaginal Bleeding Pregnant / Ante-Partum Hemorrhage....................................................... 152
Abnormal Uterine Bleeding (AUB)......................................................................................... 153
Amenorrhea ............................................................................................................................. 155
Infertility .................................................................................................................................. 156
Counselling pre-eclampsia.................................................................................................... 157
Caesarean Section Counselling wants to have CS............................................................. 159
Caesarean Section Counselling does not want to have CS ................................................ 161
OCPs / Contraception Counselling .......................................................................................... 163
HRT counselling...................................................................................................................... 166
Needle Stick Counselling HIV ............................................................................................. 168
Counselling PAP smear ........................................................................................................ 170
Antenatal Counselling.............................................................................................................. 172
Endometriosis .......................................................................................................................... 174
Woman wanting an abortion.................................................................................................... 175
Osteoporosis Counselling / OR / Short Case ........................................................................ 176
Pediatrics ..................................................................................................................................... 177
History taking Pediatrics....................................................................................................... 178
Jaundice ................................................................................................................................... 182
IUGR........................................................................................................................................ 184
Crying Baby............................................................................................................................. 185
Chronic Cough Asthma ........................................................................................................ 186
Anemia..................................................................................................................................... 188
Vomiting .................................................................................................................................. 189
Diarrhea ................................................................................................................................... 191
Mother worried about her child weight ................................................................................... 193
Fever ........................................................................................................................................ 195
Runny Nose / Flu / URTI......................................................................................................... 196
Rash ......................................................................................................................................... 197
Delayed Speech........................................................................................................................ 198
Seizing child counselling ......................................................................................................... 199
ADHD counselling .................................................................................................................. 200
Vaccination counselling........................................................................................................... 202
Child with DM counselling...................................................................................................... 204
Bed wetting counselling / Nocturnal Enuresis......................................................................... 205
Breast feeding counselling....................................................................................................... 207
Psychiatry .................................................................................................................................... 209
Mental status exam the psychiatry interview........................................................................ 210
DSM-IV-TR............................................................................................................................. 213
History taking Psychiatry...................................................................................................... 214
Psychosis.................................................................................................................................. 216
Schizotypal personality disorder.............................................................................................. 218
Panic attack.............................................................................................................................. 219
Tiredness OR weight loss ........................................................................................................ 221
Sleep / fatigue notes................................................................................................................. 222
Insomnia .................................................................................................................................. 223
Domestic Violence Spouse Abuse ........................................................................................ 224
Child Abuse ............................................................................................................................. 226
Domestic abuser....................................................................................................................... 228
Depression ............................................................................................................................... 230

OSCE-guide-III.doc

Page 4 of 255

Depression management / counselling..................................................................................... 230


Dysthymia................................................................................................................................ 231
Premenstrual Dysphoric Disorder (PMDD)............................................................................. 232
Abdominal Pain / Headache .................................................................................................... 233
Somatoform disorders DD ....................................................................................................... 234
Drug seeker .............................................................................................................................. 236
Lithium discontinuity............................................................................................................... 238
Manic patient ........................................................................................................................... 240
Suicidal attempt ....................................................................................................................... 241
Eating disorder......................................................................................................................... 243
Mini-mental status exam:......................................................................................................... 245
Dementia.................................................................................................................................. 246
Delirium................................................................................................................................... 248
Case 1: Dad has not been himself / not sleeping well.............................................................. 248
Case 2: DT ............................................................................................................................... 249
Smoking Cessation counselling ............................................................................................ 250
Refusal to treatment counselling........................................................................................... 251
Truth telling ............................................................................................................................. 252
Organ Donation........................................................................................................................ 253
OCD......................................................................................................................................... 254
NOTES .................................................................................................................................... 255

OSCE-guide-III.doc

Page 5 of 255

History taking Medicine

History taking Medicine

OSCE-guide-III.doc

Page 6 of 255

History taking Medicine

General review:







Introduction:
Chief complaint
History of present illness
Past medical history
Family history
Social history
Introduction

Chief complaint

Analysis of
CC

HPI

Impact
Red flags

TIME: Os Cf D
Character: PQRST


Chronic diarrhea: dehydration

Anemia: fatigue

Cancer: metastasis
Constitutional symptoms
Risk factors

Differential
diagnosis
Allergy
Medications
PMH: diseases (DM, HTN, heart attack, stroke, cancer)
LMP / Last tetanus shot
Events: hospitalization / surgery

PMH

A
M
P
L
E

FH

Any long term disease


Any specific disease

SH

How do you support yourself financially?


With whom do you live?
SAD

OSCE-guide-III.doc

Page 7 of 255

History taking Medicine

Introduction:
- Knock the door
- Go to the examiner  give stickers  use alcohol rub (disinfective)
- Stand at the edge of the chair
- Good evening Mr , I am Dr , I am the physician in charge today,
o < 18 years: use first name
o > 18 years: use Mr / Mrs / Ms
- I understand that you are here because of ,
Examples:
- History Taking: Good evening Mr , I am Dr , I am the physician on duty now, and I
understand that you are here today because of . In the next few minutes I will ask you some
questions about your cc, to figure out a working plan that can help you. If you have any
concerns or questions, please fell free to stop me and let me know.
-

Physical examination: Good evening Mr , I am Dr ; I am the physician in charge now. I


understand that you are here today because of . In the next few minutes I will do a physical
exam on your (e.g. shoulder), during which I will ask you to do some movements that may
cause some discomfort and may be some pain, if you feel either, please do not hesitate to stop
me. And if you have any concerns, please let me know. And I will be telling the findings to
the examiner while we proceed.

1- Chief complaint
[A] If the CC not known
- How can I help you today?
1. Start to ask based on the age:
MALE
FEMALE
> 65 yrs

Do you take meds on regular basis? Do you have a list of it? Or the
bottles? Do you take sleeping pills?

Do you have difficulties with sleeping?

Do you have difficulties with your balance1, any falls?

Do you have difficulties with urination (incontinence / retention)?

Do you have changes in your vision / hearing?

Do you have changes in your mood / memory?
50s

Do you have problem drinking  use CAGE

Depression2  identify through social history

ED / Impotence
30s

Psychiatric problems

SAD  social history
Teen / 20s

Premature ejaculation

Abortion

STDs

STDs

Eating disorders
2. Special conditions:

Fatigue

Insomnia

Headache

Abdominal pain

Vaginal bleeding
1
2

Domestic abuse

Normal pressure hydrocephalus: ataxia / incontinence / dementia


Common triad association: alcohol / depression / suicide

OSCE-guide-III.doc

Page 8 of 255

History taking Medicine







Chest pain
SOB
Heart racing
Dizziness
Numbness

[B] If the CC is known


CLEAR

Any pain: headache, chest pain

Cough

Fever

Clarify:
1. Start first open-ended questions:
- What do you mean?
- Can you tell me more about this
2. Active listening:
- Do not interrupt
- Do not duplicate
3. Body language: nod your head

Panic attack

NOT clear

Vague symptoms: dizziness, tired

When the patient uses medical terms:
abortion, jaundice, palpitation

When the patient uses the words: change /
difficulty
Clarify:
- Use closed ended questions

2- History of present illness


TIME (Os Cf D):
- Onset:
o How did it start (sudden / gradual)?
o Setting: what were you doing?
- Course: from that time till now, is your cc all the time or is it on and off (continuous vs.
intermittent)?
o Continuous:
 From the beginning
 Does it or or the same?
 Frequency: is it your first time?
o Intermittent:
 Frequency: how often did you have it in the last ()?
 Are these attacks similar or different?
in duration (longer) or in severity (more severe) or in
frequency (more often)?
 What brings it? On doing certain thing, occurring at rest, awakening you
from sleep?
- Duration:
o Usually given in the question
o If > 24 hours  empathy: were you able to sleep
Character:
+ PAIN:
PQRST
always ask from the beginning?
- Position: where did it start? Can you point with one finger on it?
- Quality: how does it feel like? Squeezing, tightness, sharp, stabbing, burning?
- Radiation: does it shoot anywhere?

OSCE-guide-III.doc

Page 9 of 255

History taking Medicine

Severity: on a scale of 10, 1 being the mildest pain you have ever had and 10 is the most
severe, how much do you rate this one?
o If bad pain  empathy: this must be difficult
Timing:
o Does it change with time; is it more in morning or towards the end of the day?
o Any variation?
Triggers:
o What brings your headache?
o Is it related to: stress / lack of sleep / over sleep / flashing lights / smells?
o If female: is it related to your periods? Are you taking any meds or OCPs?
o Any diet triggers?

+ Fluids (e.g. bleeding, diarrhea, vomitus):


- COCAB/D: colour, odour, contents/consistency, amount blood/discharge
What or :
- What increases or decreases your cc?
- Examples: noise / quiet places / movements / resting / coughing / leaning forward / lying
down /
[D] ASSOCIATED SYMPTOMS:
- In addition to your cc, did you notice any other symptoms?
- Now, I am going to ask you more questions to see if you have any other symptoms beside
your cc.
+ By systems: e.g. chest pain
1- Same system
2- Near-by systems
3- Constitutional symptoms [RED FLAGS]
4- Risk factors
Now, I am going to ask you more questions to see if you have any medical conditions
that may explain / cause / predispose your cc.
5- Causes and consequences
6- Review of systems:
+ By differential diagnosis: e.g. headache
+ By causes and consequences: e.g. Macrocytic anemia Vit B12 deficiency
3- Past medical history
Because it is the first time I see you, I need to ask you some questions about your past medical
history.

Allergy / Medications:
a. Allergy
b. Medications (OTC, Rx meds, supplements, herbs)

Past history of diseases for which you see doctor on regular basis (DM, high blood pressure,
heart attack, stroke, cancer)?

LMP for females

Events: any history of hospitalization / procedures?
4- Family history
Because it is the first time I see you, I need to ask you some questions about your family medical
history, and by this I mean your parents and siblings.

OSCE-guide-III.doc

Page 10 of 255

History taking Medicine

1- How do you describe their general health?


2- Any long term disease in the family? DM, HTN, heart attack, stroke?
3- Any specific disease runs in the family?
5- Social history
1- How do you support yourself financially?
2- With whom do you live?
a. Alone  are you in any relationship? Are you sexually active?
b. With a family  how is the relation with ? Is she/he supportive?
3- SAD:
a. Do you smoke?
b. Do you drink alcohol?
i. How much?
ii. For how long?
c. Have you ever tried recreational drugs?

OSCE-guide-III.doc

Page 11 of 255

History taking Medicine

Notes
EMPATHY:
If during history taking you noticed the patient is in pain  empathy: I can see you are in pain,
please bear with me for few minutes and I will give you a pain medication as soon as I can
- In the short cases (5 minutes)  use at least 1-2 empathy statement
- In the long cases (10 minutes)  use at least 3 empathy statements
-

Patient says Im not ok / Im not so good  I am sorry to hear that


Patient says I fell down  Oh, did you hurt yourself / No I am glad to hear that
After suicide  It looks like you have gone through difficult times, can you tell me more
about these difficulties you are facing
Patient is regaining consciousness in the ER  Mr you have had and you are in the
hospital now, you are ok now, I am Dr and we are here to make sure youre ok

I have a concern!
Whenever the patient says: I have a concern: STOP the interview!
- Can you tell me your concerns!
- Why are you concerned?
Worried / occupied patient!
Whenever the patient shows non-verbal clues of being worried / occupied:
- I can see that you are worried / occupied! Would you like to tell me more about your worries
or concerns?
-

Question types:
Types of questions you can use: open-ended, closed-ended, choices
Types of questions you can NOT use: leading questions, stacking questions

Time usage technique:


1/2/3
If you are stuck during the history taking; and could not find questions to ask;
- First time:
Summarize
- Second time:
Ask about constitutional symptoms / Review of systems
- Third time: PMH / FH / SH
Medications:
When you ask about the medications and even if the patient says NO, in certain diseases, you
need to confirm special medications, by saying, what about
- Diarrhea  what about antibiotics
- Asthma  aspirin / blockers (HTN, heart failure, thyroid disease, social phobias)
- Migraine  what about OCPs? (any birth control pills)
- Bleeding  what about aspirin / blood thinners
- Torticollis  do you take anti-psychotics? Do you see psychiatrist/ did he give you any
medications
MSD (mood / suicide / drinking): whenever you find one, screen for the others
When the patient comes with a chronic long duration complaint, ask him: and what
happened recently that made you decide to seek medical advice now?
Whenever the patient has something affecting his life / social issue: Refer to social worker
/ services
Do NOT criticize other doctors or the patient

OSCE-guide-III.doc

Page 12 of 255

History taking Medicine

For breaking bad news [SPIKES] approach:


o Setting
o Perception of the patient: what do you know about
o Invitation: how much details you would like to know about
o Knowledge: give knowledge in understandable pieces, and make sure the patient
understands this info.
o Empathy /+/ Expectations: what are your expectations from todays visit
 You will have tremendous support, you are not working alone
 We will try to make you as comfortable as possible
 Oncology group AND palliative group: Referral: do you have any doctor you
are comfortable with?
o Summarize /+/ Strategy
Counselling:
1. Inform the patient
a. The medical condition is called
b. Explain the pathophysiology
c. Consequences / complications of the condition! May happen again, may affect
ability to do certain things,
d. Investigations that might be needed to conclude the condition OR to look for
complications
2. Preventive measures: e.g. modify the poly-pharmacy
3. Treatment: life style / medications (side effects / alternatives / consequences of not
receiving treatment)
4. Offer more info: brochures / web sites / support groups
5. Break every 30-60 seconds (check & recheck that your patient understands); ask the
patient: does that make sense? Is this acceptable? Reasonable? Is it clear?
6. General tips for the counselling sessions:
- Make it interactive not lecturing
- At the beginning ask whether your patient has a specific concern
- Do not mislead your patient; if you are not sure about any thing, say that this is a very
good question and you are going to check the answer for him.
-

Whenever you hear car accident:


I am sorry to hear that!
Was anyone hurt? I am sorry for that
Were you driving or a passenger?
If you do not know the answer to a question:
This is a good question / point, I will check it for you and we will discuss it next visit.
It is better to refer you to the specialist; there are too many points regarding this issue that it
will be better to discuss it with the specialist.
A good statement to use in different counselling situations: always in medicine, we balance
the benefits and the side effects.
GIT symptoms:
Nausea / Vomiting
- Heart burn / acidic taste in mouth
Abdominal pain
- Distension / bloating / gas
Change in bowel movements: constipation / diarrhea
Blood in stools / vomiting blood
LIVER: yellowish discoloration / itching / dark urine / pale stools

OSCE-guide-III.doc

Page 13 of 255

History taking Medicine

Chest pain ACUTE


Introduction: Good evening Mr , I am Dr , I am the physician on duty now, and I
understand that you are here because you have chest pain for the last minutes. In the next few
minutes I will ask you some questions about your chest pain, to figure out a working plan that can
help you. If you have any concerns or questions, please fell free to stop me and let me know
Analysis of  OsCfD: Onset / setting: what were you doing?
CC
 PQRST:

Position: where did it start? Can you point with one finger on it?

Quality: how does it feel like? Squeezing, tightness

Radiation: does it shoot anywhere? Your jaw, your shoulders, your back?
 What or :

Breathing / position

How did you come to the clinic today? Ambulance  did they give you
aspirin / nitrates? Did it help you?
Impact
 Atherosclerosis:

Hx of stroke? Symptoms of stroke (weakness / numbness / change in vision


/ difficulty finding words)?

Any sexual dysfunction?

Do you feel abdominal pain after eating?

Do you feel cold extremities

Do you feel cold feet? Pain after walking?


 CHF:

SOB? How many pillows do you use? Do you wake up gasping for air?

Any swelling in your LL? How high does it go? Is it related to position?

Eye puffiness? Pain on the liver?


Red flags
Constitutional  Fever / night sweats / chills
 How about your appetite? Any weight changes?
symptoms
 Any lumps or bumps in your body?
Risk factors
CAD
Pericarditis
PE
Differential Same system  Nausea / vomiting
diagnosis
 Sweating / feeling tired
 SOB  if yes, analyze (OsCfD)
 Do you feel your heart racing?
 Did you feel dizzy / light headedness / LOC? Are you tired?
 Did you notice swelling in your ankles? Legs? Calf muscles?
Near by
 CHEST:
systems

Any cough or phlegm? Chest tightness? Wheezes?

Recent fever / flu like symptoms? Muscles/ joint ache?


 GIT:

Difficulty swallowing (esophageal spasm)

Heart burn / acidic taste in your mouth?

Any hx of PUD? Reflux? GERD?


 Chest wall: any trauma, any blisters / skin rash on your skin
 DVT: any pain / swelling / redness in your legs / calves? Any
recent long travel?

OSCE-guide-III.doc

Page 14 of 255

History taking Medicine

Risk Factors:

CAD (Coronary Artery Disease):


MAJOR:
o
High blood pressure
o
High blood sugar
o
High cholesterol: have you got your cholesterol measured before?
o
Family hx of heart attack at age < 50 yrs
o
SAD: Smoking / Cocaine
MINOR:
o
Look for obesity
o
Do you exercise
o
How about your diet, do you eat a lot of fast food?
o
Are you under stress?

Pericarditis:
o
Recent flu like symptoms
o
Medications (Isoniazide / Rifampicin)
o
Hx of surgery
o
Hx of heart attack
o
Hx of kidney disease / puffy face / frothy urine
o
Hx of TB
o
Hx of autoimmune disease

Pulmonary Embolism:
o
Recent long flight
o
History of malignancy
o
Family history of blood clots
o
Female: pregnancy / OCPs / HRT
Chest Pain
Acute
Chronic
Minutes hours
Hours days
Intermittent
Continuous
Cardiac:
Cardiac:
Cardiac:
- CAD
- Pericarditis
- Unstable angina
- Aortic dissection
- Unstable angina
Non-cardiac:
Non-cardiac
- Cancer
- Tension
- Pneumonia
- Herpes zoster
pneumothorax
- Pleurisy
- Trauma
Panic attack
Pulmonary embolism Panic attack
GIT:
GIT:
- GERD
- GERD
- PUD
- PUD
- Esophageal spasm
- Esophageal spasm
Questions:
 Investigations: ECG / Cardiac enzymes

OSCE-guide-III.doc

Page 15 of 255

History taking Medicine

Chest pain CHRONIC


Chest pain for 6 weeks
UNSTABLE ANGINA /+/ GERD GIT CAUSES OF CHEST PAIN
Intro
Analysis of
CC

But first I would like to ask you, how do you feel now?
OsCfD: Onset / setting: what were you doing?
PQRST:

Position: where did it start? Can you point with one finger on it?

Quality: how does it feel like? Squeezing, tightness

Radiation: does it shoot anywhere? Your jaw, your shoulders, your back?
 What or :

Breathing / position

Is it related to activity? How many blocks were you able to walk? And
now?

How about rest? And during night?

When was the last attack


 Triggers
Angina
GERD



Exertion
 Golf (leaning forward)
Stress (emotional)
 Coffee / dairy products
Cold air
 Smoking / Alcohol
Heavy meals
 Heavy / late meals
Sexual activity
 Pregnancy (progestin)
Impact
Effect
 Atherosclerosis
 Chronic cough
 CHF
 Change in the voice
Red flags
Constitutional  Fever / night sweats / chills
symptoms
 How about your appetite? Any weight changes?
 Any lumps or bumps in your body?
Risk factors
CAD
Differential Same system  Nausea / vomiting
diagnosis
 Sweating / feeling tired
 SOB  if yes, analyze (OsCfD)
 Do you feel your heart racing?
 Did you feel dizzy / light headedness / LOC? Are you tired?
 Did you notice swelling in your ankles? Legs? Calf muscles?
Near by
 CHEST:
systems

Any cough or phlegm?

Chest tightness? Wheezes?

Recent fever / flu like symptoms? Muscles/ joint ache?


 GIT:

Difficulty swallowing (esophageal spasm)

Heart burn / acidic taste in your mouth?

Any hx of PUD? Reflux? GERD?


 Chest wall: any trauma, any blisters / skin rash on your skin
 DVT: any pain / swelling / redness in your legs / calves? Any
recent long travel?
PMH
FH
SH






OSCE-guide-III.doc

Page 16 of 255

History taking Medicine

Counselling:
Concern: The patient has a concern; is this IHD? Is his heart endangered?

This is quite a reasonable concern? What made you think about that?

Especially you have many risk factors that may predispose to heart attack. Right now the
physical exam is ok; it is less likely your condition is due to heart problem. But we still
need to check your heart more, we will do some lab works and an electrical tracing for
your heart (ECG), then if we find that we still need, we may send you to have a stress
ECG, in which, we trace your heart while you are exercising. Then we know for sure the
condition of your heart.

However, we would like to take measures to try to decrease your risk of developing heart
attack, e.g. exercise / diet / smoking / cholesterol.

On the other hand, the most likely diagnosis of what you have is a medical condition
called GERD. GERD stands for Gastro-Esophageal Reflux Disease, any idea about
that? Do you know anything about GERD?

Explain with a drawing: the esophagus (food pipe) / lower esophageal sphincter /
physiologic mechanism to keep it competent / in GERD  weak sphincter  acid
refluxes / irritates the esophagus / impact (short term and long term)

Treatment:
o
Avoid triggers
o
Life style modifications:
 Raise the head of the bed
 Smaller meals
 Do not eat late
 smoking
o
Medications: proton pump inhibitors (PPIs), e.g. pantoprazole
o
Side effects of PPIs:
 In general, proton pump inhibitors are well tolerated, and the incidence of
short-term adverse effects is relatively uncommon
 Common adverse effects include: headache (in 5.5% of users in clinical
trials), nausea, diarrhea, abdominal pain, fatigue, and dizziness. Long-term
use is associated with hypomagnesemia
 Because the body uses gastric acid to release B12 from food particles,
decreased vitamin B12 absorption may occur with long-term use of protonpump inhibitors and may lead to Vitamin B12 deficiency
 Infrequent adverse effects include rash, itch, flatulence, constipation, and
anxiety

OSCE-guide-III.doc

Page 17 of 255

History taking Medicine

Headache
HPI:

OsCfD: gradual onset / all the time / increasing / for few days

PQRST:
temporal area / vague deep pain / severe
o
Severe:  empathy: this must be difficult, were you able to sleep
o
Triggers:
 What brings your headache?
 Is it related to: stress / lack of sleep / flashing lights / smells / diet?
 If female: is it related to your periods? Are you taking any OCPs?

What or ? Lying down / coughing / resting in quiet room /


Associated symptoms / differential diagnosis:
1- Infection:

Fever / night sweats / chills / constitutional symptoms

Neck pain / stiffness

Recent flu like symptoms / skin rash

Bothered by light

Nausea / vomiting
2- Subdural hematoma:

Trauma / fall

SAD (Smoking, Alcohol, Drugs)


3- Subarachnoid hemorrhage:

Very acute /+/ Very severe headache / the worst headache

History of aneurysm or polycystic kidney disease

Visual changes (pupil changes)

Your heart is beating slow


4- Neurological screening:
If while you are doing the neurological screening, you suspect particular cause, e.g.
temporal arteritis  go to TA block then return to complete the neurological screening.

Cranial nerves:
o
Any change in smelling perception?
o
Any difficulty in vision / vision loss?
o
Any difficulty in hearing / buzzing sounds?
o
Difficulty finding words? Aphasia?
o
Difficulty swallowing?

Brain:
o
Any dizziness / light headedness / LOC?
o
Any tremors / jerky movements / hx of seizures?

Personality and cognition:


o
Any memory / mood / concentration problems?
o
Did anybody tell you that you there is a change in your personality recently?

UL/LL:
o
Any weakness / numbness / tingling in your arms / legs
o
Any difficulty in your balance / any falls?

Spine:
o
Any difficulty with urination / need to strain to pass urine?
o
Any change in bowel movements?

OSCE-guide-III.doc

Page 18 of 255

History taking Medicine

5- Temporal arteritis:

Age > 55 years

When you touch this part of your head, is it painful? Can you comb your hair?

Do you feel cord-like structure?

Do you have any visual disturbances / impairment?

When you are chewing, is it painful, cramps in your jaws?

Any weakness / numbness in your shoulders / hips?

Is there any cough? Mild fever?


6- HTN:

Were you diagnosed before with high blood pressure?

Do you know your blood pressure? Have you had it checked before?

Salty food? Family history of HTN / heart disease?

Any history of repeated headaches?


7- Extra-cranial causes of headache:

Eyes: any hx of glaucoma, red eye, pain in your eyes? Do you usually wear eyeglasses?
Do you see well? Any vision problems? When was last time you saw your optometrist?

E do you have any pain / discharge in your ears?

N nasal discharge / sinusitis / hx of facial pain?

T any teeth pain / difficulty swallowing?


8- Medications:

Do you take any nitrates?

Do use too much of advil (or other NSAIDs)? For how long?

Were you used to take large amounts of coffee and then you stopped abruptly?

OCPs?
Temporal Arteritis:
Investigations:

TA biopsy

Doppler

ESR

CT head

Treatment: If suspect GCA (Giant Call Arteritis),


immediately start high dose prednisone; 1 mg/kg
OD (to prevent blindness) then maintain dose daily
(in divided doses), then taper prednisone dose
after symptoms resolve.

Polymyalgia Rheumatica:

Constitutional symptoms + Fatigue

Age > 50 yrs

ESR > 50 mm/hr

Treatment: Corticosteroids; 15 mg/day (for long


periods of time). Taper after ESR decreases < 50
mm/hr and stop if ESR normalizes (< 20 mm/hr)

OSCE-guide-III.doc

Page 19 of 255

History taking Medicine

PRIMARY HEADACHE
Intermittent / episodic
Headache
Tension
Duration
Days
Quality
Pressing / tightening /
bilateral
Place
Associated
symptoms
Aggravating
factors
Others

Band around the head

Stress
Physical injury

Treatment
- Acetaminophen
- NSAIDs
- Physiotherapy / ms
massage / heat
compresses (neck)

OSCE-guide-III.doc

Migraine
Hours
Mostly unilateral /
pulsating / interferes with
daily activities
Mostly unilateral
Photophobia / phonophobia
Physical activity / motion
Light / sound

Family history

Types:
+ Classical: with aura
+ Non-classical: no aura
Acute phase:
- Acetaminophen
- NSAIDs (ibuprofen)

Cluster
Minutes
Comes in series / severe
pain / hyperaesthesia
Around the eyes / nose
Red eyes / lacrimation /
rhinorrhea / sweating
Smoking / alcohol
Smell / exercise

- Oxygen
- NSAIDs
- Triptans / ergotamine

Prophylactic:
- Remove precipitant
- Ca channel blockers
- Triptans (somatriptan)

Page 20 of 255

History taking Medicine

Blood results / Macrocytic Anemia / B12 Deficiency


Cases:

Middle age man received blood report showing Macrocytic anemia


Elderly (65 years old) man presenting with ataxia, dizziness, macrocytic anemia.
Findings: poor diet. Most likely diagnosis: pernicious anemia

Investigations:

CBC / Differential / Peripheral blood film

B12 level in the blood / Folic acid level in the blood


Introduction
Good morning Mr , I am Dr , I understand that you are here today (OR we called you to
come) to get the results of your blood tests (OR x-ray) that you have done few days ago, I have it
and I am going to discuss it with you. However, because it is the first time that I see you, I need
first to ask you some questions to help me get better understanding and interpretation of these
results. Is it OK with you?
+ If patient anxious about results  tell her/him then continue history
1- First let me ask you few questions about the lab test itself (this applies to any blood work, xray, HIV testing, biopsy, jaundice, anemia):

Why have you done this test?

Is it the first time to have it?

Who ordered this test for you? Why?

When did you have it?


2- Give the information:

If it is bad news  SPIKES

If abnormal blood results or x-ray: explain the results to patient, to decrease the patient
anxiety. The blood tests you had show that you have special kind of anemia that we call
Macrocytic anemia in which the size of the red blood cells (which are a component of
your blood) is larger than usual, there is different causes for this.
o
If the patient panicked? Is it serious doctor?
 Do you have any concerns?
 There are different causes that may lead to this result, some of them are
serious, could be, we need to do more investigations.
3- I would like to ask you some questions to see how did this (anemia) affect you:
CONSEQUENCES of anemia:

Anemia symptoms:
o
Did anyone comment that you are pale, recently?
o
Did you notice any in your activity level?
o
Heart racing / SOB / chest pain with exercise?
o
Any dizziness / light headedness / fainting?

Neuro symptoms:
o
Any tingling / numbness / in your feet?
o
Difficulty in your balance / any falls?
o
Any difficulty concentrating / memory problems?

OSCE-guide-III.doc

Page 21 of 255

History taking Medicine

4- I would like also to ask more questions to find out what might be the cause:
CAUSES of Vit B12 deficiency:

Diet intake: Are you vegetarian? For how long? Do you take supplements?

Gastric causes:
o
Did you have any surgeries in your stomach? When?
o
History of long standing PUD? Any heaviness / fullness after meals /
indigestion? (Lack of acidity)
o
Were you ever yourself or any member of your family diagnosed with what is
called autoimmune disease; by this I mean a condition called pernicious
anemia, or rheumatoid disease / lupus?

Terminal ileum:
o
Did you have any bowel surgery before?
o
Were you diagnosed with Crohns disease before? Any repeated attacks of
diarrhea? Any foul smelling bulky stools?

Pancreatic and liver failure:


o
Any hx of liver / pancreatic disease?
o
Yellowish discoloration / itching / dark urine / pale stools?

Alcohol:
o
Do you drink alcohol? How much? For how long?

Meds:
Do you take medications on regular basis? What kind?
o
Have you ever been diagnosed with epilepsy? Do you take anti-epileptics?
o
Do you see a psychiatrist? Do you take a mood stabilizer?
o
Any hx of chemotherapy? Have you ever taken a drug called methotrexate?

Hematological causes:
o
Any recent bleeding (nose / gum / coughing / vomiting blood)? Any bruises /
dark urine / tarry stools?
o
Any fever / night sweats / chills? Change in appetite / weight loss? Lumps and
bumps in your body (for LNs)? Bony pains? Any repeated infections?

Parasites:
o
Have you ever consumed raw fish (chronic intestinal infestation by the fish
tapeworm: Diphyllobothrium)?
5- PMH
6- FH
7- SH

OSCE-guide-III.doc

Page 22 of 255

History taking Medicine

Difficulty swallowing
What do you mean by difficulty swallowing?

Do you feel difficulty initiating the swallowing?

Do you feel pain when you swallow?

Do you feel food is stuck? Can you point where it is usually stuck?
Dysphagia
(esophageal)

Progressive
All the time and

Progressive, solids
then fluids

Mechanical
Cancer OR stricture




Intermittent
On and Off

Achalasia:
respiratory
symptoms

Fluids first
Then solids

Fluids and solids

Solids only
(Large bolus)

Achalasia
Scleroderma

Esophageal spasm

Esophageal webs and


rings

Scleroderma:
reflux / tight skin of
fingers / change
hand color when
exposed to cold
(Reynauds
disease)

Mechanical Dysphagia:
Analysis
 OsCfD: gradual, progressively, to solids then fluids / PQRST / What /
of CC
 Associated symptoms:

The same system:


o
Nausea / vomiting / undigested food
o
Change in bowel movements
o
Change in the size of the abdomen / abd pain / blood in stools
o
Liver: yellowish discoloration / itching / dark urine/ pale stools

Near-by systems:
o
Any chest pain / tightness
o
Any cough / change in your voice / neck swelling (thyroid lump)
Impact
Weight loss
Red flags
 Constitutional symptoms: fever/ night sweats/ chills / change in appetite / loss
of weight / lumps & bumps
 Risk factors:

GERD / PUD:
o
Hx of heart burn
o
Were you ever diagnosed with a condition called GERD / PUD
o
Have you ever checked with a camera or a light (endoscope)
inserted into your stomach

Smoking / Alcohol

Family history: esophageal cancer

Radiation to chest

Have you ever swallowed any chemical?


Barium swallow: string sign /or/ apple core sign / graded narrowing of intra-esophageal diameter
extending from T5-T8 level  most likely diagnosis: esophageal cancer
Investigations: endoscopy and biopsy / chest x-ray and CT / liver function tests / abdominal US

OSCE-guide-III.doc

Page 23 of 255

History taking Medicine

Elevated liver enzymes


Introduction
HPI:
1- First let me ask you few questions about the lab test itself (this applies to any blood work, xray, HIV testing, biopsy, jaundice, anemia):

Why have you done this test?

Is it the first time to have it?

Who ordered this test for you? Why?

When did you have it?


2- Give the information:

Liver enzymes: AST / ALT 2

Explanation: there is increase in one of the markers used to assess / check the liver
functions, it may indicate that there is an injury to your liver, I am happy you came here
today to discuss it so that we can figure this out.
3- I would like to ask you some questions to see how did this affect you:
CONSEQUENCES of liver injury:

Acute phase:
o
Any yellowish discoloration / itching / dark urine/ pale stools
o
Recently, have you noticed any fever / flu-like symptoms / muscle/joint aches
o
Constitutional symptoms: sweats / chills / appetite / weight / lumps

Chronic manifestations:
o
Did you notice any increase in the size of your abdomen? Puffiness in your face?
Swelling in your legs/ ankles?
o
Bruises in your body?
o
Vomited blood? Blood in stools?
o
WITH ALCOHOL: did you notice changes in memory and concentration? Any
weakness / numbness? Balance and falls?
4- I would like to ask you more questions to find what might be the cause:
CAUSES of liver injury:
Now, I would like to ask you some questions to see if you were exposed to liver disease
without being aware of that, some of these questions may be personal, but it is important to
ask (start from least offensive to most offensive)
Including the alcohol, during which  Drinking assessment
5- PMH:

Were you ever diagnosed with liver disease before

Were you ever checked for liver disease before

Were you ever vaccinated for liver disease before


6- FH: suicide / depression / drinking / liver cancer
7- SH

OSCE-guide-III.doc

Page 24 of 255

History taking Medicine

Now, I would like to ask you some questions to see if you were exposed to liver disease without
being aware of that, some of these questions may be personal, but it is important to ask (start
from least offensive to most offensive)
Oral

Any recent travel outside Canada?


Did you eat any raw shell fish? Did you eat in new place that you are not used to?

Surgical

Any history of surgeries / hospitalization?

Any history of blood transfusion?

Any history of blood donation?


Social

How do you support yourself financially? If hazardous occupation?


Did you get exposed to blood products / body fluids?

Risky behaviour

Any tattooing or piercing?

SAD?
o
Do you smoke?
o
Drink alcohol? How about the past?  Drinking assessment
o
Have you ever tried recreational drugs? Any injected drug use? When was the
last time?

With whom do you live? For how long have you been together?

Before being with your current partner, did you have other partners?

When did you start to be sexually active? How many sexual partners did you have from
that time till now?

What is your preference, men, women or both?

What type of sexual activity do you practice? Did you practice safe sex all the time? And
by that I mean using condoms!

Any history of sexually transmitted infections? And screening for STIs?

Have you ever had sex with sex worker?

Within the last 12 months, have you had any other sexual partners?
When do you need to take extensive
sexual history? Risky behaviour!
 Liver enzymes / Jaundice
 Fever / Tired
 LNs
 Discharge
 PAP results
 HIV test results

Liver investigations











OSCE-guide-III.doc

AST / ALT / GGT / Alkaline phosphatase


LDH
Bilirubin
INR / PTT
Albumin
Glucose
Serum ferritin / TIBC / serum ceruloplasmin
Viral serology: Hep A/B/C antibodies and
Hep B Ag
Abdominal US
Liver biopsy

Page 25 of 255

History taking Medicine

Drinking / Alcohol
Drinking assessment

Use / abuse

MOAPS

Drinking hx
How much?
CAGE

Mood
Organic
Anxiety
Psychosis
Self-care / suicide

Impact
Medical
Social
Liver
Home
Memory / conc. Work
B12
Heart

Legal

Use / Abuse:

Do you drink alcohol? How about the past?

What do you drink?


o
For how long?
o
How often?

How much?
o
2 bottles of wine a day? 12 beers a day? Have you drunk more than 6 drinks in
one setting? Have you ever exceeded the amount you intended to drink?
o
Do you drink alone or with other people?
o
Did you ever drink to the extent of black out?
o
What do you feel if you do not drink? Any shaking / heart racing / sweating?
Have you ever had seizures before? Were you hospitalized? Did you have
delirium tremens?
o
Do you avoid going to places where you do not have access to alcohol?

CAGE:
o
Did you ever think that you need to cut down on your drinks?
o
Do you get annoyed by other people criticizing your drinking?
o
Do you feel guilty for your drinking habits?
o
Early morning drink?
Problem drinking: 2 of CAGE list for males OR 1 for females
MOAPS:

Mood:
How is your mood? Interest? If ok  do not proceed
o
If not ok  MI PASS ECG
Anxiety:
o
Are you the person who worries too much?
o
Do you have excessive fears or worries?
Psychosis:
o
Do you hear voices or see things that others do not?
o
Do you think that someone else would like to hurt you?
Self care / suicide
o
Any chance you might harm yourself or somebody else?
o

OSCE-guide-III.doc

Page 26 of 255

History taking Medicine

Impact:
I am going to ask some questions to check what effects does alcohol have on your life?

The medical is already done in the consequences of liver injury

Social:
o
With whom do you live? For how long? How is the relation? Is there any
problems? Is it related to your drinking habits?
o
How do you support yourself financially? Where do you work? For how long?
 How is the relation with your coworkers / manager?
 Do you miss working days because of your drinking habits?
 Do you need to drink at work?

Legal:
o
Did you have any legal issues related to your drinking?
o
Fights? Arguments? Were you arrested before because of drinking?
o
Were you charged before for DUI (driving under influence)?

Alcoholic beverages:

Beer:
o
Alcohol percent around 5%
o
Pitcher (60 oz) = 3 pints (pint = 20 ounces)
o
Ounce (oz) = around 28-30 ml

Wine / Champaign:
o
Alcohol percent around 12%
o
Bottle: 750 ml
o
Glass: 150 ml

Hard liquor (whisky / gen):


o
Alcohol percent around 40%
o
1 glass (shot) = 1 oz (50 ml)
Ethical challenges (patient asks)
Doctor is there any possibility that these test results are wrong? Can we repeat it to make
sure?
 Whenever we do blood work, we take a lot of measures to make sure it is accurate and
usually if it is positive, the labs double check it before they send it. Chances of having a
mistake are very low.
 Based on your history, symptoms and examination, we might not only repeat the test, we
might need to do more tests. It is early to tell now, let us proceed with history and
examination, and at the end I will be able to tell you.
Do you drink yourself, doctor?
 Why do you ask?
 Whether I drink or no does not matter, it is better to discuss your case now.
By the way, are you Canadian graduate, doctor?
 I fulfill all the requirements to practice medicine in Canada!
 I passed all the exams, follow Canadian guidelines, and practice under supervision!

OSCE-guide-III.doc

Page 27 of 255

History taking Medicine

Fever / Tired
Intro
CC

But first I would like to ask you, how do you feel now?

Fever

Do you have other concerns?
Analysis
OsCfD

Did you measure it? How often? How? What is highest?

And medications? Did it help?

Any flu / illness / sickness

Any diurnal variation? Any special pattern? Is it more every 3rd or 4th day?
(malaria)
Impact
Are you able to function?
Red flags
Constitutional symptoms
Differential  CNS: headache / neck pain / stiffness / nausea / vomiting / vision changes /
diagnosis
bothered by light / weakness / numbness
 ENT:

Ears: pain / discharge


Extensive
review of

Nose: runny nose / sinusitis (facial pain)

Throat: sore throat / teeth pain / difficulty swallowing


systems
 Cardiac: chest pain / heart racing (pericarditis)
 Lung (pneumonia, PE (DVT), TB, cancer): cough / blood / phlegm / wheezes /
chest tightness / contact with TB pt
3
 GIT (except the liver ): abd pain / distension / change in bowel movements /
blood in stools
 Urinary: burning / frequency / flank pain / blood in urine
 Do you have any discharge? Ulcers? Blisters? Warts?
 MSK: joint pain / swelling / ulcers in your body / mouth / skin rash / red eye
 Autoimmune: fm hx / dx before with autoimmune dis
 The LIVER:

Local: yellow / itching / dark urine / pale stools

Dx before with liver dis? Screened? Vaccinated?

Transition to risky behaviour


PMH
Cancer / Autoimmune disease
FH
Cancer / Autoimmune disease
SH
Does your partner have any fever? Discharge? Skin rash?




During the last 6 weeks, did you seek medical attention?


What made you choose to come today? Compared to before, any special changes?

The liver will be put at the end as a transition to ask about risky behaviour (see liver enzymes case)

OSCE-guide-III.doc

Page 28 of 255

History taking Medicine

Diarrhea ACUTE
Analysis of
CC

Impact
Red flags

Differential
diagnosis

OsCfD
COCA
B/Mucous

How many times?


What bout during night?

Yes  organic

No  irritable bowel syndrome (IBS) day only

How does if affect your sleep?


 Consistency: watery / loose / formed / bulky. Any floating
fat droplets / difficult to flush / undigested food
 Did you notice blood? When did it start?

Before you have your bowel move?

Mixed (higher source of bleeding)?

On the surface?
AS
 Pain  OCD / PQRST

If pain improves after bowel movement: IBS


 Vomiting
 Alteration with constipation
 Acute dehydration: thirsty / dizziness / light headedness / LOC / weak
 Constitutional symptoms for infection / cancer
 Flu like symptoms
 Any body around you have the same diarrhea?
 Other causes:

Hyperthyroidisms: heat intolerance

Stress? What do you do for life? Any stress? Does the diarrhea with
stress? How about your mood?

Infectious: travel / camping / with whom do you live? Any other person
at home with diarrhea?

HIV if risk factors

Diet: Celiac disease / a lot of dairy products / lactose intolerance / lots of


juice / sugars

Medications: antibiotics / stool softeners


 Rheumatic diseases: red eyes / mouth ulcers / skin changes/ rash / nail
changes / hx of psoriasis / joint pain / swelling / back pain / stiffness
(especially in morning) / discharge / renal stones



PMH
FH
SH
Bloody diarrhea DD:
GE (gastroenteritis)
IBD (inflammatory bowel disease)
Bleeding peptic ulcer
Investigations for clostridium difficile
 CBC / differential / lytes and chemistry
 Stool culture for parasites
 Stool assay for clostridium toxin
 Endoscopy
 Blood grouping and cross matching

OSCE-guide-III.doc

Treatment for clostridium difficile


 Stop the antibiotic
 Metronidazole (500 mg tid x 7 days)
 If metronidazole is not effective or severe
case  vancomycin (125 mg qid x 14 d)

Page 29 of 255

History taking Medicine

Diarrhea CHRONIC
The same as acute diarrhea, except the impact and red flags
Introduction
CC
Analysis of
CC

Impact
Red flags

Differential
diagnosis

OsCfD
COCA
B/Mucous

How many times?


What bout during night?

Yes  organic

No  irritable bowel syndrome (IBS) day only

How does if affect your sleep?


 Consistency: watery / loose / formed / bulky. Any floating
fat droplets / difficult to flush / undigested food
 Did you notice blood? When did it start?

Before you have your bowel move?

Mixed (higher source of bleeding)?

On the surface?
 Pain  OCD / PQRST
AS

If pain improves after bowel movement: IBS


 Vomiting
 Alteration with constipation
 Acute dehydration: thirsty / dizziness / light headedness / LOC / weak
 Chronic  weight loss
 Constitutional symptoms for infection / cancer
 For cancer: Age / family hx of Ca colon / change in the calibre of stools /
what kind of diet
 Rheumatic diseases: red eyes / mouth ulcers / skin changes/ rash / nail
changes / hx of psoriasis / joint pain / swelling / back pain / stiffness
(especially in morning) / discharge / renal stones
 Other causes:

Hyperthyroidisms: heat intolerance

Stress? What do you do for life? Any stress? Does the diarrhea with
stress? How about your mood?

Infectious: travel / camping / with whom do you live? Any other person
at home with diarrhea?

HIV if risk factors

Diet: Celiac disease / a lot of dairy products / lactose intolerance / lots of


juice / sugars

Medications: antibiotics / stool softeners





PMH
FH
SH
Rheumatic diseases: IBS / ankylosing spondylitis / psoriasis / reactive arthritis

OSCE-guide-III.doc

Page 30 of 255

History taking Medicine

Counselling:

Explanation:
o
From what you have told me, the most likely explanation for your diarrhea is the
medical condition known as Irritable Bowel Syndrome, it is like unhappy colon
o
What do you know about IBS?
o
We do not know the exact mechanism behind this disease, and it is a common
condition, a lot of people have it, this is a long term disease, but it is treatable.
o
What I need to do is to do physical exam, and do some blood works and stool
analysis to rule out other causes, how do you think about that?
o
Is it serious condition doctor?
 It is not serious, as it does not affect life expectancy, and around 80% of
patients improve over time

Management:
o
Psychotherapy:
 Establish good relationship with the patient
 CBT (cognitive behavioural therapy)
 If mood is low  depression counselling, it might be a mood problem
o
Life style modifications:
 Stress management and relief
Relaxation techniques such as meditation
Physical activities such as yoga or tai chi
Regular exercise such as swimming, walking or running
 Diet modification: lactose-free diet or a diet restricting fructose is sometimes
recommended
 If drinks too much alcohol  advise to decrease alcohol
o
Medications
 Abdominal pain:
Hyoscyamine (antispasmodic): 0.125 to 0.25 mg PO or SL q4h or PRN
/OR/ extended-release tablets: 0.375 to 0.75 mg orally every 12 hours
(do not exceed 1.5mg in 24 hours)
Amitriptyline (10 mg qhs)
 Diarrhea:
Imodium up to 8 tab / day
Lomotil
 Constipation:
fibre content in diet
Metamucil (psyllium): bulk-producing laxative and fibre supplement
 SSRIs
o
Alternative medicine:
Probiotics
Herbal remedies, e.g. peppermint oil:

Offer more information:


o
I will give you some brochures and web sites in case you want to read more
about that
Associated diseases:
- IBS
- Fibromyalgia
- Chronic fatigue syndrome
- Interstitial cystitis

OSCE-guide-III.doc

Page 31 of 255

History taking Medicine

ASTHMA
Mr comes to your office as post-ER visit follow-up, he had asthmatic attack three days ago.
He went to ER; he was treated and discharged with advice to see his family physician.
Introduction
EVENT

Asthma history

Triggers

Infection
Medications

Outdoor

Indoor

Stress
PMH and FH

How do you feel now?


 O S Cf D
 Which medication was used? How many times did you need to puff?
 Symptoms: SOB / Tightness / Wheezes / Sweating / heart racing /
LOC / did you turn blue? Were you able to talk?
 Did you call 911 or someone called for you? Did they give you meds?
What were these meds?
 Were you admitted to hospital? ER? Did they need to put a tube?
What were the discharge meds?
 When were you diagnosed? How? Type of buffers?
 Were you controlled? How many times do you puff (excluding
exercise)? Are you using spacer?
 Recently, did you notice a need to the doses?
 Any attacks during the night?
 Do you use peak flow meter?
 Did you have PFTs (pulmonary function tests) done?
 How many times did you have to go to ER?
 Recent chest infection? Flu-like symptoms? Fever / chills?
 How do you use puffers? Stored properly? Not expired?
 Did you start new medication? -blockers? Aspirin? Any recent in
dose of these medications?
 Exercise
 Cold air
 Pollens (is it seasonal?)
 Dust: construction / smug (smoke/ fog/ exhaust)
 Do you smoke? Anybody around you?
 Do you have pets? People around you?
 Fabrics related: carpets floor? Any change in linen? Pillows?
Blankets? Mattress? Curtains?
 Relation to any type of food?
 Perfumes
 Do you live in a house (basement  mold)?
 Any construction renovation? Exposure to chemicals?
 Any new stressful situations?
 Skin allergies
 Other allergies

Asthma Management
1- Confirm diagnosis:

Symptoms:
o
Cough (dry / more at night / more with exercise / induced by allergens)
o
Wheezes (noisy breathing)
o
Chest tightness

Examination: wheezes

Diagnosis:
o
Chest x-ray: R/O pneumonia / infection / cancer
o
Pulmonary Function Tests (PFTs):
 FEV1/FVC < 80% of expected  obstructive lung disease
 Give bronchodilators, repeat PFTs after 20 min, if > 12%  Asthma

OSCE-guide-III.doc

Page 32 of 255

History taking Medicine

2- Management:

Environment control: avoidance of irritant and allergic triggers (e.g. avoid smoking /
change -blocker for treatment of HTN)

Patient education: the allergic nature of the disease and triggering factors

Written action plan: see the diagram below (next page)


3- Medications:
Type
Mild
intermittent

Symptoms
< 2 times / week

Mild
persistent

> 2 times / week


but < 1 time / day

Moderate

Daily

Severe

Continuous /
Uncontrolled

Treatment
Short acting 2-agonist:
1-2 puffs (PRN and
before exercise)
Short acting 2-agonist
(Ventolin 100 mcg 12 puffs qid)
LABA
(Serevent 50 mcg 1
puff bid)
Add LABA or LTRA
(Singulair 10 mg PO
qhs)

Notes
Does not need daily
medication
Low dose ICS
(Flovent 125
mcg 1 puff bid)
Moderate dose
ICS
(Flovent 250
mcg 1 puff bid)
High dose ICS
(Flovent 250
mcg 2 puffs
bid)

Oral
prednisone

LTRAs are second-line


monotherapy for mild
asthma
6-11 yrs: ICS should be
to moderate dose
> 12 yrs: LABA should be
considered first
Omalizumab (anti IgE)
may be considered in
patients > 12 yrs

ICS
: Inhaled Corticosteroids; 1 puff = 100 mcg
LABA : Long-acting beta2-adrenoceptor agonist
LTRA : Leukotriene receptor antagonist

Puffers and LTRA tablets:


Medication
Ventolin
Serevent
Flovent

Color
Blue
Blue
Orange

Active ingredient
Salbutamol
Salmeterol
Fluticasone propionate

Advair

Purple

Pulmicort

Brown

Fluticasone
Salmeterol
Budesonide

Symbicort

Red

Atrovent
Spiriva
Singulair

Budesonide
Formoterol
Green
Ipratropium bromide
Spiriva Tiotropium bromide
Handihaler
Tablets Montelukast

/ puff
100 mcg
50 mcg
125 mcg
250 mcg
500 mcg
250 mcg
50 mcg
100 mcg
200 mcg
400 mcg
400 mcg
12 mcg
20 mcg
18 mcg
4 mg
10 mg

Class
Short acting 2-agonist
LABA
ICS

Notes

ICS
LABA
ICS

Also available as diskus

ICS
LABA
Anti-cholinergic
Long-acting, 24 hrs, anticholinergic bronchodilator
Leukotriene receptor
antagonist (LTRA)

Also available as diskus

For Asthma and COPD


For COPD

4- Emergency treatment of Asthma:

Oxygen, aim for SaO2 > 94%

Bronchodilators:
o
Beta 2 agonists: salbutamol 100 mcg 4 puffs q 15-20 min x 3 AND
o
Ipratropium bromide 4 puffs q 15-20 min x 3

Corticosteroids: hydrocortisone 200 mg IV or prednisone 40-60 mg PO

Rehydration: aggressive IV fluids to liquefy bronchial secretions

OSCE-guide-III.doc

Page 33 of 255

History taking Medicine

Canadian Thoracic Society Asthma Management Continuum 2010

Asthma action plan:

OSCE-guide-III.doc

Page 34 of 255

History taking Medicine

COPD management
Prolong survival

Smoking cessation
Vaccination: influenza virus, pneumococcus (Pneumovax)
Home oxygen: to prevent cor pulmonale and decrease
mortality if used > 15 hrs/day (indications: PaO2 < 55
mmHg; or < 60 mmHg with cor pulmonale or
polycythemia)
Ventolin (q6h PRN) + Atrovent (1-2 puffs q6h)
 LABA (Serevent 50 mcg/dose) Atrovent /or/
 LACA (Spiriva): 18 mcg qAM + must stop Atrovent




Bronchodilators Short acting


Long acting
Respiratory rehabilitation
Corticosteroids Inhaled

Combination of ICS + LABA


Advair: Fluticasone / Salmeterol (250mcg/50mcg) od or
bid
 Symbicort: Budesonide / Formoterol (400mcg/12mcg) 2
puffs bid
Short course of oral corticosteroids: 50mg/d x 5 days


Oral

COPDE (COPD exacerbation):


 Definition: episode of increased dyspnea, coughing, increase in sputum volume or purulence
 Etiology: viral URTI, bacteria, air pollution, CHF, PE, Ml must be considered
 Management:

Assess ABC, consider assisted ventilation if decreasing LOC or poor ABGs

Supplemental O2 (controlled FiO2: target 88-92% SaO2 for CO2 retainers

Bronchodilators by nebulizer
o
Short acting beta2-agonists used concurrently with anti-cholinergics
o
Salbutamol and Ipratropium bromide via nebulizers x 3 back-to-back

Systemic corticosteroids: IV solumedrol or oral prednisone (50mg/d x 5 days)

Antibiotics: often used to treat precipitating infection:


o
Indications (2 out of 3): increased SOB, increased sputum amount, or increased
sputum purulence (change in colour, e.g. greenish)
o
Type of antibiotics: see below

Post exacerbation: rehab with general conditioning to improve exercise tolerance


 ICU admission

For life threatening exacerbations

Ventilator support
o
Non-invasive: NIPPY, BiPAP
o
Conventional mechanical ventilation
Antibiotics:

Outpatient: resp fluoroquinolones: levofloxacin 750 mg PO q24h x 5 days OR


beta-lactam + macrolide (amoxicillin 1000 mg PO tid + clarithromycin 500 mg PO bid)

Risk factors (group home / hospital infection / immunocompromised):


ceftriaxone (1 g IV q24h) + azithromycin (500 mg IV q24h x 5 days).
Step-down to oral therapy when tolerated

Susceptible for pseudomonas / recent use (within 3 months) of antibiotics or cortisone:


piptazo (piperacillin / tazobactam); 3.375 gm IV q6h)

MRSA: Vancomycin 1 gm IV q24h

OSCE-guide-III.doc

Page 35 of 255

History taking Medicine

Ankle swelling Bilateral


Introduction
CC
Analysis of
CC

Impact
Red flags
Differential
diagnosis

uni- vs. bi- lateral


OsCfD
 What ? Walking / standing
what ? Raising legs
 How high does it go?

AS
 Local symptoms:

Pain / fullness / heaviness / tightness

Skin changes (redness / swelling / do you feel your feet warm?)

Nail changes
 Other swellings in your body:

How about swelling in your face? Eye puffiness? Do you find it


difficult to open your eyes in the morning?

How about your belly? Did you need to the size of your belt?

Hands, did you feel it is tight to wear your ring?


 How does this affect your life?
 Constitutional symptoms for infection / cancer
Differential diagnosis of BILATERAL ankle swelling:
 Failure Heart
 Failure Liver
 Failure Kidney: history of kidney disease (changes in urine / bruising /
frequency / burning / frothy urine / clear or no)
 Hypoalbuminemia
 Thyroid diseases
Specific cause within this system (e.g. kidney)
 Hx or Dx of DM
 Any medications (penicellamine, gold, NSAIDs, )
 Recent sore throat
 Any skin infection / rash
 Hx of autoimmune disease
 How about diet? Is it balanced? Any diarrhea?

PMH
FH
SH
Case: patient with face swelling, BP 150/90, protein in urine, ketones, no blood, no glucose, no
WBCs
Diagnosis: nephritic syndrome (minimal changes)
Investigations:

Kidney function tests / urinalysis / 24 hrs protein in urine / renal biopsy

Lipid profile / blood glucose studies

Hepatitis B serology / ANA / C3 and C4


Management:

Salt restriction / avoid fats

Diuretics / monitor fluids in and out

Anti-HTN: ACE inhibitors

Prednisolone

OSCE-guide-III.doc

Page 36 of 255

History taking Medicine

Ankle swelling Unilateral


Introduction
CC
Analysis of
CC

Impact
Red flags
Differential
diagnosis

uni- vs. bi- lateral


OsCfD
 What / ?
 How high does it go?

 If pain  PQRST
AS
 Local symptoms:

Pain / fullness / heaviness / tightness

Skin changes (redness / swelling / do you feel your feet warm?)

Nail changes
 Other joints? Toes? Other ankle?
 How does this affect your life?
 Constitutional symptoms for infection / cancer
Differential diagnosis of UNILATERAL ankle swelling:
 Any trauma, any twist in your ankle?
 Gout; previous attacks, screen kidney  for kidney stones
 Infection, sepsis, cellulitis; fever, pus, discharge, tenderness
 Gonorrhea septic arthritis; Sexual history, penile discharge? Unprotected sex
recently?
 DVT
Specific cause within this system (e.g. gout)
 Tell me more about your diet? Too much protein?
 How about alcohol?
 Medications? Pain meds (aspirin) / diuretics (furosemide, thiazides)?
 Hx of cancer / chemotherapy (cytotoxic drugs) / radiation?
 Family hx of gout / kidney stones?

PMH
FH
SH
DVT: see the physical examination section

OSCE-guide-III.doc

Page 37 of 255

History taking Medicine

Congestive heart failure CHF


68 years old man comes to ER with 4 weeks of SOB
Analysis of
CC

Impact

Red flags
DD

PMH
FH
SH

Clarification

1- When do you say SOB; what do you mean? Cardiac or chest?

Is it difficult to breathe in and out?  cardiac / anemia

Is it difficult to breathe out?  COPD / asthma


2- Do you have any hx of asthma? Lung disease?

Any wheezes? Chest tightness? Cough?


3- Do you have any hx of heart disease?

No  newly dx

Yes  ? acute on top of CHF

Any racing heart? Dizziness? LOC? Any hx of HTN?


 Is it first time? Or you had it before? When and how were you
OSCfD
PQRST
diagnosed? How about treatment?

 Is it related to activity? How many blocks were you able to walk?


And now?
 How about at rest? And at night?
 Left ventricle:

SOB? How many pillows do you use?

Do you wake up at night gasping for air?

Cough / crackles?
 Right ventricle:

Any swelling in your LL? How high does it go? Related to position / standing?
Weight gain?

Eye puffiness? Swollen face? Pain on the liver?


 Other cardiac symptoms:

Chest pain? Nausea/vomiting? Sweating?

Heart racing / dizziness / LOC? Do you feel tired?


 Constitutional symptoms for infection / cancer
 Risk factors for ischemic heart diseases IHD
Causes (that precipitated acute on top of CHF):
 Compliance
 Diet
 Medical
DM / Kidney / Liver diseases
HTN / heart attacks
SAD

DD (Causes that precipitated acute on top of CHF):




Compliance:
Are you receiving treatment? Which medications do you take? How much? For how long? Any
change in medications? Change in dose?

Do you take it on regular basis? Any chance that you may skip one or more doses?

Do you take it by yourself or do you need help?

Did you get your Digoxin level measured before / recently?

Did you start new medication? Rx or (OTC) over the counter? e.g. indomethacin

How about water pills?

Are you under regular F/U? How often? When was the last time? Were you symptoms free at that
time?
Diet:

Do you have special diet? Salt-free diet? Do you monitor that?

Any new changes in diet?

Any chance of salty food, e.g. pickles, canned food, dried meet and fish

OSCE-guide-III.doc

Page 38 of 255

History taking Medicine

Medical:
Do you take medications on regular basis? Any new medication? Advil?
Any hx of thyroid dx, any sweating / diarrhea?
Any hx of heart disease / HTN ( A Fib) / heart attack / CAD (ischemia) / did you feel your
heart bouncing (arrhythmias)? Any congenital or valvular disease / Chest pain / tightness /
dizziness / light headedness / LOC?

Any chest / lung disease (wheezes, cough, chest tightness)

Any kidney disease? Renal failure?

Any bleeding? Anemia?

New York Heart Association functional Classification of heart failure:


 Class I: ordinary physical activity does not cause symptoms of HF
 Class II: comfortable at rest, ordinary physical activity results in symptoms
 Class III: marked limitation of ordinary activity; less than ordinary physical activity  symptoms
 Class IV: inability to carry out any physical activity without discomfort; symptoms may be present at rest
Investigations:
 Labs: CBC / lytes / ABG (arterial blood gases) / glucose / INR / PTT / serial cardiac enzymes (q8h x 3)
/ ECG / fluid balance
 Chest x-ray findings of CHF: (1) Enlarged heart, (2) Upper lobe vascular redistribution, (3) Kerley B
lines (thin linear pulmonary opacities caused by fluid or cellular infiltration into the interstitium of the
lungs), (4) Bilateral interstitial infiltrates, (5) Bilateral small effusions
Treatment:

Acute heart failure:


o
Treat acute precipitating factors (e.g. ischemia. arrhythmias)
o
L
Lasix (diuretics)  pre-load (furosemide: 40-500 mg IV)
o
M
Morphine; 2-4 mg IV decreases anxiety and preload (venodilation)
o
N
Nitrates (venous and arterial dilator  kidney perfusion)
o
O
Oxygen
o
P
Positive airway pressure (CPAP/BiPAP) decreases preload and need for
ventilation / Position (sit patient up with legs hanging down unless hypotensive)
o
In ICU or failure of LMNOP: sympathomimetics (dopamine or dobutamine)

Chronic heart failure (long term management):


o
ACEI (slow progression and improve survival) or ARBs (if ACEI not tolerated)
o
Beta blockers: slow progression and improve survival
 Should be used cautiously, titrate slowly because may initially worsen CHF
 Side effects: fatigue / bradycardia
 If pt on -blockers  exacerbation  stop the -blockers for 2 days
o
Digoxin (if A Fib OR symptomatic on ACEI)
o
Diuretics: symptom control, management of fluid overload; furosemide 80 mg OD
(furosemide opposes the hyperkalemia induced by beta-blockers, ACEIs)
Spironolactone for class Ill-b and IV CHF already on ACEI and loop diuretic
If still uncompensated: Implantable Cardioverter Defibrillator (ICD)
o
Anti-arrhythmic drugs: for use in CHF with arrhythmia can use amiodarone, betablocker, or digoxin
o
Anticoagulants: warfarin for prevention of thromboembolic events
Digoxin overdose:

Anorexia, nausea, vomiting

Bradycardia, dizziness, LOC

ECG: PVC, heart block

Vision: yellow hallos around objects

OSCE-guide-III.doc

Page 39 of 255

History taking Medicine

Heart racing
For few weeks
Introduction But first I would like to ask you, at the moment, how do you feel?
CC
Analysis of
Clarification When do you say your heart is racing; what do you mean?

Do you feel your heart is going fast


CC

Or is skipping beats

Can you tap it for me please?

It sounds irregular for me!


 Is it first time? Or you had it before? When and how were you
OSCfD
PQRST
diagnosed? How about treatment?
 Is it related to activity? How many blocks were you able to

walk? And now?


 How about at rest? And at night?
 When was the last attack? And what is the duration of the
longest attack?
 Is it related to caffeine, chocolate, coke, any other type of
food?
 SAD (cocaine or any other stimulant)
Impact
 CVA (any weakness / numbness / difficulty finding words / visual problems)
 Heart failure (SOB / limitation of activity / swelling in your legs / how many
pillows do you use??
 Other cardiac symptoms:

Chest pain? Nausea/vomiting? Sweating?

Heart racing / dizziness / LOC? Do you feel tired?


Red flags
 Constitutional symptoms for infection / cancer
 Risk factors for ischemic heart diseases IHD
DD
 Do you take medications on regular basis? Any new medication?
 Any hx of thyroid dx, any sweating / diarrhea?
 Any hx of heart disease / HTN ( A Fib) / heart attack / CAD (ischemia) /
did you feel your heart bouncing (arrhythmias)? Any congenital or valvular
disease / Chest pain / tightness / dizziness / light headedness / LOC?
 Any chest / lung disease (wheezes, cough, chest tightness)
 Any kidney disease? Renal failure?
 Any bleeding? Anemia?
PMH
DM / Kidney / Liver diseases
FH
 Family history of sudden death at a young age? (cardiomyopathy)
 HTN / heart attacks
SH
SAD
Physical
 Vitals
examination  Cardiac exam (looking for mid-diastolic, mitral stenosis, rumbling character)
 Thyroid exam
 Neurological exam: brief / gross motor and reflexes

OSCE-guide-III.doc

Page 40 of 255

History taking Medicine

The patient daughter has a concern: my mother was diagnosed with AF,
Should I worry about this?
1.

2.

This is a reasonable concern?



AF may lead to embolic event (CVA)

AF may lead to heart failure

AF may lead to V. Fib
However, this is not uncommon condition, and it is treatable with medications

Causes for AF:


1- IHD
2- Hypertension / CHF
3- Hyper-thyroidism
4- Medications (e.g. digoxin, some antiarrhythmic meds class I)
5- Electrolytes imbalance
6- Too much alcohol (holiday heart)
7- Cardiomyopathy
8- Valvular heart diseases
9- Congenital heart diseases
10- Loan AF
11- Myocarditis
12- COPD / pneumonia
13- Cardiac surgery
14- Cocaine
15- Any condition that lead to tachycardia in a
susceptible person

Causes for TACHYCARDIA


1- Exercise
2- Pregnancy
3- Caffeine / stimulants
4- Anemia
5- Hypovolemia
6- Fever
7- Stress
8- Smoking
9- Hyper-thyroidism
10- Pheochromocytoma

Atrial fibrillation
Stable
< 48 hours
If in doubt  TEE

> 48 hours
 Rate control:

-blockers

Ca ch blockers (diltiazem, verapamil)

Digoxin (in patients with heart


failure)
 Anti-coagulation (3 weeks prior and 4
weeks after cardioversion)

Unstable





SOB
BP < 90/60
Chest pain
Confusion

Cardioversion:
 Electrical: 150 joules for A Fib (50 joules for A Flutter)
 Pharmacological: procainamide; 1 g / 1 hr infusion
Anti-coagulation:
Assess stroke risk: determine CHADS2 score in patients with non-valvular AF
Risk factor
Points
CHADS2 score
Anti-coagulation
CHF
1
0-1
Aspirin 81-325 mg daily
Hypertension
1
2 moderate risk
Warfarin
factors or any high risk
Age > 75 yrs
1
factor (prior stroke,
Diabetes
1
TIA or embolism,
Stroke / TIA
2
mitral stenosis,
prosthetic valve)

OSCE-guide-III.doc

Page 41 of 255

History taking Medicine

Fall
Orthostatic hypotension
76 years old male patient came to clinic because he fell few days ago. He was getting out of bed,
when he fell to the ground
Introduction
HPI: analysis of CC
Associated
Symptoms

For any
Fall,
LOC
or seizure

During

Before

After

Impact
Red flags

DD

OSCE-guide-III.doc

Did you hurt yourself? How do you feel?


Was this the first time, or did it happen before?
Was it related to Emotions? Coughing? Urination?
Did you lose conscious? Did you hit your head?
Were you alone or with someone? Did your wife describe it to
you? Is she with you? If it is ok with you, after we finish, I would
like to speak with her to get some info.
 Were you shaking? Certain part of your body or whole?
 Were you breathing? Did you turn blue?
 Did you bite your tongue? Roll your eyes? Wet yourself?
 Were you able to take few steps or did you fall immediately
(orthostatic hypotension)?
 Before you lose consciousness, did you feel:

Dizzy, lightheaded, nausea, vomiting, any chest pain, heart


racing (cardiac)

Things are spinning around you (vertigo)

Weakness, numbness, vision changes (stroke)

Any flashing light, strange smells (epilepsy)

Sweaty, shaky, hungry (hypoglycemia)


 How long did it last?
 How did you regain consciousness? By yourself or did you need
intervention?
 After you regain consciousness; were you able to recognize the
surroundings? Able to talk? Able to move?
 Did you feel any weakness, numbness?
 Did you hurt yourself? How do you feel now?
 Constitutional symptoms
 Risk factor for IHD
Any geriatric
 Balance
patient; ASK about:
 Vision
 Hearing
 Urination
 Diseases: arrhythmia / CAD, CVA, seizure, hypoglycemia
(already analyzed before the event)
 Environment: is your room well lit? Any chance you tripped?
 Do you take any medications? Do you have a list?

Go through it one by one

Which one was added / changed recently?

Each medication: ask about the disease, when started?

Any OTC? Aspirin? Who prescribed it to you?


 Are you getting enough fluids






Page 42 of 255

History taking Medicine

Counselling:
 Inform the patient

The most likely explanation to what happened is a condition called postural orthostatic
hypotension. It means drop in the blood pressure with change of posture.

Explain the pathophysiology:


o
When we change position from lying or sitting to standing, blood tends to pool in
the lower extremities, and this leads to drop in blood pressure. Normally, blood
vessels in our body react by narrowing in order to prevent this and to maintain
normal blood pressure.
o
In patients having orthostatic hypotension, and this could be due to age /
medications / DM or combination, their blood vessels fail to react fast enough,
this leads to pooling of blood in lower extremities  amount of blood reaching
to heart  blood reaching the brain  they end-up losing their consciousness
temporarily.

Consequences: this might happen again

Investigations:
o
Blood works / CBC / differential / lytes / kidney and liver function tests
o
ECG
 Preventive measure:

Contact the psychiatrist to check the poly-pharmacy, to discuss with him the possibility
of decreasing the dose or changing medications.

Meanwhile, if you are changing positions, do this slowly, on steps, e.g. from lying down,
sit for a couple of minutes on the bed before standing up, and before you stand up, push
your feet against the ground for few seconds.
 I will give you brochures and web sites in case you need to read more.

Notes:
 The patient will have a list of medications:

Lipitor

Hydrochlorothiazide  ask about fluids intake

-blocker

Aspirin  ask about bleeding

Lorazepam

Oxazepam  I can see that you are taking 2 sleeping pills, who prescribed them to you?
The same doctor or no?

Metformin

B12 / B complex


If the patient looks sad / depressed  you look down for me, any chance you are depressed

OSCE-guide-III.doc

Page 43 of 255

History taking Medicine

Peripheral vascular disease:


Calf pain / swelling
Introduction
Analysis of
CC

Impact

Red flags

DD

PMH
FH
SH

OsCfD
PQRST

P: unilateral or bilateral

R: what about other joints, knees? Thighs? Feet?


 What or : did you notice that your pain while walking up or down hill?

while walking uphill: peripheral arterial disease

while walking downhill: spinal stenosis


 Is it first time? Or you had it before? When and how were you diagnosed?
How about treatment?
 Is it related to activity? How many blocks were you able to walk? And
now?
 How about at rest? And at night?
 When was the last attack? And what is the duration of the longest attack?
 History of strokes / TIAs / neurological symptoms
 Chest pain / SOB / heart racing
 Pain after eating (intestinal ischemia)
 Effect of pain on daily activities / work?
 Leriche syndrome (aorto-iliac occlusive disease): numbness in buttocks &
thighs / absent or decreased femoral pulses / impotence
 Constitutional symptoms for infection / cancer
 Risk factors for ischemic heart diseases IHD

Smoking? How much and for how long?

High blood pressure? For how long? Controlled or not?

Diabetes mellitus

Cholesterol measured? When? What was it?


Peripheral Arterial Disease versus Spinal Canal Stenosis
Vascular symptoms
Neuro symptoms
 Cold feet / ulcers
 Weakness / numbness / tingling
 Swelling / redness
 Back trauma / back pain
 Delayed wound healing
 Sexual dysfunction / difficulty with
 Nail changes / hair loss
erection
Past history of heart disease / stroke / symptoms of stroke / DM / Kidney / Liver
diseases
Family history of heart disease / HTN / heart attacks
SAD



N.B. the 6 Ps of ischemia:


Pallor / Pain / Parathesia / Paralysis / Pulseless / Polar (cold)

OSCE-guide-III.doc

Page 44 of 255

History taking Medicine

Urinary symptoms:


Obstructive (prostatic disease in  anuria):


Difficulty to initiate urine? Do you need to strain?

Any changes in the stream?

Any dripping?

After you pass urine, do you feel that you emptied your bladder completely or do you
need to go again?
Irritative (frequency  UB disease):

How many times do you go to the washroom?


o
How about before? Any change?
o
How about during night time? How does this affect your sleep? How about your
concentration and mood?

Do you need to rush to washroom? Are you able to make it all the time?

Have you ever lost control or wet yourself?

Any burning sensation? Any flank pain?

Fever / night sweats / other constitutional symptoms


Urine analysis (changes):

COCA + B (content: frothy / cloudy / not clear)

Summary of irritative symptoms:


FUND
Frequency / Urgency / Night time / Discomfort

OSCE-guide-III.doc

Page 45 of 255

History taking Medicine

Anuria
Introduction
Analysis of CC

Impact

Red flags

DD

PMH
FH
SH

Empathy how do you feel right now?


 OsCfD
 PQRST
 What or
 Is it first time? Or did it happen before? When and how were you diagnosed? How
about treatment?
Associated symptoms:
 Obstructive symptoms
 Irritative symptoms
 Urine analysis (changes): COCA Blood
Local symptoms:
 Any problems with passing stools? What? When?
 Any masses in the groin / pelvic mass / pain?
 Abdominal pain? Distension?
Metastasis
 Back: pain / weakness / numbness
 Liver: yellow / itchy / urine / stools
 Lungs: cough / phlegm / hemoptysis
 Brain: headache / nausea / vomiting
Renal failure
Generalized swelling / face puffiness / itching
Sexual
Sexual dysfunction
 Constitutional symptoms for infection / cancer
 Risk factors for cancer prostate / bladder

Were you ever diagnosed with prostate disease? Screened for prostate
diseases? (DRE or PSA)

Family history of prostate disease / cancer?

Ca bladder (radiation / exposed to chemicals / aniline dye)

Smoking? Alcohol?
 Renal stones: Have you ever had a renal stone? Any history of colicky pain in
flanks? Have you ever passed a small crystals or stone during voiding? Hx or
repeated UTIs?
 Medications: glaucoma / anti-psychotic meds / anti-cholinergic drugs; like those
used for incontinence; e.g. Ditropan (Oxybutynin), Detrol (Tolterodine)
 2 Neuro:

Back problem: trauma metastasis cauda equine (spoiled himself with stools
/ buttocks numbness)

Stroke (diagnosed / weakness / numbness / difficulty)


 2 Cancer:

Cancer prostate

Ca bladder (hematuria)
AMPLE
DM / anemia / polycystic kidney disease / renal stones
SAD

Most likely diagnosis:


BPH
Other possible diagnoses:
UTI / prostatitis / Ca prostate
Investigations:
urea & creatinine / urinalysis / renal US / DRE & PSA / TRUS
If cancer is suspected: bone scan / CT
Treatment:

Watchful waiting: may resolve spontaneously

Medical treatment: -adrenergic antagonists (doxazosin, terazosin) / 5--reductase


inhibitors (finasteride)

Surgery: open surgery / TURP / minimally invasive (stent / laser ablation / cryosurgery)

OSCE-guide-III.doc

Page 46 of 255

History taking Medicine

Hematuria
Introduction
Analysis of
CC

Impact

Red flags

DD

PMH
FH
SH
Investigations:

Empathy how do you feel right now?


 OsCfD
 Timing:

Initial versus terminal or total

Diurnal variation
 What or
 Painful or Painless
 Is it first time? Or did it happen before? When and how were you
diagnosed? How about treatment?
Associated symptoms:
 Obstructive symptoms

prostate disease
 Irritative symptoms

UB disease
 Urine analysis (changes): COCA Blood
Local symptoms:
 Any problems with passing stools? What? When?
 Any masses in the groin / pelvic mass / pain?
 Abdominal pain? Distension?
Metastasis
Renal failure
Generalized swelling / face puffiness / itching
Sexual
Sexual dysfunction
 Constitutional symptoms for infection / cancer
 Risk factors for cancer prostate / bladder / RENAL

Were you ever diagnosed with prostate disease? Screened for prostate
diseases? (DRE or PSA)

Family history of prostate disease / cancer?

Family history of cancer bladder or kidney?

Ca bladder (radiation / exposed to chemicals / aniline dye)

Smoking? Alcohol?
 Renal stones: Have you ever had a renal stone? Any history of colicky pain
in flanks? Have you ever passed a small crystals or stone during voiding?
Hx or repeated UTIs?
 Medications: blood thinners / aspirin / bleeding from other sites?
 Pseudo-hematuria:

Diet: eating too much beet

Medications: Rifampicin

Other bleeding: bleeding per rectum / vaginal bleeding


 AMPLE
 History of hemolytic anemia / polycystic kidney
DM / anemia / polycystic kidney disease / renal stones
SAD

(1) Kidney: urinalysis (casts / crystals / C&S / cytology) / ultrasound (abd/pelvic) / IVP / KFTs
(2) Bladder: cystoscopy
(3) Prostate: PSA / TRUS
(4) Others: CBC / differential / INR
Case: patient on warfarin for A. fib for 2 yrs; went to walk in clinic for sore throat and was prescribed
Biaxin, developed hematuria. Diagnosis: coagulopathy.

OSCE-guide-III.doc

Page 47 of 255

History taking Medicine

Renal stones
Risk Factors

Hereditary: RTA, G6PD, cystinuria, xanthinuria, oxaluria, etc.

Dietary excess: Vitamin C, oxalate, purines, calcium

Dehydration (especially in summer months)

Sedentary lifestyle

Medications: thiazides

UTI (with urea-splitting organisms)

Hypercalcemia disorders: hyperparathyroidism, sarcoidosis, histoplasmosis, etc.


Investigations

Screening labs
o
CBC: elevated WBC in presence of fever suggests infection
o
Electrolytes, Cr, BUN  to assess renal function
o
Urinalysis: R&M (WBCs, RBCs, crystals), C&S

Imaging
o
Kidneys, ureters, bladders (KUB) x-ray to differentiate opaque from non-opaque
stones (e.g. uric acid, indinavir) / 90% of stones are radiopaque
o
CT scan: no contrast; distinguish radiolucent stone from soft tissue filling defect
o
Abdominal ultrasound: may demonstrate stone (difficult for ureters) / may
demonstrate hydronephrosis
o
IVP (not usually done): anatomy of urine collecting system, degree of
obstruction, extravasation

Cystoscopy for suspected bladder stone

Strain all urine  stone analysis

If recurrent stone formers, conduct metabolic studies


o
Serum electrolytes, Ca, PO4, uric acid, creatinine and urea
o
PTH if hypercalcemic
Approach to renal stone:

OSCE-guide-III.doc

Page 48 of 255

History taking Medicine

Treatment Acute:

Medical:
o
Analgesics (Tylenol #3)
o
NSAIDs help lower intra-ureteral pressure
o
antibiotics for UTI
o
(antiemetic + IV fluids) for vomiting

Interventional:
o
Ureteric stent (cystoscopy)
o
Percutaneous nephrostomy (image-guided)

Admit if necessary:
o
Intractable pain
o
Intractable vomiting
o
Fever (? infection)
o
Compromised renal function
o
Single kidney with ureteric obstruction / bilateral obstructing stones
Treatment Elective:

Medical:
o
Conservative if stone < 5 mm and no complications
o
Fluids to increase urine volume to > 2 L/day (3-4 L if cystine)
o
Specific to stone type:
 Calcium oxalate stones: thiazides / potassium citrate (alkalinization of urine)
 Calcium struvite: antibiotics for 6 wks (stone must be removed to treat infection)
 Uric acid: allopurinol / potassium citrate (alkalinization of urine to pH 6.5 to 7) /
shockwave lithotripsy not effective
 Cystine: alkalinize urine (bicarbonate / potassium citrate) / penicellamine / captopril
(forms complex with cystine) / shockwave lithotripsy not effective

Interventional:
o
Procedural / surgical: If stone is > 5 mm or presence of complication
o
Kidney
 Extracorporeal shockwave lithotripsy (ESWL) if stone < 2.5 cm
 Percutaneous nephrolithotomy; indications:
+ Size > 2.5 cm
+ Staghorn
+ UPJ obstruction
+ Calyceal diverticulum
+ Cystine stones
o
Ureter
 ESWL is the primary modality of treatment
 Ureteroscopy (extraction or fragmentation) if failed ESWL / Ureteric stricture
o
Bladder
 Transurethral cystolitholapaxy
 Remove outflow obstruction (TURP or stricture dilatation}
Management of UTI:

Investigations:
o
Urine for culture and sensitivity
o
Blood: CBC / differential
o
Imaging (if suspect complicated pyelonephritis or symptoms do not improve with 72
hours of treatment): Abd/pelvic U/S / IVP / Cystoscopy / CT

Pregnant: amoxicillin 500 mg TID x 7 days

Non-pregnant:
o
Septra (sulfamethoxazole and trimethoprim) DS (800/160): 1 tab bid x 7 days
o
/OR/ Ciprofloxacin 500 mg bid x 7 days

Pyelonephritis:
o
Ceftriaxone (third-generation cephalosporins): 1 g IV q24hrs x 2 days
o
Then continue oral ciprofloxacin x 7 days

Abscess: + drain

OSCE-guide-III.doc

Page 49 of 255

History taking Medicine

Incontinence
Obstructive / 62 years old female, with hx of 3 years of urinary incontinence
Introduction
Empathy how do you feel right now?
Analysis of
 OsCfD
CC
 What or : lifting objects / coughing / straining
 Is it first time? Or did it happen before? When and how were you
diagnosed? How about treatment?

Impact
Red flags

DD

Associated symptoms:
If at any time there is a frequency or
 Obstructive symptoms
some new symptom  analyze it
 Irritative symptoms
first then resume!
 Urine analysis (changes): COCA
Frequency in  UTI
Blood
Local symptoms:
 Any problems with passing stools? What? When?
 Any masses in the groin / pelvic mass / pain?
 Any perineal skin lesions?
 How does it affect your life? Daily activities?
 Constitutional symptoms for infection / cancer
 Risk factors (MGOS):
 Menopausal symptoms, and HRT
M
 LMP
 Gynaecological history
G
 Previous abdominal or pelvic surgeries
 Obstetric: How many pregnancies? Route of delivery?
O
 Sexual: Repeated infections / dryness / dyspareunia
S
 Overflow incontinence
 Urge incontinence

Detrusor overactivity: CNS lesion, inflammation / infection (cystitis),
bladder neck obstruction (tumour, stone)
 Stress incontinence

Urethral hypermobility: childbirth, pelvic surgery, aging

Intrinsic sphincter deficiency (ISD): pelvic surgery, neurologic
problem, aging and hypoestrogen state

Diagnosis:

History

Urinalysis + C&S (if infection suspected)

Urodynamics

Stress test
Treatment of urge incontinence
Treatment of stress incontinence

Bladder habit training

Weight loss

Botox (botulinum toxin) injection

Kegels exercises

Medications: anti-cholinergics;

Bulking agents
Tolterodine (Detrol), Oxybutynin

Surgery (slings, TVT / TOT4, artificial


(Ditropan), TCAs
sphincters)
N.B. Causes of reversible urinary incontinence (DIAPERS): Delirium, Inflammation / Infection, Atrophic
vaginitis, Pharmaceuticals / Psychological, Excess urine output, Restricted mobility, Stool impaction
4

TVT: Tension-free Vaginal Tape, TOT: Trans Obturator Tape

OSCE-guide-III.doc

Page 50 of 255

History taking Medicine

Lump Neck Swelling


Introduction
Analysis of CC:
The lump

Associated (local)
symptoms

Impact

Red flags

Differential
Diagnosis
PMH
FH
Physical exam

Can you point to it?


OSCfD / Anything special at that time? Fever? Rash?
Is it painful? PQRST
Can you estimate its size for me? Is it like a lent, olive, lemon, or
larger? Did it change in size? How fast was the change in size?
 Did you try to feel it? Does it feel soft / rubbery / hard?
 Do you feel it is fixed or moving?
 Any skin changes? Redness? Ulcers?
 Any history of trauma?
 Is it the only one? Did you notice other lumps in your body? How about
other side of your neck? Arm pits? Groins?
 Rule out infection: Any recent flu-like symptoms? Do you feel tired/
fatigue? History of sinusitis/ Pain in your face? Runny nose?
Pain/discharge in ears? Any sore throat/ oral ulcers/ tooth pain?
Difficulty swallowing? Neck stiffness/pain? Headache? Vomiting?
 Thyroid (if central): heat vs. cold intolerance / sweating / hand shaking
/ heart racing / diarrhea vs. constipation
 How does this affect your life?
 Do you feel tired?
? HIV
 Easy bruising? Repeated infections? ? Leukemia
 Constitutional symptoms
 Bone pains / Tender points
HEAD SSS  risky behaviour:
 A: includes recent travel
 SAD: how about injection drugs? Did you share needles?
 Sexual hx: Detailed (safe sex, last time, how many partners). Did you
notice any vaginal discharge/ bleeding? Any pain/ blisters/ warts?
Discoloration/ itchiness?
HIV / Lymphoma / Leukemia / Infectious mono-nucleosis





History of cancer
History of cancer / lymphadenopathy
 Vital signs
 Neck exam / Thyroid exam if the swelling is central
 LNs / Lymphatic system / LNs in groin / pelvic exam
 Liver / Spleen

Notes:

Whenever there is IV drugs  screen for liver symptoms / HIV


Whenever there is risk for STIs  screen for liver symptoms and PID

OSCE-guide-III.doc

Page 51 of 255

History taking Medicine

Lump Breast
Introduction
Analysis of CC:
The lump

Can you point to it?


Is it one breast or both? Where did you notice it? You can ask
verbally, is it LT / RT? Upper / Lower? Outer / Inner? How about
the other breast?




DO NOT POINT WITH YOUR HANDS OR FINGERS!


OSCfD / Anything special at that time? Fever? Rash?
Is it painful? PQRST
Can you estimate its size for me? Is it like a lent, olive, lemon, or
larger? Did it change in size? How fast was the change in size?
 Did you try to feel it? Does it feel soft / rubbery / hard?
 Do you feel it is fixed or sliding (moving)?
 Any skin changes? Redness? Ulcers?
 Any history of trauma?
 Is it the only one? Did you notice other lumps in your body? How about
your neck? Arm pits? Groins?
 Is it related to your period? Does it change with the period?
 Any nipple changes? Discharge? Bleeding? Itching?
Associated (local)  Rule out infection: Any recent flu-like symptoms? Do you feel tired/
fatigue?
symptoms
Impact
 Headache/ vomiting?
(consequences of
 Back pain/ weakness/ numbness/ tingling in arms or legs?
cancer:
 Chest pain/ cough/ phlegm/ wheezes/ heart racing?
metastasis)
 Liver: yellow discoloration/ itching/ urine/ stools?
Red flags
 Constitutional symptoms
 Bone pains / Tender points
Risk factors of cancer: MGO
 Menstrual history: first period / last period / regular?
 G: OCPs?
 Obstetric: History of pregnancies? Number of pregnancies? First
pregnancy at what age?
 Breast feeding?
 Diet rich in fat
 PMH or FH of cancer breast / ovarian carcinoma
Differential
 Benign disease
Diagnosis
 Trauma  fat necrosis
PMH
History of cancer breast / ovarian carcinoma
FH
History of cancer breast / ovarian carcinoma
DD for Breast Mass:

Breast Cancer

Sclerosing adenosis

Fibrocystic changes

Lipoma

Fibroadenoma

Neurofibroma

Fat necrosis

Granulomatous mastitis (e.g. TB,

Papilloma / papillomatosis
sarcoidosis)

Galactocele

Abscess

Duct ectasia

Silicon implant

Ductal / lobular hyperplasia






OSCE-guide-III.doc

Page 52 of 255

History taking Medicine

Galactorrhea (prolactinoma): normal prolactin level: 10-20 ng/ml (non-pregnant)


 If < 40 ng/ml:
follow-up
 If > 40 ng/ml:
CT and bromocriptine
 If progressive / headache / affecting vision  surgery
Benign Breast Lesions:

Non-proliferative lesions:
o
Aka fibrocystic changes, chronic cystic mastitis, mammary dysplasia
 Benign condition characterized by fibrous and cystic changes in the breast. No risk
of breast cancer / Age 30 to menopause / pre-menstrual.
 Breast pain, focal areas of nodularity or cysts often in upper outer quadrant
 Treatment: Evaluation of breast mass and reassurance / Analgesia (ibuprofen, ASA) /
If > 40 years old: mammography every 3 years or biopsy

Proliferative lesions:
o
Fibroadenoma:
 Most common benign breast tumour in women under age 30
 Risk of subsequent breast cancer is increased only if fibroadenoma is complex, there
is adjacent atypia or a strong family history of breast cancer
 Clinical features: nodules: smooth, rubbery, discrete, well-circumscribed, nontender, mobile, hormone-dependent. Needle aspiration yields no fluid
 Investigations: Core or excisional biopsy required
 Treatment: Generally conservative; serial observation. Consider excision if size 2-3
cm, rapidly growing on serial US, if symptomatic or pt preference
o
Intra-ductal Papilloma
 Solitary intra-ductal benign polyp
 Present as nipple discharge (most common cause of spontaneous, unilateral bloody
nipple discharge), breast mass, nodule on U/S
 Treatment: excision of involved duct to ensure no atypia

Other lesions:
o
Fat Necrosis
 Uncommon, result of trauma (may be minor commonly a tight bra, positive history
in only 50%), after breast surgery (i.e. reduction)
 Firm, ill-defined mass with skin or nipple retraction, tenderness
 Regress spontaneously, but complete imaging biopsy to R/O cancer
o
Mammary Duct Ectasia
 Obstruction of a subareolar duct  duct dilation, inflammation, and fibrosis
 May present with nipple discharge, bluish mass under nipple, local pain
 Risk of secondary infection (abscess, mastitis)
 Resolves spontaneously
o
Abscess
 Lactational vs. periductal / subareolar
 Unilateral localized pain, tenderness, erythema, subareolar mass, nipple discharge,
nipple inversion
 Rule out inflammatory carcinoma, as indicated
 Treatment: initially broad-spectrum antibiotics and I&D (incision and drainage), if
persistent total duct excision (definitive)
 If mass does not resolve: fine needle aspiration (FNA) to exclude cancer, U/S to
assess for presence of abscess
Breast Cancer:

1/9 women in Canada will be diagnosed with breast cancer in their lifetime

Risk factors:
 Prior history of breast cancer
 1st degree relative with breast cancer (greater risk if relative was premenopausal)
 Increased risk with high breast density, nulliparity, first pregnancy >30 years old,
early menarche (< 12 yrs), late menopause (> 55 yrs), >5 years HRT
 Decreased risk with lactation, early menopause, early childbirth

OSCE-guide-III.doc

Page 53 of 255

History taking Medicine

Investigations
o
Mammography
 Screening: every 1-2 years for women age 50-69 / If positive family history in 1st
degree relative: every 1-2 years starting 10 years before the youngest age of
presentation
 Diagnostic: investigation of patient complaints (discharge, pain, lump)
 Follow-up after breast cancer surgery
 Findings indicative of malignancy: mass that is poorly defined, spiculated border,
micro-calcifications, architectural distortion, normal mammogram does not rule out
suspicion of cancer based on clinical findings
o
Other radiographic studies:
 Ultrasound differentiates between cystic and solid
 MRI high sensitivity, low specificity
 Galactogram / ductogram (for nipple discharge): identifies lesions in ducts
 Metastatic workup as indicated (usually after surgery or if clinical suspicion of
metastatic disease) bone scan, abd U/S, CXR, head CT
Diagnostic Procedures
o
Needle aspiration: for palpable cystic lesions; send fluid for cytology if blood or cyst
does not completely resolve
o
Fine needle aspiration (FNA): for palpable solid masses; need experienced practitioner
for adequate sampling
o
U/S or mammography guided core needle biopsy (most common)
o
Excisional biopsy: only performed as second choice to core needle biopsy; should not be
done for diagnosis if possible
Genetic Screening: consider testing for BRCA 1/2 if:
o
Patient diagnosed with breast AND ovarian cancer
o
Strong family history of breast / ovarian cancer (e.g. Ashkenazi Jewish)
o
Family history of male breast cancer
o
Young patient ( <35 years old)
Pathology
o
Non-invasive: ductal carcinoma in situ (DCIS): completely contained within breast ducts,
often multifocal / 80% non-palpable, detected by screening mammogram.
 Treatment: lumpectomy with wide excision margins + radiation OR mastectomy if
large area of disease, or high grade
o
Invasive:
 Invasive ductal carcinoma (most common 80%): hard, infiltrating tentacles
 Invasive lobular carcinoma (8-15%): 20% bilateral. Does not form micro
calcifications, harder to detect mammographically (may benefit from MRI)
 Paget's disease (1-3%): ductal carcinoma that invades nipple with scaling,
eczematoid lesion
 Inflammatory carcinoma (1-4%): ductal carcinoma that invades dermal lymphatics,
most aggressive form of breast cancer.
Clinical features: erythema, edema, warm, swollen, tender breast lump
Peau d'orange indicates advanced disease (III-b IV)
Treatment of breast cancer:
Stage
Primary treatment options
Adjuvant systemic
therapy
0 (in situ)
BCS + radiotherapy
None
I
BCS (or mastectomy) + axillary node dissection +
May not be needed
radiotherapy
II
Chemotherapy and /
or hormone therapy
III
mastectomy + axillary node dissection + radiotherapy
Inflammatory
IV
Surgery as appropriate for local control
BCS = breast-conserving surgery

OSCE-guide-III.doc

Page 54 of 255

History taking Medicine

Dizziness
Causes

Vertigo

Symptoms
Imbalance
N&V
Auditory
Neurological
Nystagmus

Non-vertigo

Peripheral
Benign paroxysmal
positional vertigo
Mnire's disease
Vestibular neuritis
Labyrinthitis
Acoustic neuroma
EAR
Mild-moderate
Severe
Common

Central
Stroke
TIAs
Brain tumour
MS
Cerebellar lesion

Unidirectional

Common
Bidirectional

(horizontal or
rotatory)

Clarification

Analysis of CC

Impact
Red flags
DD
Syncope

Vertigo

Cerebellar
lesion

Cardiac
Arrhythmias
CAD / MI
CHF
Aortic stenosis
Postural
hypotension

Non-cardiac
Vaso-vagal
episode
Panic attack
Somatization

Brain / Neuro
Severe
Variable

(horizontal or
vertical)

Do you feel
You have blackout (syncope)
OR the room is spinning around you (vertigo)
OCD
Timing: when / frequency
What or : certain position
Did you lose consciousness?
Did you fall to ground? Did you hit your head?
Constitutional symptoms
 Vasovagal attack: LOC / while straining or urinating / nausea / do you feel
warning signs before the dizziness?
 Cardiac (tight AS / arrhythmia): heart racing / chest pain / immediate (no
warning signs)
 Hypotension: antihypertensive meds; change of dose or new medication
 Postural hypotension: diabetes / dehydration / parkinsonism
 Neuro (stroke / TIAs): vision changes / loss, speech impairment, weakness








Condition
Benign Paroxysmal
Positional Vertigo

Duration
Seconds to minutes

Hearing loss

Tinnitus

Mnire's disease
Vestibular neuritis
Labyrinthitis
Acoustic neuroma

Minutes to hours
Hours to days
Days
Chronic

Fluctuating
Unilateral
Unilateral
Progressive

Whistling
+






Other features
Certain
positions
Nystagmus
Ear fullness
Recent AOM
Ataxia
CN VII palsy

Risk factors for CAD: HTN / DM / cholesterol / smoking


Unbalanced gait
Aspirin / blood thinners
Alcohol: neuropathy / cerebellar degeneration

PMH / FH / SH

OSCE-guide-III.doc

Page 55 of 255

History taking Medicine

Condition
Benign
Paroxysmal
Positional
Vertigo
(BPPV)

Mnire's
disease

Vestibular
neuritis

Management
Acute attacks of transient vertigo lasting
seconds to minutes initiated by certain
head positions, accompanied by torsional
(rotatory) nystagmus
Diagnosis:
History
Positive Dix-Hallpike manoeuvre

Reassure patient that process resolves spontaneously


Particle repositioning manoeuvres: Epleys manoeuvre

Episodic attacks of tinnitus, hearing loss,


aural fullness (pressure / warmth), and
vertigo lasting minutes to hours

Acute onset of disabling vertigo often


accompanied by nausea, vomiting and
imbalance without hearing loss that
resolves over days leaving a residual
imbalance that lasts days to weeks

Labyrinthitis

Acute infection of the inner ear

Acoustic
neuroma

resulting in vertigo (days) and


hearing loss
May be serous (viral), or purulent
(bacterial)
Occurs as complication of acute and
chronic otitis media, bacterial
meningitis and cholesteatoma
Schwannoma of the vestibular portion of
CN VIII

OSCE-guide-III.doc

(performed by MD) OR Brandt-Daroff exercises


(performed by patient)
Surgery for refractory cases
Anti-emetics for nausea/vomiting
Drugs to suppress vestibular system delay eventual recovery
and are therefore not used
Acute management may consist of bed rest, anti-emetics,
anti-vertiginous drugs (betahistine)
Long term management may include:
 Medical: (1) Low salt diet, diuretics
(hydrochlorothiazide), (2) Local application of
gentamicin to destroy vestibular end-organ, results in
complete SNHL, (3) Betahistine (Serc) prophylactically
to decrease intensity of attacks
 Surgical: selective vestibular neurectomy or
transtympanic labyrinthectomy
Must monitor opposite ear (bilaterality in 35% of cases)
Acute phase:
 Bed rest, vestibular sedatives (Gravol), diazepam
Convalescent phase:
 Progressive ambulation especially in the elderly
 Vestibular exercises: involve eye and head movements,
sitting, standing, and walking

Investigations:
CT head
If meningitis is suspected: lumbar puncture, blood cultures
Treatment:
IV antibiotics
Drainage of middle ear
mastoidectomy
Investigations:
MRI with gadolinium contrast is the gold standard
Audiogram SNHL (sensori-neural hearing loss)
Vestibular tests: normal or asymmetric caloric weakness (an
early sign)
Treatment
Expectant management if tumour is very small or in elderly
Definitive management is surgical excision
Other options: gamma knife, radiation
Dix-Hallpike Positional Testing: the
patient is rapidly moved from a sitting
position to a supine position with the
head hanging over the end of the table,
turned to one side at 45 holding the
position for 20 seconds. Onset of
vertigo is noted and the eyes are
observed for nystagmus

Page 56 of 255

History taking Medicine

INR Counselling
Analysis
Impact
Red flags

History / Give the information


DVT relapse / Bleeding
Female: OCP / pregnancy / LMP / vaginal bleeding

DVT
Warfarin / Blood thinners
Decision

Causes / Complications

Conclusion

Offer brochures, support. If you have time: SAD / PMH

Analysis:
History:

o
o
o
o
o
o

Why are you doing this INR?


When were you diagnosed? How?
Were you admitted through the ER or outpatient?
Was there any involvement of your lungs?
Which medications were you taking?
Do you measure your INR regularly? When was the last time? What was the result? What
is your target INR?

Give the information: Your measurement today shows INR of 1, any idea why?
o
Compliance: Are you still taking your warfarin? On regular basis? Did you stop your
medication? Why?
o
Forget: Do you take your medications on your own, or does someone else help you? Any
chance that you missed a dose?
o
New medications: Did you start a new medication? What? Why? When?
o
Diet: Do you eat a lot of spinach? Or dark green vegetables? (rich in vit K)

Impact:
Now, I would like to ask you some questions to check if you have relapse of your DVT or bleeding,
then we will go from there

DVT relapse:
Because you stopped your medication, I would like to make sure that there is no relapse
o
DVT: Have you had any pain / swelling / redness in your calf muscles?
o
PE: Have you had any SOB, chest pain, heart racing?
o
Stroke: Any confusion? Vision changes? Difficulty finding words? Weakness?

Bleeding:
o
Did you notice any bleeding?
o
Did you notice bleeding from your gums / nose / coughing or vomiting blood / bruises in
your body / dark urine / urine in stools?
o
Any weakness / numbness / difficulty finding words / vision difficulty?
o
Did any one tell you that you look pale? Do you feel fatigued?
Based on what you have told me, there are no obvious serious consequences, if it is ok with you, we
can discuss your situation now!

Red flags: for FEMALES


Are you taking any OCPs? OCPs might increase the risk of developing DVT.
Are you pregnant? Warfarin is not to be used during pregnancy; we will use heparin instead of it.
Have you had any vaginal bleeding?

OSCE-guide-III.doc

Page 57 of 255

History taking Medicine

What is your understanding about DVT?


Causes:
o
It might happen after prolonged sitting without movements (like very long flights)
o
Or due to certain medical condition,
The blood tends to form clots in the deep veins of the lower extremities

Complication:
o
Relapse 8%: without treatment, and that is concerning!
 Whenever we treat the patient, our target is to decrease the relapse rate to 0.8%
which is 1/10 of the risk without treatment
o
These clots are not fixed, and sometimes they get dislodged from your leg and travel
along your blood vessels, all the way to the lungs (chances are 3%):
 If large enough  might cause sudden death
 If showers of small clots  you may not feel anything right now, but it later will
cause what we call pulmonary HTN, which is a debilitating disease, with serious
consequences and we do not have treatment for it right now
o
Always in medicine, we try to balance the benefits and the side effects, and in this
condition, the benefits largely outweigh the risks.

Now, what do you know about blood thinners?


o
It is a medication used to make our blood thin, preventing our body from forming clots
by competing with vitamin K, which is needed for the formation of the elements of blood
clots.
o
We take warfarin seriously, and that is why we monitor it closely and regularly, by
assessing the INR which is an indicator of the effect of warfarin. Therefore, as long as
your INR is within your target, the risk of bleeding is less than 1%, and almost near 0%
to have intra-cranial (brain) hemorrhage without having external bleeding first. That is
why you need to keep monitoring yourself, and seek medical attention if you notice any
signs of bleeding.

Decision:
If the patient decides that he will restart the treatment:

We will do it the same way as we did the first time:


o
We will start heparin and warfarin together then stop heparin after 3 days

We will need to measure the INR daily (till we reach our target) then twice a week, then weekly,
then every 2-4 weeks

NOTES:

Numbers to remember:
o
Relapse (recurrence) of the DVT: 8% without treatment and 0.8% with treatment.
o
Possibility of DVT  clots and PE: 3%
o
Chances of having bleeding with warfarin: 1%, and almost near 0% chance of having
intra-cranial bleeding without having an extra-cranial bleeding.

The initial DVT counselling should have been done in the first time, when the patient was
diagnosed; which includes:
o
General knowledge about DVT
o
Causes and risk factors
o
INR follow up

My best friend was taking warfarin, and he had brain hemorrhage!


o
I am sorry to hear that, this must be stressing / worrying, especially that you are taking
the same medication and he is a close friend to you.
o
We prescribe warfarin for many reasons, the issue here is that your friend was not my
patient, and I do not know about his condition, so I am not in a position to comment on
this situation.
o
I am glad you came here today, so that we can discuss this together.

OSCE-guide-III.doc

Page 58 of 255

History taking Medicine

Patient is receiving blood counsel for adverse reactions


You were called to assess a patient who is receiving blood, and the nurse has concerns.
Adverse effects of blood transfusion:

Febrile reaction:
most common / not serious

Anaphylactic reaction:
not common / serious

Haemolytic reaction:
not common / serious
Introduction to nurse / what is your concerns / ethical challenge
Introduction to patient / ethical challenge
 ABCD
 History
 Brief physical exam
Adverse reactions of blood transfusion
Plan


Introduction to nurse / what is your concerns / ethical challenge


Good afternoon, I am Dr May I get your name please?
How can I help you? OR What are your concerns?
o
There is a mistake!

What do you mean by mistake? How is the patient doing?


o
This was wrong blood!

Did you stop the transfusion?


o
Yes

That is great, this is the first step in the right direction

Now, what do you mean by wrong blood? Is it the same bld group or no? Cross matched or not?
Do we have the patient name on the units?
o
It is the same group but with other patient name
o
Doctor, please do not tell the patient!

Why? Do you have any concerns?


o
I might be fired!

I see you have concerns here, but we need to stabilize the patient first. Then we will speak about
that. However, we need to investigate before making decisions.

Can you tell me when did this happen? How much did he receive?

Introduction to patient / ethical challenge


Good afternoon Mr , I am Dr The nurse was updating me about your condition. It looks like
there was an unintentional medical error took place, and I need to make sure you are ok.
o
Is it serious doctor?

Could be! There are different possibilities; I need first to check you.
o
Whose mistake is this? Is it the nurse mistake?

Usually in the blood transfusion process, there are many steps; any one of those might go wrong.
It is early now to judge. I need first to make sure you are ok and stable, and then I will file an
incidence report. Investigations will be done, and you will be informed with the results.

ABCD
Let us make sure you are safe and stable first.
AB:

Can you please open your mouth? Mouth is clear with no swelling. Do you have any itchiness or
swelling in your mouth?

Trachea is central, no engorged jugular veins. Can I listen to your heart please! Normal heart
sounds.

OSCE-guide-III.doc

Page 59 of 255

History taking Medicine

C:

Can I know the vitals please? Normal / stable.


Can you remove the blood unit please, and send it to the blood bank. We need to re-cross this
patient blood with this unit.
Can you put another IV line please! We need to take samples for: CBC / differential / lytes / blood
grouping and re-crossing / haptoglobin / bilirubin level

D:

I am going to shine light in your eyes!


Can you hold my fingers please? Do not let go.
Do you feel me touching you?
Patient is grossly neurologically free.

If fever: give 2 tablets Tylenol 325 mg


Can you please prepare:
o
Allergic reaction: Benadryl
o
Anaphylactic reaction: Benadryl / Epinephrine / Steroids
o
Haemolytic reaction: Diuretics and fluids

History
Now, I would like to ask you some questions:

Why are you taking blood?


They have found that I have anemia

Did you take blood before? Or is this the first time?

Do you feel warm? Shivering? Chills?

Do you feel any itching or swelling in your lips / mouth?

Any heart racing? SOB? Wheezes? Dizziness?

Any flank pain? Back pain? Weakness?


Brief physical exam
No IV line oozing / No hives on skin / No mouth swelling
Listen to heart / lungs  clear
Press on flanks / spine  no tenderness

Adverse reactions of blood transfusion


Blood transfusion is a commonly used procedure, and it is life saving. A lot of precautions are
taken to make sure it is completed safely. However, like any other medical intervention, it has
some side effects.

The most common reaction that might happen is called febrile reaction. This is not serious
reaction and it is self limited. It might happen again, so if it happens, next time we give you
Tylenol before the transfusion.

Another adverse effect, which is less common but more serious, it is called anaphylactic
reaction. This is a form of severe and serious sensitivity reaction, in which the blood pressure
drops suddenly, and there is a swelling of the tongue, lips, and mouth, with difficult breathing. We
do not have a method to predict it. However, based on your symptoms, your physical exam and
vital signs it is less likely you have that.

The third adverse effect is called hemolytic reaction, and it happens if the patient receives blood
that belongs to another blood group. It causes damage to blood cells which leads to back pain and
flank pain, and could have serious consequences.
Again, based on your symptoms, physical exam and vital signs, your condition does not cope with
this reaction too. And the fact that you received blood from the same blood group makes it less
likely you will have hemolytic reaction.

We prepared medications to deal with any reaction and we will keep you for a while to monitor
you, to make sure that will not happen.

Plan
Call the blood bank to withhold the other units (previously cross-matched)
File an incident report

OSCE-guide-III.doc

Page 60 of 255

History taking Medicine

Counselling Ventilator
Mr Johnson is 75 years old gentleman, his life-long wife for 50 years has a terminal COPD, with
severe pneumonia, and she is on ventilator for the last 3 weeks, and it is not possible to wean her
from ventilator, you called him to inform him about the condition.

What do you know about your wifes condition?

Listen carefully

Show understanding and empathy


Give information about her condition,

Give clear simple information

Stress on the progressive, irreversible nature of the disease


Give alternatives:
Remain on ventilator, with no evidence that she will be able to breath by own, and with
the possibilities of fatal complications like infections, bed sores, Some people does not
like to have this quality of life

Stop the ventilator and she will pass away in peace


As regarding her condition now, have you ever discussed this with her? Has she ever
expressed her wishes about what would she like to be done to her if she needs to be
resuscitated or put on ventilator? Does she have any advance directives or living will?
What do you think about this now?




Offer time if he needs to discuss it with other close family members, or if he needs to arrange
any thing (e.g. I am just giving you information, and we can arrange a meeting with the
family within 2 days so that I can explain to them).

What if she does not want to be on ventilator but he would like to leave her on the ventilator?
Mr Johnson, I am sorry to tell you that, actually it is not our decision or your decision, it
is her choice. And she expressed her wishes before; she decided that she does not want to
have this poor quality of life. We have to respect her wishes.

OSCE-guide-III.doc

Page 61 of 255

History taking Medicine

Ethical questions







Patient has the right to access his/her medical file, we can not withhold it
Patient wants to leave you as family physician  it is his right, and he/she has the right to
take all his/her medical data and file
If you want to terminate a patient from not seeing you as family physician:
o
Give proper notice period
o
See him/her for emergency
Confidentiality; when to break confidentiality? To report for the ministry of transportation for
example:
o
Dementia / delirium
o
Vision problems
o
Seizure disorders
o
Schizophrenia (case-based)
o
Heart attack  1 month not allowed to drive
o
Alcoholic with liver failure (based on Childs criteria: albumin / ascites / INR /
bilirubin)
Report for child safety  CAS (Children Aid Society)
o
Even if POTENTIAL or SUSPECTED
o
Child neglect / abuse
Patient wants to leave hospital against medical advice; e.g. patient has just had a heart attack,
and still insists to leave the hospital!
o
I would like to make sure he is competent, not under influence of alcohol or any
substance, and to rule out suicidal ideation
o
I would explain to the patient: diagnosis / treatment / side effects of treatment /
complications of not receiving treatment / alternatives
o
I will document this, and I will ask the patient to sign a LAMA (leaving against
medical advice), and I will let him go
Biological parent wants to know the medical details of his/her son, who is adopted by another
family!
o
In order to determine whether I should release any information or no, I would
like first to know who has the legal custody (guardian) of this child. It might be
the adopting father, a social worker (case manager)
Any unconscious patient  ask for DNR or advanced directives
MMS exam score < 24  patient is incompetent;
o
You have a case of patient, who had surgery, is taking medications, but he
developed delirium post-operative and now he wants to discontinue his
medications  NO; he is delirious, incompetent to change decisions, he already
consented to take the medications before he entered this delirium.
o
What if this patient broke his leg; do you want to operate him without consent?
This is a new condition; we do not know what would be his competent wishes 
look for SDM (substitute decision maker).

OSCE-guide-III.doc

Page 62 of 255

History taking Medicine

HIV post-test counselling

Usually you are covering for other physician to give the test result which means this is a new pt
to you.
Be sensitive, empathetic, and flexible

 Introduction:
Your Dr. is away, I am covering for him/her, and I have your file with me, I just need to understand the
situation here,

What have you discussed last time?

Why did you ask for the test last time?

Did you feel sick in any way?

Was there anything made you worried about your own health?


Give the test result:


It is positive, sure about the result, because they do two tests before giving the positive result.
Check with patients reaction: normalize patients feeling; all the feelings you are having now
are very normal.

What kind of thoughts are going through in your mind? What concerns you the most right
now?

AIDS: dont have AIDS, will not die tomorrow or so. Prognosis is variable, but many people carry
it without feeling it for quite long time, years

Consequences of HIV:
Repeated infections / LNs
Tired / fatigue
Memory dementia
Depression

Causes of HIV:
SAD shared needles
Sexual:
o
Risky behaviour
o
Confidentiality how to inform the partner?
 Get the background info: duration of the relationship, how close to each other,
 Partner has to know: Risk of infection / Needs to be tested
 Will know anyway, either from public health or him. Prefer him to tell, offer help to
tell.

Before discharge the patient:


Arrange follow-up visit in the next couple days after patient digests the info
Make sure that patient is safe to go home, safe to drive back, no suicidal or homicidal ideation,
inquire about support system.

Education: emphasize the importance of safe sex: advice use barrier contraceptive methods all the
time with all partners in the future to prevent the transmission.

Case: HIV patient with diplopia  cranial nerve examination

CT brain:
enhanced multiple rings  toxoplasmosis with HIV

DD:
TB / toxoplasmosis / CMV / CNS lymphoma

Management:
refer to infectious disease specialist

OSCE-guide-III.doc

Page 63 of 255

History taking Medicine

Lung Nodule


Introduction:
Why X-ray was taken? When?
When was last normal X-ray? Do we have it?
Give the test result:

Solitary Lung Nodule. Definition: a round or oval, sharply circumscribed radiographic lesion, size
up to 3-4 cm, which may or may not be calcified, and is surrounded by normal lung. Can be
benign or malignant

Any ideas about what could be causing this nodule

Consequences:
Local symptoms: cough, phlegm, haemoptysis, SOB, wheezing
Constitutional symptoms: fever, chills, night sweat / change of appetite, weight loss, fatigue /
pumps or lumps in the neck or elsewhere in the body

Impact / screen for metastasis:


o
Brain: headache/ vomiting?
o
Back: back pain/ weakness/ numbness/ tingling in arms or legs?
o
Lungs: chest pain/ cough/ phlegm/ wheezes/ heart racing?
o
Liver: yellow discoloration/ itching/ urine/ stools?

Causes:
Smoking
Exposure to chemicals / smokes at work
T.B.: Contact with sick person (T.B.) / Recent travel / T.B. skin test
Sarcoidosis: associated symptoms; joint pain, skin rash
History of lung disease
History of cancer
HIV status
Family History of T.B. or Lung cancer

Management:
Investigations
o
CXR: always compare with previous CXR
o
CT densitometry and contrast enhanced CT of the thorax
Sputum cytology / stains
TB skin test
o
Biopsy: bronchoscopic or percutaneous(CT-guided) or excision (thoracoscopy or
thoracotomy): if clinical and radiographic features do not help distinguish between
benign or malignant lesion
 If at risk for lung cancer, biopsy may be performed regardless of radiographic
features
 If a biopsy is non-diagnostic, whether to observe, re-biopsy or resect will depend on
the level of suspicion
o
PET scan not yet routine but can help distinguish benign from malignant nodules
Watchful waiting: repeat CXR and/or CT scan at 3, 6, 12 months

Algorithm:
Evaluation of a Solitary Pulmonary Nodule; check previous CXR
o
Looks benign or unchanged  repeat CXR q 3-6 months for 2 years
o
Significant risk factor on history or looks malignant or changed  CT chest
 Cause (infection or cancer)  stage and treat
 Calcification  observe
 No diagnosis  trans-thoracic needle biopsy
Inflammatory  treat the cause
Cancer  stage and treat
Still NO diagnosis  resect for diagnosis

OSCE-guide-III.doc

Page 64 of 255

History taking Medicine





Past Medical History


Family history: of T.B. or Lung cancer
Social History

Evaluation of a Solitary Pulmonary Nodule:

OSCE-guide-III.doc

Page 65 of 255

History taking Medicine

High Creatinine


Introduction:
Why the test was done?

When was the last normal test?

Any idea about the meaning of the test

Give the test result:

Consequences: manifestations of renal failure


Nausea and vomiting / stomach pain

Itching

Pallor

Fatigue

S.O.B with exertion

Swelling (ankle, around eye)

Bone pain

Causes:
Renal:

Hypertension
Diabetes
o
Repeated kidneys infection
o
Poly-cystic kidneys
o
Medications: NSAIDs / gold / penicellamine / ACEIs
Post-renal:
o
Kidney stones
o
Bladder cancer
o
Prostate problem
o
o




Past Medical History:


o
Kidney disease
o
Previous hospitalization
o
Nephrectomy
o
Allergies
Family history: of renal problems
Social History
o
Smoking
o
Alcohol
o
Drugs
o
Work
o
Home
o
Support

OSCE-guide-III.doc

Page 66 of 255

History taking Medicine

Impotence / Erectile Dysfunction




Introduction:
ED is a common problem in men, with a broad DD, encompassing organic & psychogenic causes.
This is often a difficult topic for men to discuss with their doctor. Confidentiality.

Penile erection is a multi-factorial process dependent on integration of neurologic, hormonal,


vascular and emotional factors.
Analysis of the CC:

Primary vs. Secondary


o
Chronology (frequency, onset, duration, course)
 Onset: acute (more likely psychogenic) or gradual (organic)?
 Course: intermittent (more likely psychogenic)? Libido affected?
o
Severity or amount? All the time?
o
Aggravating / precipitating and alleviating factors

Organic vs. Psychogenic


o
Do you have early morning erection?
o
Do you have night time emissions?
o
Do you have desire?
o
Are you able to masturbate to an erection or climax?
o
Situational dysfunction; does function vary depending on the setting? Partner / Place /
Time?
Consequences: How does this affect your life? Your relationship?
Causes:

Many endocrine disorders and systemic diseases cause ED by influencing libido, autonomic
pathways and/or blood flow.

Organic causes:
o
Medical causes: history of DM, HTN, hyperlipidemia, peripheral vascular disease,
intermittent claudication
o
Neuro: back trauma / constitutional symptoms (back metastasis) / back pain / weakness,
numbness / history of MS,
o
Low testosterone: changes in secondary sex characteristics, e.g. hair pattern changes /
history of gynecomastia / galactorrhea / history of thyroid disease / pituitary disease (
visual defect, headache)
o
Medications; e.g. anti-depressants, hormonal treatment, opioids, MAO inhibitors
o
SAD: smoking / alcohol / recreational drugs

Psychogenic causes:
o
Any problems with their partner(s)
o
History / screening of depression
o
Any recent changes in life (home, work, socially) / anxiety attacks? Any stress? Past lifebackground, upbringing,




Counselling:

Normalize patient feelings

ED can often be improved with:


o
Life style modifications: exercise / weight loss / improved diet / DM control / smoking
cessation / alcohol / stress management / anxiety / sleep hygiene
o
Improvement of patient relationship with partner: marital counselling / address sexual
boredom / refer to specialist in sexual education and therapy

Unfortunately, many organic causes are irreversible, but we have treatment options:
o
Testosterone preparations (if low testosterone)
o
Viagra or Cialis
o
Penile self-injection
o
Vacuum rubber ring device
o
Penile prosthesis

Follow-up appointment for BOTH partners

OSCE-guide-III.doc

Page 67 of 255

History taking Medicine

Rheumatology History Taking




Chief complaint: pain / stiffness / weakness / deformity / limitation of movement / joint


clicking

OCD / Acute (< 6 weeks) versus chronic (> 6 weeks)

PQRST /
Associated Symptoms:
Morning stiffness
o
Inflammation: morning stiffness (>30 min), better with use, constitutional
symptoms
o
Non-inflammatory: worse with use, worse at end of day, can have some stiffness
but usually not prolonged

Joint swelling / redness

Other Joints / Pattern of joint involvement:


o
Mono-arthritis, oligo-arthritis (4 or less), poly-arthritis (5 or more)
o
Symmetric vs. asymmetric
o
Peripheral joints versus axial involvement (spine, SI joints)
o
Small joints (hands / feet) versus large joints (hips / shoulders)
o
Additive joints vs. migratory joints
o
Tendon involvement

Constitutional symptoms

Extra-articular features:
Seropositive (e.g. RA, SLE, Sjogrens, scleroderma, inflammatory, myositis)
Seronegative (Ankylosing spondylitis, psoriatic arthritis, enteropathic arthropathy,
reactive arthritis)
o
Eyes: iritis, scleritis, conjunctivitis, dry eyes
o
Oral ulcers
o
Respiratory: pleural effusion, pleuritis, pulmonary fibrosis, pulmonary nodules
o
Cardiac: pericarditis, pericardial effusion, conduction defects
o
GIT: GERD, inflammatory bowel disease, malabsorption, bloody diarrhea
o
Dermatology: malar rash, discoid, nodules, telangiectasias, sclerodactyly,
calcinosis, alopecia, periungal erythema, psoriasis, nail pitting, onycholysis,
erythema nodosum, pyoderma gangrenosum

Crystal arthropathies
o
Mono-arthritis (red, hot), chronically can be poly-arthritis: gout (tophi, alcohol
history, renal failure, drugs)
o
CPPD (hyperparathyroidism, hypomagnesemia, hemochromatosis, Wilsons
disease, hypothyroidism)

Septic arthritis: usually mono-arthritis, fever, red, hot. Gonococcal arthritis can be
migratory, with tenosynovitis and skin pustules

Disability and adaptation


How does it affect your life? Ask about daily activity!

Effects on ability to work! Support!

OSCE-guide-III.doc

Page 68 of 255

History taking Medicine

Multiple Sclerosis
Middle aged man (or woman) with episodes of numbness in one leg.

History:
Review of systems

Diagnosis:
MS

Investigations: MRI / CSF




History of present illness (neurological screening)


Ethnic background.

History of weakness: MS can mimic ANY neurological disease process.


o
Determine which groups of muscles are involved: proximal vs. Distal, symmetric
vs. Asymmetric
o
Pattern of weakness: stepwise decline, relapse and remitting

Dysphagia, dysarthria

Dry eyes, dry mouth

Numbness in upper or lower limbs

History of previous vertigo, loss of vision (optic neuritis), diplopia,

Clumsiness, loss of balance, falls

Any associated sensory symptoms

Bowel or bladder dysfunction (difficulty fully emptying bladder, urgency, incontinence)

Fatigue, arthralgia, depression, behavioural changes

Constitutional symptoms

Impact on daily functioning

Lhermitte's sign: electrical sensation down back on neck flexion


Uhthoff's phenomenon: worsening of symptoms in heat (hot bath, summer)

Past Medical History


Previous transient focal neurological deficits: vertigo, sensory deficits, etc.

History of headache

Sarcoidosis

HIV status

Systemic inflammatory disease

Family History: MS, SLE, weakness NYD, visual deficits NYD

OSCE-guide-III.doc

Page 69 of 255

History taking Medicine

Obesity


Analysis of chief complaint:

Weight analysis: now and how about one year ago? Maximum and minimum weights?

Weight loss history:


o
Attempts, medically vs. surgically
o
What weight achieved, what caused failure
o
Assistance: dietition, exercises programs
o
Explore why patient want to do it again
o
Admire and encourage the pt

General health now:


o
HTN, IHD
o
DM, cholesterol
o
Sleep apnea, asthma
o
OA, functional impairment, disabilities, social activities involvement
o
Psychological assessment: embarrassment, low self esteem, depression, anxiety

Diet: detailed history; how many meals, how much, bedtime meals,

Exercises: how often, type





Past medical history: AMPLE,


Social history: SAD, social activities, home and work environments
Family history: obesity in the family

Counselling:



Encouragement: admire patient, it is important for your general health, requires a lot of effort;
it is very difficult process, very common multiple tries.
Methods:

Set up a goal first, start slowly

Diet: can refer you to a dietition


o
Type of food: high fibre, vegetables and fruits, less fat/cholesterol, low
carbohydrate,
o
Caloric intake should be calculated /+/ does not exceed 1800 Cal/d
o
I will give you tables and graphs to show you the ideal meal composition, but
generally, lunch and supper must be formed of: 50% vegetables and fruits /+/
25% protein /+/ 25% carbohydrates

Exercises:
o
Program: 3-5 times per week /+/ 30-50 min each time
o
Set up personal instructor to guide

Medications: locally to absorb fats or centrally working on the satiety centre; do not like
to start with

Surgical procedures, in very advanced cases and there is medical impairment, we can
discuss it later.

Avoid: smoking / alcohol /+/ Healthful life style.


Educational information: group program / booklet and brochures
Follow up on regular basis

OSCE-guide-III.doc

Page 70 of 255

History taking Medicine

Epilepsy Counselling


Young 16 yr old male for driving license counsel

Introduction
Analyze
epilepsy history

Triggers

MOAPS
HEAD SSS


Why does the patient want a note from doctor for a drivers license?
Usually Dr does not give such note unless there is underlying condition!
 Age of onset? / When was the diagnosis? / What was the diagnosis?
 How frequently do the attacks occur?
 How long does each attack last? LOC
 Aura prior to attack?
 When was last attack? Similar to previous ones?
 What happens during an attack? Does the patient shake / all over / partly /
roll up eyes/ bite tongue?
 How do you regain consciousness / how do you feel after the attack
 Which medication does the patient take? Compliance? When was the drug
level checked?
 Any other medications that might interact with epileptic drugs?
 Sleep deprivation / Long screen time before sleep?
 Alcohol? Stimulants?
 Are you under stress
Scan the mood and anxiety
Home / Education / SAD (do you take stimulants)


Counselling: What is your understanding of seizures?


It is a common condition due to increased electrical activity in the brain, causing abnormal body
movements. Some people lose consciousness, and some do not. Usually, it does not cause learning
disability or brain damage

The diagnosis of epilepsy requires the occurrence of at least 2 unprovoked seizures 24 hours apart

Most of patients have no clear reason to explain why they are having this, but it tends to run in the
family (idiopathic) or structural brain damage (post-meningitis)

Other conditions that should be considered include: Syncope (arrhythmias), Vascular (TIAs),
Metabolic (hypoglycemia), Psychiatric (conversion, panic attacks, malingering)

Prognosis: The patient's prognosis for disability and for a recurrence of epileptic seizures depends
on the type of epileptic seizure and the epileptic syndrome in question. Regarding morbidity,
trauma is not uncommon. Regarding mortality, seizures cause death in a small proportion of
individuals. Most deaths are accidental due to impaired consciousness

Plan:
Diagnostic workup

Patient education

Treatment

Pregnancy

Diagnostic workup:
o
Two imaging studies must be performed after a seizure. They are neuro-imaging
evaluation (MRI or CT) and electroencephalography (EEG).
o
Lumbar puncture for CSF examination has a role in the patient with obtundation or in
patients in whom meningitis or encephalitis is suspected.
o
Metabolic screen
o
Serum studies of anticonvulsant agents (e.g. phenytoin); if therapeutic level but side
effects or poor seizure control  add another drug (carbamazepine / valproic acid)

OSCE-guide-III.doc

Page 71 of 255

History taking Medicine

Patient education:
o
Dangerous activities: to prevent injury, educate patients about seizure precautions. Most
accidents occur when patients have impaired consciousness. Restrictions apply on:
 Driving (report to ministry of transportation), must be seizures-free for more than 1
year
 Diving, swimming, hiking, mountain climbing
 Taking unsupervised baths, better take shower not bath, with open door
 Working at significant heights, operating machines and the use of fire and power
tools.
o
Avoid the triggers for seizure attack:
 Alcohol will exacerbate (chronic alcohol: blood level of anti-epileptics due to
metabolism / excess alcohol: seizure threshold)
 Stress; if the patient is having stress / anxiety / alcohol issues: counsel and offer
social support
 Sleep deprivation / long screen time before sleep
 Head trauma,
 Forgetting to take medication on time
 Taking other medications that interact with the treatment
o
Life style:
 You have to take the treatment almost for your whole life
 Talk with your physician about any new medication you want to take
 Medications are teratogenic, females to take proper contraceptive measures
 Patient might choose to wear a bracelet indicating he has epilepsy
 If a seizure will happen: go to the ER
 Regular follow-up visits and monitoring of anti-convulsion level in blood
Treatment:
o
The mainstay of therapy for people with recurrent unprovoked seizures is an
anticonvulsant. If a patient has had more than 1 seizure, administration of an
anticonvulsant is recommended. However, standard of care for a single, unprovoked
seizure is avoidance of typical precipitants (e.g. alcohol, sleep deprivation); no
anticonvulsants are recommended unless the patient has risk factors for recurrence
o
Medications will be taken for long term, there are many options, will start with one
medication, if no full control, we may increase the dose and/or add another drug
o
Side effects of medications: movement disorders (ataxia, dysarthria), teratogenic, liver,
kidney, drowsiness, poor concentration
o
Discontinuation: After a person has been seizure free for typically 2-5 years, physicians
consider discontinuing the medication. About 75% of relapses after discontinuation occur
in the first year, and at least 50% of patients who have another seizure do so in the first 3
months. Therefore, patients to observe strict seizure precautions (including not driving)
during tapering and for at least 3 months after discontinuation. Authors recommend that
anticonvulsants be gradually discontinued over 10 weeks
Pregnancy:
 Are you sexually active?
 Do you take use contraception?
o
No  Are you planning to get pregnant? Yes! Let us talk about pregnancy and the meds
you will start. Can you postpone the pregnancy for a while? It is better to have good
control of seizures for a while; to get any seizure during pregnancy will pose great risk
for both of you and baby. And the medications can cause serious malformation to the
baby
o
Yes is it OCPs? Yes! There might be drug interaction, so for the time being you need
to continue to use your pills and add another method (mechanical) till you contact your
gynecologist

OSCE-guide-III.doc

Page 72 of 255

History taking Medicine

Medical note



For past date!


For headache

1. What type of doctors note pt wants; reason for the note?


2. Analyze the case  headache case  full history
3. Deal with the patient request:
 Mention that patient is probably suffering from migraine or tension headache.
 Mention that you cant write note for previous visit; but can give copy of todays visit and
notes about what happened;
 Encourage the patient to talk to the tutor (or supervisor) frankly about what happened.
Mention you believe the professor would be reasonable; ask about any possibility to make up
the class he missed;
 Mention you are willing to do any thing to help him out; and apologize that you cant lie or
write a note for the pt you didnt see at that time;
 Mention you are happy to see this patient again to see how things going after he talks to
supervisor and for follow-up regarding his headache.

OSCE-guide-III.doc

Page 73 of 255

History taking Medicine

Pre-diabetes Counselling


What is DM?

Fasting blood sugar (FBS) > 7 mmol/L


Random blood sugar (RBS)
> 10 mmol/L + symptoms
Glucose tolerance test (GTT)
> 11.1 mmol/L
What is pre-diabetes? Impaired glucose tolerance
o
Fasting blood sugar (FBS) 6.1 6..9 mmol/L
o
Glucose tolerance test (GTT)
7.8 11 mmol/L
o
o
o

Introduction

Impact

Red flags

Pre-diabetes: does not mean that you have diabetes, but it shows that you have an
increased chance of having it, about 15% per year. It also shows increased risk of
you having complications in the large blood vessels causing heart diseases, strokes
and peripheral vessel diseases
Diabetes:

Increase of blood sugar in our blood due to deficient or ineffective insulin.

Explain the role of insulin in helping cells to utilize glucose, two types of DM,
type I and type II.

With one reading we can not say that you are prone or have DM, so let me ask
few questions, to see if you have the symptoms of DM!


Symptoms of
hyperglycemia






Eat more, drink more, pee more even at night


Blurred vision
Tired / weight loss
Yeast infections, are there itching / rashes in your groins, in the
toes and finger webs?
 Do your wounds get long time to heal?
N/V, abdominal pain, dehydration, LOC

Symptoms of
Ketoacidosis
Symptoms of
If patient is on insulin: sweating, shaking, palpitation, fatigue,
hypoglycaemia
headache, confusion, seizures
Complications of
 Micro-vascular: nephropathy / neuropathy / retinopathy
high blood sugar
 Macro-vascular: CAD / peripheral arterial dis / impotence
 Lifestyle: too much simple sugars, lack of exercises, overweight, family history
 Medications: steroids / beta blockers (-blockers are contraindicated in DM: it causes
hyperglycemia / and it masks hypoglycemia)

PMH

Medications: used long term steroids, thiazides, phenytoin, clozapine or other antipsychotics, HTN, Cholesterol, CAD, CVD, kidney, hospitalization
FH
DM in first degree relatives
SH

Sexual function: any concerns

Smoking
 From the conversation we had, it looks like you are likely to get DM. However I am going to examine
you and do blood tests (FBS, Hb A1C which shows your blood sugar level over the past 3 months,
lipid profile, micro albumin / Cr ratio, ECG).
 I strongly recommend you to work on lowering your chance of having diabetes by half by: watching
your diet (healthy balanced diet, avoid saturated fats and simple sugars, choose low glycemic content
foods), exercising (30 -45 min of moderate exercise for 4-5 days/wk) and life style changes (limit Na,
alcohol, caffeine, stop smoking).
 I can refer you to diabetes educational program if you wish.
 Treatment targets:
Hb A1C < 7
FBS 4 6
Lipids: LDL < 2, Triglycerides < 1.5 or TC/HDL < 4
BP < 130/80

OSCE-guide-III.doc

Page 74 of 255

Emergency Medicine

Emergency Medicine

OSCE-guide-III.doc

Page 75 of 255

Emergency Medicine

Emergency Room
Trauma
ATLS: Advanced Trauma
Life Support protocol
Primary survey:
- Any trauma (kicked, car
accident, thrown from
height, )
- Manage over the phone:
trauma or meningitis
- Secondary survey: patient
in the ER after car
accident, primary survey
was done, do the
secondary survey
N.B. if knife: leave it in place,
fix with gauze

Non-trauma
Cardiac
Chest pain:
- MONA -blockers
- STEMI: catheterization,
thrombolytics
- No ST elevation: heparin,
angiography
Arrhythmias:
- V Fib
- V Tachy
ACLS: Advanced Cardiac
Life Support code
Heart block: old patient
- DNR
- Advanced directive

Non cardiac
-

Acute abdomen for 24 hrs,


BP, HR
Upper / lower GIT
bleeding
Severe headache for 2
hours
Seizure for the last 20
minutes
16 years brought
unconscious to the ER
Patient receiving blood,
counsel him

Management:
Trauma
I
A
B
C
D
AMPLE
Head to toe
Management

OSCE-guide-III.doc

Medical
I
A
B
C
D
OCD
PQRST

Associated symptoms
Risk factors
PMH
Focused physical exam
Management

Page 76 of 255

Emergency Medicine

Trauma
I: introduction:
- Because it is a trauma case, I would like to activate the ATLS protocol
- I would like also to get protection for me and my team; gloves, gowns, goggles and
masks
-

Mr I am Dr , the physician in charge in the emergency room;


o
If the patient is wearing a neck collar, proceed
o
Pt is not wearing a neck collar: Mr please do not move your head, nurse can
you please fix his head, we need to put neck collar

I understand that you are here because you had a car accident
How are you feeling / doing right now?
o
I would like to make sure that you are stable, I will check with the nurse and we
will start the management then I will be asking you more questions.
o
I can see that you are in a lot of pain, please bear with me for few minutes, and I
will give pain killer as soon as I can.
o
Doctor, where is my wife? How is she doing? Was she with you? I can see that
you are concerned about your wife, I will look for her and I will get back to you
as soon as I can, meanwhile my first concern is to make sure you are stable

We will start the primary survey now:


Airways / Breathing
Mouth / O2 / inspect chest, neck / listen to lungs, heart
Can you please open your mouth?
Nurse, what is O2 saturation, plz?

Inspect the chest

Open the collar window, or fix pt head


and remove anterior part:

Trachea

Jugular veins (JV)


Listen to lungs
Listen to heart sounds

OSCE-guide-III.doc

Mouth is clear; no FB, no dentures, no vomitus


Pt is talking to me that means airways are patent
Can you give him O2 4 L with a nasal canula
Any change in saturation? Can you plz let me know if
any change in saturation happens!
By inspection, chest is symmetrical, no bruises, no
open wounds, no paradoxical movements of the chest,
no use of accessory muscles for breathing
Trachea is central, JV not engorged, bilateral air entry,
normal heart sounds (HS) S1 and S2
Trachea JV
Air
HS
Diagnosis
entry
shifted
Engorged same normal tension
away
side
pneumothorax
shifted
depleted
same normal Hemosame
side
thorax
side
central
engorged bilatera muffle cardiac
l
d
temponad
e
Usually no cardiac temponade in the exam

Page 77 of 255

Emergency Medicine

+ If BP and HR / other signs of tension pneumothorax  nurse, I need to put a large needle
(16 / 14 G) in the 2nd intercostal space at MCL (upper border of the 3rd rib);

Is there any gush of air?

Check the trachea centrality and air entry

We need to put a chest tube in the 5th intercostal space


+ If BP and HR / other signs of hemothorax  nurse, I need to put chest tube in the 5th
intercostal space at anterior Axillary line;

What is the amount of blood?

If > 1.5 L  stat surgery

Otherwise, monitor; if > 200 ml/hr  surgery


Circulation
Vital signs / fluids / withdraw blood samples / look for source of bleeding
Can I get the vital signs please

Comment, patient is hypo- / hyper- / tension,


comment on HR, pt is stable / unstable
I would like to have two large IV lines, 16 G in both anti-cubital fossae:

One to start fluids: bolus 2 L ringer lactate or normal saline

The other line is to withdraw samples for: CBC/differential/lytes /+/ blood grouping and
cross matching / and prepare 6 units of blood (4 matched and 2 O) /+/ stat glucose /+/
INR/PTT/LFT /+/ Bun/creatinine /+/ toxic screen/alcohol level /+/ continuous cardiac
monitoring/cardiac enzymes and ECG
Can you please inform me with the vitals; after the bolus fluid is
done and every 5-10 minutes or if there is a change in the vitals
Look for the source of bleeding
Abdomen:
 Inspect the abdomen  bruises
I am going to look at and feel your
 Palpate the abdomen  rigidity and guarding
abdomen
 If positive; I am suspecting intra-abdominal
bleeding, I would like:

To get stat surgery consult

To arrange for FAST (focused abdominal


sonogram for trauma)

To do DPL (peritoneal lavage)


I am going to press on your pelvis
 Press from the sides
 Press open book
 If positive; I am suspecting pelvic fracture:

Cut pt sheet and wrap around the pelvis to


support, and check blood on penile meatus

Stat orthopedics consult


Lower extremities
By inspection, patient lower extremities are
symmetrical, no abnormal posture or deformity. No
inequality in length, no pain, no deviation
 If positive: I am suspecting femur fracture;

Check the pulses

Thomas splint

Stat orthopedics consult


Log rolling
I need more team members to roll the patient on his left
side:
 To check for external source of bleeding
 To press on the spinal processes
 To perform digital rectal exam

OSCE-guide-III.doc

Page 78 of 255

Emergency Medicine

I would like to get trauma X-ray series: for neck, chest, LSS and pelvis
D:
D1: Deficits / Disability D2: Detoxification
Neuro screen /
I am going to shine light in your eyes?
Can you please squeeze my fingers, do
not let them go
Can you wiggle your toes?
Do you feel me touching you here,
here, and here
Glasgow coma scale eyes
AVPU

D3: Drugs

Pupils are round, symmetrical and reactive


Patient is gross neurologically free

Alert
Verbal

4
3

Pain
2
Unresponsiveness

AMPLE
A Do you have any allergies?
M Do you take any medications on regular basis?
PMH, any history of HTN, heart attack, stroke, DM, any long term disease
P
L  Last meal
 Last tetanus shot
 LMP
E Event:
 Can you describe to me want happened?
 Car accident! Were you the driver or passenger / front passenger?
 Were you wearing your seat belt?
 Did you hit your head? Did you lose your conscious?
 Do you remember what happened, before and after the accident?
Conclusion:
I am suspecting an intra-abdominal bleeding; we are waiting for (surgeon, orthopedics surgeon)
to intervene
Summary:
If you are done  go for secondary survey:
Introduction to examiner

Expose the patient
Hello
 Examine him head to toe, looking for fractures,
Neck collar
more detailed neurological examination
Introduction to patient
A/B / C / order x-rays / D / AMPLE

OSCE-guide-III.doc

Page 79 of 255

Emergency Medicine

NOTES:
FLUIDS:
- Trauma / GIT bleeding: we always start with 2 L bolus
o
If the patient is stable for the beginning  do not give anything more
o
It the patient was not stable, but becomes stable after the first 2L bolus  give
maintenance fluids
o
If patient was not stable, and remains unstable  start bld transfusion: 1 unit of
packed RBCs for every 3 units of fluids, and continue till you find source of
bleeding
Stable
2 L bolus
Stable
Give nothing
Unstable
2 L bolus
Stable
Give fluids for maintenance
Unstable
2 L bolus
Unstable Start blood transfusion 2 RBCs
Then continue 1 (RBCs) : 3 (NS)
-

Anaphylactic shock:
o
0.5 L bolus
o
Give epinephrine / steroids / anti-histaminics (Benadryl)
Acute abdomen (pancreatitis / DKA):
o
1-2 L bolus
o
Followed by 1 L / hour till the urine output improves
Heart attack:
o
KVO (keep vein open) 100 cc / hour
If trauma, BP, HR with warm extremities  neurogenic shock (spinal cord injury) 
give only 2 L of fluids then give vasopressors

For the exam:


 Tension pneumothorax
 Hemothorax
 Rupture spleen
 Fracture pelvis
 Fracture femur

OSCE-guide-III.doc

Page 80 of 255

Emergency Medicine

Unconscious Patient Neuro


1- Introduction:
- Mr I am Dr I am the physician in charge in the emergency room now,
- Mr Mr if you hear me open your eyes please. Tap on the shoulder, do you hear me
 I would like to activate the ACLS code please / start primary survey
 Check the mouth, listen for patent air way
 Give 4L O2 via nasal
A
 What is the O2 please
canula
B
 Trachea central, chest is moving
 Monitor O2 for need
 Listen to lungs, heart
to intubation
 2 large IV lines; for
I would like to get the vital signs please:
C
 Based on vitals, no in BP and no in HR (as seen
IV fluids and to
in Cushing triad)
withdraw samples
 No fever
 No abnormal breathing patterns
 Mr I will open your eyes, and shine light in it, pupillary reaction  both
D
pupils are round, symmetrical and reactive
 Mr if you hear me can you please move your eyes up and down  NO
locked-in syndrome
 D3: universal antidotes: thiamine 100 mg
2- Glasgow Coma Scale  if < 8  intubate (ask about DNR)
1
2
3
4
5
6
No
Pain
Voice
Spontaneou N/A
N/A
Eyes
response
s
Words
Confused
Normal
N/A
Verbal No sounds Sounds
Extension Flexion
Withdrawal Localizes Obeys
Motor No
movements (decerebrat (decorticat
commands
e response) e
response)
3- Cranial nerve exam: excluding the motor and the sensory:
- Inspection:
o
Face is symmetrical, no deviation of the angle of the mouth
o
Both eyes are symmetrical, no nystagmus
- I would like to do a fundoscopic examination, to look for disc edema, or retinal
hemorrhage
- Reflexes: Pupillary reflex (II, III) / Corneal reflex (V, VII) / Gag reflex: (IX, X)
4- Upper extremities:
- Inspection: no abnormal posture or contractures
- Tone: check the wrist and elbow in both sides  No cog wheel, no lead pipe rigidity, no
clasp knife spasticity
- Reflexes: Biceps / Triceps
- Check the radial pulse
5- Lower extremities:
- Inspection:
- Tone: check by leg rolling  rigidity, elevate the knee rapidly  spasticity
- Reflexes: Knee reflex / Ankle reflex / I would like to do Babinski reflex / Clonus
- Check dorsalis pedis pulse
- If the neck is cleared (by CT), I would like to do nuchal rigidity, Kernig's sign,
Brudzinski's sign
6- To check for brain death: I would like to arrange for: dolls eyes, caloric reflex test

OSCE-guide-III.doc

Page 81 of 255

Emergency Medicine

Unconscious Patient Diabetic


16 years old female found unconscious in her class, next 10 minutes manage and counsel
Introduction:
- Ms I am Dr I am the physician in charge in the ER,
- Ms if you hear me; can you open your eyes please? Tap on the shoulder, do you
hear me  I would like to activate ACLS code please / start primary survey
A
B

 Check the mouth, listen for patent air way



 What is the O2 please
 Trachea central, chest is moving

 Listen to lungs, heart
 I would like to get the vital signs please: BP and HR
 2 large IV lines; for IV fluids5 and to withdraw samples6
When you ask the nurse for stat glucose by finger prick:

Give 4L O2 via nasal


canula
Monitor O2 for need to
intubation

Hypoglycemia
Hyperglycemia
 Stat 100 mg thiamine IV
 Stat insulin 10 units IV
 Stat 50 ml D50 (Dextrose 50%) IV
 Stat 100 mg thiamine IV
 If no IV line  glucagon IM
 2 L fluids
At that time, the patient will Orient her; your blood sugar was low, your class-mates
start to regain her conscious brought you here, you are in the ER in hospital, you are
doing well now, how do you feel right now?
Patient states that she is
 Reassure her
worried she will lose her
 I can help by giving you a doctors note
exam / or other important
 This is a very serious condition, you need medical
appointment!
attention for some time it is not safe to leave
D1: Brief neurology
 Start D5 (Dextrose 5%): 250 ml / hr
D3: Dextrose
 Nurse, I would like to monitor her blood glucose
every 5-10 minutes

+ In case of DKA and physical exam:


History
 The same as in hypoglycemia (see below)
 The causes are (5 Is): insulin missed / infection / intoxication / ischemia /
infarction
Physical exam
 Brief neuro exam
 Brief DM exam
+ In case of hypoglycemia:
History
Are you diabetic?
Analysis
Diabetic
 When were you diagnosed? And how?
history
 Do you take insulin?
 Have you had coma (DKA or hypoglycemia) before?
 When was your last DM follow-up visit? Any reason?
 At that time; were you controlled? Symptoms free?
 When was your last Hb A1C test? What was it?
 How about last few days, were you measuring your glucose?
EVENT
 This morning, did you get breakfast, your insulin? Did you check your glucose?
Did you exercise?
 Before you lost conscious, hoe did you feel? Hungry / shaky / dizzy / sweating?
5
If the HR is normal and other VS are normal, you can give only 50 ml/hr to keep vein open (KVO)
But if HR  give 2 L fluids for follow-up
6
For any female patient: -HCG with the blood works you will order

OSCE-guide-III.doc

Page 82 of 255

Emergency Medicine

Impact

Red flags
DD

PMH




If pt dx < 5 years  no worry about complications


If pt dx > 5 years screen for complications (urine changes / visual changes /
numbness and tingling in her feet)
 Constitutional symptoms
 How much insulin do you take? Do you make sure you eat after your insulin
dose?
 Any or in your weight? Stress?
 Did you start new medication?
HEAD SSS /+/ Pregnancy

Counselling:
- What is your understanding about diabetes mellitus?
Pathophysiology:
- It is a condition related to our blood sugar. Whatever we eat, the food contains different
components, including sugar. The food travels through the food pipe to our stomach, to our
bowels where it is absorbed and goes to all our body. Our organs (brain / muscle) use this sugar as
source of energy. In order for muscles to use this sugar, it needs a key to enter into cells, this key
is the insulin.
- We have two types of DM, type I and type II.
- Patients with DM type I, their body does not produce insulin, so we need to compensate for that
by giving it from external source.
Complications:
- High blood sugar is harmful for our bodies, because it affects all our blood vessels, the small and
big ones, and may give a lot of complications! It might cause kidney, eye, or nerves injury and
harm on the longer term.
- On the other hand, low blood sugar is even more dangerous; do you know why? Because our brain
can not survive without blood sugar for more than 5-7 minutes, it is the only source of energy to
our brains.
Prevention:
- What happened to you is a very serious condition, and it might happen again. The best way to treat
is to prevent this from happening; by:
o
Make sure that you always eat after your insulin dose
o
Monitor your blood sugar frequently
o
If you exercise, adjust your insulin dose based on your blood sugar level
- Now, if this happens again, do you know how to identify it before you totally lose your conscious?
o
Whenever you feel hungry / sweating / shaky / dizzy / heart racing
o
You need to stop, and immediately eat a candy / chocolate / juice
o
So, you need to keep glucose tablets in your bag, to take it in case of emergency
If you are at home; keep monitoring your blood sugar,
If you are out; reach to the nearest ER
Emergency measures:
- If you exercise, there is a special type of injections (glucagon emergency kit); if your blood sugar
drops suddenly, use it, or other people can use it to inject you.
- That is why it is important that you have a bracelet that mentions you are diabetic, so if you lose
conscious and some one finds you, they can identify the situation and provide help.
Follow-up:
- You should see your family physician within few days, and he can refer you to diabetes clinic,
for more education and assessment.
- I will still give you some brochures and web sites in case you would like to know more.
Notes: If you are the family physician, what referral will you do for a diabetic patient?
- Diabetes clinic / Foot specialist / Dietician
- If DM type I > 5 years, OR type II at any time: Ophthalmologist / Nephrologist / Neurologist

OSCE-guide-III.doc

Page 83 of 255

Emergency Medicine

Unconscious Seizing Patient DT / Epilepsy / Brain tumour /


Patient arrives to the ER with his wife, on the way he had attack of seizures, and received 1 dose
of diazepam, he is unconscious now. In the next 10 minutes; manage.
Introduction

Very brief introduction to wife, I will make sure he is stable then I will ask
you more questions
Mr ;
Patient is unresponsive, I will start my primary survey:
can you hear me
 A: can you open your mouth (open and comment) / trachea central / JV
not engorged
STABILIZE
 B: listen to lungs and apex / normal air entry on both sides / normal
heart sounds
 C: can I get the vitals please! Normal! 2 large IV lines please

One to give IV fluids 50 ml/hr to keep vein open (KVO)

The other one is to withdraw samples


 D1; deficits: pupillary reaction
 D3; drugs: universal antidotes  thiamine 100 mg / if O2 is ok, no need
for naloxone, if blood sugar is ok, no need for dextrose
If at any time, the patient starts to seize, give ativan 2 mg IV and reassess ABCD
History
Event
 First time to seize?
 Can you describe what happened? Did he fall to the ground?
(wife)
 Before he seized; did he shout? Starred at the wall? Complained of
strange smell?
 Was all his body seizing or part of it? For how long? Did he bite his
tongue? Rolled up eyes? Did he wet himself? Was breathing?
 Did he regain conscious alone or with intervention?
Cause
 History of epilepsy? Medications for epilepsy?
 And mood stabilizers medications?
 RECENTLY, did he complain of: Neurological / Constitutional
symptoms
 Any history of trauma / head injury?
 Recent ear infection?
 SAD: sweating / shaking
 Any medications / blood thinners
PMH
 Long term disease; e.g. HTN, DM, kidney, lung, or heart disease
 Previous hospitalization / surgeries?
 FH of epilepsy
FH
Examination
 Vitals from the examiner
 Glasgow coma scale (if < 8  arrange for intubation)
Neurological examination:
 Cranial nerve examination
 UL and LL: tone and reflexes
Management
 Stat neurology consult
 Stat CT brain

OSCE-guide-III.doc

Page 84 of 255

Emergency Medicine

N.B. Causes of seizing:


- Epilepsy relapse
- Brain tumour / brain hemorrhage
- Infection / meningitis
- Organ failure
- Electrolytes imbalance
- Delirium Tremens
- Withdrawal from sleeping pills
- Cocaine overdose
How to identify if this is narcotics overdose?
- While doing AB; if O2 saturation is low even after you give O2, and respiratory rate is
low; jump to shine light in the pupils  if pin-point-pupil  give stat naloxone 0.2 2.0
mg IV
- Then back to ABCD
Any unconscious patient:
- A; if fluids  suction
- B; if not breathing  mask and bag
- C
- D
- Then assess Glasgow coma scale, if < 8  intubate (ask for DNR here)
Common 5 causes for unconscious patients:
- Hypoglycemia / DKA
- Epilepsy
- Meningitis
- Stroke lateralization
- Heart block

OSCE-guide-III.doc

Page 85 of 255

Emergency Medicine

Heart Attack Chest Pain (MI or Heart Block)


4 cases:
- Chest pain for 45 minutes, with normal ECG
- Chest pain for 45 minutes, with ST elevation
- V. fib: chest pain and the patient is intoxicated
- V. fid: chest pain and the patient is calm
Introduction
Brief history

ABCD

MONA

Oxygen
Aspirin
Nitrates

Morphine

-blockers

No ST elevation

ST elevation







How are you feeling right now?


 Os Cf D
 PQRST
 ... position or deep breathing
 Patient is talking to me  air ways ok.
 Listen to chest to exclude pneumothorax  breathing is ok
 C and D briefly
 4 L O2 via nasal canula
 Aspirin 325 mg
 Plavix 300 mg
 0.3 mg puff X 3 times; 5 minutes apart
- If no low BP
- If the pt did not consume Viagra recently
 If patient is in pain after 3 puffs of nitrates and there is NO
hypotension or bradycardia  morphine 2 mg
- If antero-lateral infarction (V2/3/4/6 & aVL): 5 mg morphine
- If (II, III, aVF)  usually involves the right ventricle, and the
morphine increase the pre-load  2 mg morphine only
Given to all patients; EXCEPT:
 History of asthma
 BP, HR, heart block, and inferior MI (II, III, aVF)
 Cocaine (unopposed alpha  blood pressure)  labetalol 20 mg
(2 mg/min for 10 min)
You give metoprolol 2 mg IV
- Stat cardiology consult
- Catheterization lab  angiography
 Heparin (ask about the hospital protocol for heparin usually 5000
U IV then 1000 U/hr by infusion) or give enoxaparin 1 mg/ kg)
- Stat cardiology consult
- Catheterization lab  angioplasty
 Heparin (enoxaparin 1 mg/ kg)
 If the cardiologist is not available  start thrombolytics, but first
rule out the contra-indications for thrombolytics:
- 2 recent (< 2 weeks): Recent surgery / Recent bleeding
- 2 brain: Brain tumour / Bleeding in brain stroke
- 1 heart: Aortic dissection

Chest pain presents with heart racing / SOB / nausea / vomiting / sweating
History will be: chest pain analysis / cardiac symptoms / risk factors
If blood pressure is low: we only give oxygen / aspirin / and plavix
If inferior MI (II, III, aVF)  I need 15 lead ECG / do not give -blockers
Risk of bleeding with thrombolytics is 1%, but being serious, this needs consent

OSCE-guide-III.doc

Page 86 of 255

Emergency Medicine

Case 1: Chest pain with initial normal ECG


Chest pain for 45 minutes, manage
Introduction
Pain history

ABCD

Brief history

How are you feeling right now?


Os Cf D
PQRST (R: how about your jaws / back / arms)
... position or deep breathing
Start O2 4 L by nasal canula
vitals please: comment and ask for regular VS check
2 IV lines: KVO / take samples and serial ECG
- ECG normal: order Serial ECG every 5 minutes / if any VS / any
change in pain
- How did you come here today?
- Did you take any aspirin? aspirin 325 mg / plavix 300 mg
- Have you recently used Viagra? If no  nitroglycerine puffs 0.3 mg X
3; 5 minutes apart
 D1: brief neuro exam  pt is grossly neurological free
 Any results back?
 Order chest x-ray?
 Cardiac symptoms: heart racing / SOB / nausea / vomiting / sweating /
dizziness / LOC
 Chest symptoms
 Constitutional symptoms
 DD: GIT (difficulty swallowing / acidic taste / GERD / heart burn / hx of
PUD) / Trauma / relation to respiration / calf pain / swelling
 PMH: medications / blood pressure / blood glucose / cholesterol / SAD
(especially for smoking and cocaine) / stress
 FH: heart diseases / heart attacks







Another ECG
Normal ECG

Do physical exam:
Vitals / compare BP in both arms ? aortic dissection
General status of the patient
Eyes / mouth
Heart and chest examination
LL edema







OSCE-guide-III.doc

Page 87 of 255

Emergency Medicine

ST elevation










Morphine 2 mg (up to 5 mg in antero-lateral MI)


-blockers: metoprolol 2 mg IV (if NO contraindications: history of
asthma, BP, HR, heart block, inferior MI, cocaine)
Mr , based on your symptoms and your ECG, it seems that you
have a heart attack. (Am I dying Dr?). This is a serious condition,
however you are stable and doing well, you are in a safe place and
we have experience in dealing with that. Heart attack means one or
more arteries providing blood supply to your heart are blocked by a
clot, we need to reopen this clot.
Stat cardiology consult
Do we have catheterization lab?
What we need to do now is to use medication called thrombolytic
or clot buster. I would like first to make sure that you are a good
candidate; recently have you had any surgery or bleeding? Have you
ever had stroke before? Were you diagnosed or told that you have
brain mass? Lesion? Based on the ECG, I do not see signs consistent
with cardiac aneurysm, and based on the BP, it does not look like
having dissection. Based on this, I can say that you are a good
candidate for thrombolytics, it is a life saving procedure, however
like any other medical intervention, it has side effects; it might cause
bleeding in 1% of cases, this includes stroke, so we need your
consent if you would like to have it!
Heparin (ask about the hospital protocol for heparin usually 5000 U IV
then 1000 U/hr by infusion) or give enoxaparin 1 mg/ kg)
Can you get the thrombolytics kit please

ST elevation:

Lateral MI

Inferior MI

The patient wants to leave the hospital:


- I would like to make sure he is competent, not under influence of alcohol or any
substance, and to rule out suicidal ideation
- I would explain to the patient: diagnosis / treatment / side effects of treatment /
complications of not receiving treatment / alternatives
- I will document this, and I will ask the patient to sign a LAMA (leaving against medical
advice), and I will let him go

OSCE-guide-III.doc

Page 88 of 255

Emergency Medicine

Case 2: Chest pain with STEMI


Introduction
How are you feeling right now?
 Os Cf D
Pain history
 PQRST (R: how about your jaws / back / arms)
 ... position or deep breathing
ABCD
 Start O2 4 L by nasal canula
 vitals please: comment and ask for regular VS check
 2 IV lines: KVO / take samples and serial ECG
- ECG: ST elevation in V 2/3/4, aVL  ant-lat MI
- How did you come here today?
- Did you take any aspirin? aspirin 325 mg / plavix 300 mg
- Have you recently used Viagra? If no  nitroglycerine puffs 0.3 mg X
3; 5 minutes apart
 D1: brief neuro exam  pt is grossly neurological free
 Any results back?
 Order chest x-ray?
 Cardiac symptoms: heart racing / SOB / nausea / vomiting / sweating /
Brief history
dizziness / LOC
 Chest symptoms
 Constitutional symptoms
 RISK factors
 PMH: medications / blood pressure / blood glucose / cholesterol / SAD
(especially for cocaine) / stress
 FH: heart diseases / heart attacks
ST elevation
 Morphine 2 mg (up to 5 mg in antero-lateral MI)
 -blockers: metoprolol 2 mg IV (if NO contraindications: history of
asthma, BP, HR, heart block, inferior MI, cocaine overdose)
Mr , based on your symptoms and your ECG, it seems that you
have a heart attack. (Am I dying Dr?). This is a serious condition,
however you are stable and doing well, you are in a safe place and
we have experience in dealing with that. Heart attack means one or
more arteries providing blood supply to your heart are blocked by a
clot, we need to reopen this clot.
 Stat cardiology consult
 Do we have catheterization lab?
What we need to do now is to use medication called thrombolytic
or clot buster. I would like first to make sure that you are a good
candidate; recently have you had any surgery or bleeding? Have you
ever had stroke before? Were you diagnosed or told that you have
brain mass? Lesion? Based on the ECG, I do not see signs consistent
with cardiac aneurysm, and based on the BP, it does not look like
having dissection. Based on this, I can say that you are a good
candidate for thrombolytics, it is a life saving procedure, however
like any other medical intervention, it has side effects; it might cause
bleeding in 1% of cases, this includes stroke, so we need your
consent if you would like to have it!
 Heparin (hospital protocol or give enoxaparin 1 mg/ kg)
 Can you get the thrombolytics kit please
Have time
Physical exam: listen to lung bases / S1 and S2 / compare BP both arms

OSCE-guide-III.doc

Page 89 of 255

Emergency Medicine

Case 3: Chest pain v fibrillation / v tachy


-

I would like to initiate code blue ACLS


Is this is the patient ECG? Is it the last one? We need to start defibrillation!
o Done.
o Can you give me the last ECG please?  NORMAL ECG
How do you feel? Do you feel drowsy? Because you have just had cardiac arrest, we
would like to make sure you are stable.
Manage as the first case  the chest pain with normal ECG

Case 4: Chest pain v fibrillation intoxicated patient


-

I would like to initiate code blue ACLS


Is this is the patient ECG? Is it the last one? We need to start defibrillation!
o
Done.
o
Can you give me the last ECG please?  NORMAL ECG
How do you feel? Do you feel drowsy? Because you have just had cardiac arrest, we
would like to make sure you are stable.
The patient will be rude / angry / aggressive. ? intoxicated
o
If the nurse gave you cocaine and said it was found with the patient; ask her to
keep it with the hospital security
o
Mr you have just had a cardiac arrest, this is a very serious condition, and
might happen again, you should not leave, it is important to stay; we want to
make sure you are stable.
o
If agitated  1 mg lorazepam injection

Cocaine (arrhythmias / HTN / neurological manifestations):


- What were you doing in the party? Any alcohol? Drugs?
- We will give you a medication that interacts with cocaine, it is very important to tell us if
you took any cocaine recently! (if cocaine  Ca channel blockers)
- Any weakness / numbness / tingling / vision problems / hearing problems?
- Do you hear voices / see things? Do you think someone want to hurt you?
-

Manage as the first case  the chest pain with normal ECG

OSCE-guide-III.doc

Page 90 of 255

Emergency Medicine

Heart Block
2 cases:
- One of them is DNR (must be dated, valid, and signed)
- The other case is: do not intubate / do not defibrillate. You can still pace maker
1- Introduction:
- Is this is the last ECG for this patient? I do not see any signs of V. fib or V. tachy. I
would like to see the patient first to make sure he is stable, and then I will look at the
ECG.
- Mr I am Dr working in the ER, do you hear me?
- I would like to activate the ACLS code please / start primary survey
 Check the mouth, listen for patent air way
 Give 4L O2 via
A
 What is the O2 please
nasal canula
B
 Trachea central, chest is moving
 Monitor O2 for
 Listen to lungs, heart
need to intubation
 I would like to get the vital signs please;
C
 2 large IV lines; for IV fluids and to withdraw samples
2- Glasgow Coma Scale  [if < 8  intubate (ask about DNR)]
- This patient Glasgow coma scale is
- What is this patient code status? Any advanced directives?
o Is it signed, dated and valid?
o What does he have?
- We will respect his wishes, we will not (if DNR  do nothing)
- If no DNR:
D2 Third degree heart block:
 1 mg atropine  any changes?
 Pace maker:
- Rate: 20 more than his base heart rate
- Leads on sternum and apex
D1 Brief neurological scan, pupils,
Collateral
 Do we have his file?
history
 Can we contact his family physician? Or family member?
 Does he have a med alert?
 Do we have paramedics report?
 Can we check his belongings? He is taking , thiazides and digoxin
+
 I need to get his K level / digoxin level and I would like to check the dose for
digi-bind and digi-fib
Physical
 Neuro exam
exam
 Cardiac exam
Notes:
- For any unconscious patient: ask about advanced directives or DNR! What is this patient
code status?
- Whenever the examiner or the nurse tries to give you an ECG at the room entrance,
assess for V. fib or V. tachy and report: there are no signs of V. fib or V. tachy. I would
like to see the patient first to make sure he is stable.

OSCE-guide-III.doc

Page 91 of 255

Emergency Medicine

Headache
Introduction
CC
Analysis of CC

Headache for 2 hours (very acute very serious)


Os Cf D
 Is this your first time
 Did you get any trauma?
 Would you describe it as the worst headache in your life? Thunder
clap?
- Can you please lie down?  Put the bed 45, I would like to make
sure u r stable!
- ABCD: IV lines / D1: Pupils
 PQRST
 ... position or coughing
 Acute neuro: fever / neck pain / stiffness / vision / hearing / gait / falls
/ weakness / numbness
 The patient says: I am diabetic  stat blood glucose (prick)
HTN / blood thinners / kidney diseases
Kidney cysts / disease / aneurysm
Cocaine

Associated
symptoms
PMH
FH
SH

If the patient loses his conscious  manage as unconscious patient


 If BP and HR  I am suspecting Cushing triad, put the bed in 45
ABCD
 D1: Pupils, D3: thiamine 100 mg
Secondary
 Glasgow coma scale
As in the case of the
survey
unconscious patient
Physical exam

Management






Orientation (if conscious)


Quick cranial nerves
Quick neuro (tone/ motor / sensory / reflexes)
Nuchal rigidity, Kernig's, Brudzinski's signs






CT scan stat LP
Others: Chest x-ray / Urinalysis
Blood works (CBC / differential / lytes)
Septic workup (samples / C&S)






OSCE-guide-III.doc

Stat neurosurgery consult


Empiric antibiotics
IV mannitol 20% 1 gm / kg
Intubate:
- To keep air ways patent
- To intra-cranial pressure

? subarachnoid
hemorrhage
? meningitis
(FEVER)

Treatment of
meningitis:
vancomycin 1 g IV
q12h + ceftriaxone
2 g IV q12h
ampicillin 2 g IV
q4h (if >50 years or
hx of alcohol use or
immunocompromise
)

Page 92 of 255

Emergency Medicine

Acute Abdominal Pain


OsCfD /+/ PQRST  will give you an idea
In abdominal pain cases:

If you suspect certain diagnosis  scan for the risk factors for that disease

If no diagnosis  continue GIT symptoms and general differential diagnosis
You suspect obstruction  nausea / vomiting (COCA+B / coffee ground material) AND bowel
movements  if vomiting  screen for dehydration
 If you dx obstruction  check risk factors of obst  then rest of GI symptoms
 If not obst  scan GIT  near-by systems  PMH for systemic disease
If you suspect kidney stones  screen with renal symptoms
 If you dx renal stone  check risk factors (diet, medications, hx of renal stones, uric
acid, bone pains / fractures)  then rest of urinary symptoms
Intestinal obstruction
Intro
Analysis of
CC

But first I would like to ask you, how do you feel now?
Analysis: OsCfD: gradual, started colicky, and now continuous dull pain /
PQRST / What or (position / eating / bowel movements / vomiting)
 Screen for obstruction:

Nausea/ vomiting
o
Relation to pain, which started first, does it relief pain
o
COCA + Blood (coffee ground material)
Impact
 Screen dehydration (dizziness / light headedness / thirsty / LOC)
 Bowel movements

How about any blood? Any time?

Still passing gas?


Red flags
Risk factors for intestinal obstruction:
 Previous surgery? What? When?
 Fever/ night sweats/ chills / appetite / loss of weight / lumps & bumps
 PMH or FH of cancer or benign tumour
 Hx of Crohns disease (hx of abd pain/ bloody diarrhea) / family hx
 Hx of hernia / groin mass
 Gall bladder stones / right upper quadrant pain
Differential  Gastroenteritis:
diagnosis

What did you eat yesterday? Place that you are not used to?

Diarrhea / blood in stools?

Anybody else ate with you and suffered from the same problem
 Renal: flank pain / burning sensation / going more to washroom / stone
 Liver: yellowish discoloration / itching / dark urine/ pale stools
 Hx of HTN / SOB / cough / phlegm (aortic dissection)
PMH / FH / SH



X-ray findings of small intestinal obstruction: (1) Multiple air/fluid levels, (2) Dilated loops
of small intestine, (3) No air under the diaphragm.
Management: (1) NPO / NG tube, (2) Oxygen mask, (3) IV fluids, (4) Stat surgical consult,
(5) Foleys catheter, (6) Correct electrolytes.

OSCE-guide-III.doc

Page 93 of 255

Emergency Medicine

Right / Left lower quadrant abdominal pain middle aged female


 History: nausea / vomiting / fever / LMP / vag discharge / risky behaviour
 Physical exam: MUST mention pelvic and bimanual exam
 DD: (Left side): ectopic / PID / ovarian cyst / torsion / kidney stone
 DD: (Right side): ectopic / PID / ovarian cyst / torsion / kidney stone / appendicitis / intest
obstruction
 Investigations: -HCG / CBC / differential / abd-pelvic US / culture of the vag and cervical
secretions
 Long term complications: abscess / ectopic / infertility / intestinal obstruction / peritonitis
 Signs at PV exam: left adnexal mass / cervical motion tenderness
Acute abdomen in a female  missed period (ectopic), bleeding (abortion), discharge (PID)
PID
 Diagnostic plan:

Pregnancy test -HCG

CBC / ESR

Cervical culture (for Gonorrhea and Chlamydia)

Syphilis serology
 What is the treatment of pelvic inflammatory disease?

Cefoxitin 2 g IV every 6 hours X 2 days (covers anaerobic bacteria)

Doxycycline 100 mg orally BID X 2 weeks

Remove any IUD (if present)


 What are the indications of hospitalizing the patient?
(1) Pregnancy, (2) Pelvic abscess on U/S scanning / high fever (> 38.5 C), (3) PID at young
age, (4) Recurrent PIDs, (5) Failure to respond to outpatient management, (6)
Immunodeficiency (patients with HIV infection) or severe illness
 Complications of PID: abscess / ectopic / infertility / intestinal obstruction / peritonitis
Left lower quadrant abdominal pain elderly  Diverticulitis
 Signs:

Fever

Peritoneal signs: +ve cough tenderness / percussion tenderness / tenderness / rebound


tenderness / DRE: severe pain
 DD: diverticulitis / abscess / cancer
 Investigations:

AXR, upright CXR:


o
May be normal
o
Localized diverticulitis (ileus, thickened wall, SBO, partial colonic obstruction)
o
Free air may be seen in 30% with perforation and generalized peritonitis

CT scan (optimal method of investigation). 97% sensitive, very useful for assessment of
severity and prognosis. Very helpful in localizing an abscess

Hypaque (water soluble) enema SAFE (under low pressure):


o
Saw-tooth pattern (colonic spasm)
o
May show site of perforation, abscess cavities or sinus tracts, fistulas

Barium enema: contraindicated during an acute attack: risk of chemical peritonitis


(perforation)

Sigmoidoscopy/colonoscopy:
o
Not during an acute attack, only done on an elective basis
o
Take biopsies to rule out other diagnoses (polyps, malignancy)

OSCE-guide-III.doc

Page 94 of 255

Emergency Medicine




Management: (1) NPO / NG tube, (2) Oxygen mask, (3) IV fluids, (4) Stat surgical consult
(5) IV antibiotics (IV ciprofloxacin 500 mg BID / IV Metronidazole 500 mg TID)
Indications for surgery for diverticulitis:

Unstable patient with peritonitis

Hinchey stage 2-4 (large abscess / fistula / ruptured abscess / peritonitis)

After 1 attack if: (a) immuno-suppressed, (b) abscess needing percutaneous drainage

Consider after 2 or more attacks, recent trend is toward conservative management of


recurrent mild/moderate attacks

Diabetic ketoacidosis  acute abdominal pain


Analysis

Impact
Red flags
DKA

DD causes of
DKA

Management of
DKA


























OSCE-guide-III.doc

Pain: OCD / PQRST /


Vomiting: number of episodes + COCA
AS: fever / malaise / cough / urinary symptoms / diarrhea /
gastroenteritis
Diabetic history
Dehydration: feel weak / pee less / dry tongue and skin
Hyperglycemia symptoms: thirst, polyuria, polydipsia, and nocturia
Acidosis: shallow rapid breathing or air hunger (Kussmaul or sighing
respiration), abdominal tenderness, and disturbance of consciousness
Complications; of diabetes mellitus
Non-compliance OR wrong dose
Recent surgery
Pregnancy
Trauma
MI
Infection
IV fluids NS (1 L/hr x 2 hrs then 500 ml/hr x 2 hrs then 250 ml/hr
x 4 hrs)
Foleys catheter
Insulin drip 2 units / hour check glucose and lytes every 2 hours
When glucose reaches down to 15  fluids will continue as
maintenance, 2/3 : 1/3 of D5W : NS + 20 mEq KCl/L. 4:2:1 rule: 4
ml/kg/hr for the first 10 kg, then 2 ml/kg/hr for the next 10, then 1
ml/kg/hr for the next whatever
Serial blood glucose
ABG / serum ketones
CBC / lytes
Septic workup (chest x-ray / blood cultures / urinalysis)
ECG (for the in K+)

Page 95 of 255

Emergency Medicine

Acute Abdomen
Patient came to ER with abdominal pain / vomiting / diarrhea / BP 90/60 / HR 120
Acute pancreatitis / dissecting aortic aneurysm (no vomiting) / perforated peptic ulcer
Introduction

Analysis

Impact
Red flags
DD

PMH

I can see that you have a lot of pain, bear with me for few minutes and I will
give you a pain killer as soon as I can.
 In the moment, I would like to make sure you are stable
 What are the vitals pleas?
Stable
Unstable
Proceed to
I am going to start my primary survey  ABCD
 When you send blood works: add lipase / amylase
history

Did you vomit blood? How about coffee ground? (if yes: order
blood)
Os Cf D / PQRST / / relation to position / breathing / eating
 Vomiting  COCA + Blood
 Change in the bowel movements
 Dehydration
 How do you feel right now? What are the vitals please?
 Constitutional symptoms
Liver / GB
 Yellowish discoloration / itching / dark urine / pale stools?
 Recent flu-like illness?
 Do you have hx of gall bladder stones? Repeated attacks abd
pain?
Stomach
 Hx of PUD? GERD? Acidic taste / heart burn?
 Alcohol? How much? When was the last time? Did u drink >
usual?
 Gastroenteritis (What did you eat yesterday? Place that you
are not used to? Diarrhea / blood in stools? Anybody else ate
with you and suffered from the same problem?)
Medications  If vomited blood: Do you take steroids / NSAIDs / blood
thinners?
Kidney
 Flank pain? Burning sensation? Dark urine? Frequency?
Aorta
 Hx of HTN / atherosclerosis / DM / cholesterol / smoking /
SOB
Trauma
 Did you have trauma?
 Medications / allergies / long term disease?


Physical exam: abdominal exam


Face
Eyes for jaundice / mouth for dehydration / ulcers
7
8
Inspection
 No Cullens sign / no Grey Turner sign
 Cough tenderness

Cullens sign: peri-umbilical ecchymosis. It arises from spread of retroperitoneal blood associated with: pancreatitis / ruptured
ectopic preg / ruptured aortic aneurysm / ruptured spleen / perforated duodenal ulcer
8
Grey-Turner sign: ecchymoses of the skin of the flanks, also with retroperitoneal bleeding

OSCE-guide-III.doc

Page 96 of 255

Emergency Medicine

Auscultation
Percussion
Palpation

Patient in severe pain:


I can see that you are in pain, can you bear with me for few minutes
then I will give you pain killer as soon as I can.

In order for me to examine you properly, I need to get good look at your
abdomen, for that reason, can you please lie on your back!

Do you feel relieved like that, I understand. It will be only few minutes,
do you want to give it a try? Slowly! Do you want me to help you! It is
crucial to reach a proper diagnosis!

If still refusing  offer 2 mg morphine S.C. Finally she will lie down.
ASK FOR X-RAY (3 view x-ray abdomen)  ? perforation

Perforation  severe guarding, will not be able to proceed

Obstruction

Patient is obviously in severe pain, I will not be able proceed with examination
Management

I am suspecting acute peritonitis


(? perforated peptic ulcer)
 NPO / nasogastric tube
 O2 mask
 IV fluids (250 ml/hr) / Cross
match blood

I am suspecting pancreatitis




NPO / nasogastric tube


O2 mask
ICU admission

Pain killer: Meperidine 1 mg / kg


or Fentanyl
Stat SURGICAL consult

Pain killer: Meperidine 1 mg / kg


or Fentanyl
Stat GIT consult





3 view abdominal x-ray


Abd U/S
CT abdomen





3 view abdominal x-ray


Abd U/S for gall stones
CT abdomen

Lab: CBC / blood sugar / calcium


/ amylase / lipase

Lab: CBC / blood sugar / calcium /


amylase / lipase
Albumin level / serum Ca

DD:
 Perforated PUD:
vomiting coffee ground material
 Aortic dissection: NO vomiting / severe pain shooting to the back
 Acute pancreatitis:

NO upper GIT bleeding

Fever (due to chemical irritation not infection)

Pain improves when leaning forward

Paralytic ileus

Tetany
Ethical question:
The patient girl friend is on the phone, she is asking about his condition?!

I am still doing my examination,

I can assure you that he is well taken care of, and we will do our best to help him,

All the details of his medical information is absolutely confidential, and I can not release

OSCE-guide-III.doc

Page 97 of 255

Emergency Medicine

Upper GIT bleeding


Patient vomited blood; esophageal: varices / gastric: perforated PUD
Introduction
ABCD
Analysis

Impact
Red flags
DD

PMH
FH
SH

Because this is a case of GI bleeding, protection


Mr , make sure that you are stable, then I will ask you
How do you feel right now?
Be aggressive with fluids 2 L IV
 Os Cf D / COCA + dehydration
 Associated with pain (painless: esophagus / pain: stomach)  PQRST
 Nausea / vomiting / diarrhea
 Recently; any blood in stools / dark stools?
 Weak / drowsy /
 Constitutional symptoms cancer stomach
Liver
 Yellow / itchy / urine / stool
 Alcohol; how much / for how long / more than usual? SAD?
 Long term liver disease? Abdomen size? Bruises?
Memory? Concentration? Numbness LL?
 Risk factors for hepatitis: piercings, bld transfusion,
unprotected sex
Stomach
 History of PUD / pain / GERD, heart burn
 Risk factors for ca stomach: smoking
 Do you take any blood thinners? Aspirin? Steroids? NSAIDs?
Medications
Any medications / allergies
Family history of gastric cancer




or fluids (if stable: fluids 250 ml/hr) and monitor vitals


General appearance of the patient: cachectic / distressed /
I would like to check if there is any postural drop in the blood pressure
Exam
Liver exam: extra-hepatic signs of liver cell failure /+/ Bruises
Abdominal exam: epigastric mass / pain / liver / ascites
If painful: manage as acute abdomen case (perforated PUD)
Management
STAT GIT consult for UPPER GIT endoscopy
IV pantoprazole (80 mg bolus then 8 mg/hr)
IV octereotide (25 mcg/hr)  portal circ VD  portal pressure
Abdominal x-ray
Admission to ICU
Longer term management:
 If portal HTN: non-selective -blockers
 Advice on cutting down the alcohol
 Advice to follow-up with the family doctor
Vitals
General












OSCE-guide-III.doc

Page 98 of 255

Emergency Medicine

Lower GIT bleeding


Introduction
 Because this is a case of GI bleeding, protection
 Mr , make sure that you are stable, then I will ask you
 How do you feel right now?
ABCD
Be aggressive with fluids 2 L IV
 D1: you might find some deficits  previous cerebro-vascular accident (may
be history of weakness and numbness)
Analysis
 Os Cf D / COCA + dehydration
 Associated with abdominal pain  PQRST / / related to meals
 Vomiting blood? Nausea / vomiting?
Impact
 Weak / drowsy /
Red flags
 Constitutional symptoms cancer colon
DD
Liver
 Liver: yellow / itchy / urine / stool
Colon
 Hx of dx / screening / for Ca colon? When? What was the
result?
 Recent change in bowel movement?
 Recent changes in calibre of stools?
 Risk factors for Ca colon: Fm hx of ca colon / diet ( fibres
fat)?
 Risk factors ischemic colitis? Atherosclerosis (similar to
CAD)?
Medications
 Do you take any blood thinners? Aspirin? Steroids? NSAIDs?
PMH
Any medications / allergies
FH
Family history of colon cancer
SH
SAD
Vitals
or fluids (if stable: fluids 250 ml/hr)
General
 General appearance of the patient: cachectic / distressed /
 I would like to check if there is any postural drop in the blood pressure
Exam
 Abdominal exam:

If soft abdomen, and no findings (PE disproportionate to the pain) 


ischemic colitis
Management  STAT SURGERY consult
 NPO / IV fluids / Foleys catheter
 Admission to ICU
 Abdominal x-ray
 CT angiography
DD

With pain

Cancer colon: chronic intermittent

Ischemic colitis: acute / sudden

Painless

Diverticulosis

Angiodysplasia

Upper GIT bleeding

Aspirin

Hemorrhoids

Colitis: Radiation / Infectious / Ischemic / IBD (UC > CD)


Abdominal x-ray findings:

Intestinal ischemia: thumb print sign

Ischemic colitis: pneumatosis intestinalis (coli)


N.B. ischemic bowel  metabolic acidosis

OSCE-guide-III.doc

Page 99 of 255

Emergency Medicine

ECG

Normal

V fib /+/ V tachy /+/ Torsades du pointes

A fib /+/ Atrial flutter

ST elevation:
o Pericarditis: all leads
o MI:
 V 2/3/4 V5/6, aVL: antero-lateral MI (left coronary)
 II, III, aVF: inferior MI (right coronary, posterior and inferior surfaces)

Hear block third degree /+/ Bundle branch block

Hyperkalemia /+/ Hypokalemia /+/ Hypercalcemia

Digitalis toxicity

OSCE-guide-III.doc

Page 100 of 255

Emergency Medicine

1. Rate:

Regular: 300/number of big squares (R-R)

Irregular: Number of Rs x 6
2. Rhythm  check for P wave in lead II
Tachyarrhythmias:

Sinus tachycardia

Irregular irregularities: A. Fibrillation

Saw teeth (regular irregularities): A. Flutter

Rapid SVT

Ventricular arrhythmias: Premature ventricular beats / V. Tachy / V. Fib / Torsades de
pointes
Bradyarrhythmias: (< 60/min)

Sinus bradycardia

Heart block:
- 1st degree: P-R intervals increasing, but every P  QRS.
- 2nd degree:
 Mobitz type I: P-R intervals increasing, with missing QRS
 Mobitz type II: P-R intervals constant, with missing QRS
- 3rd degree: P-P has a rate, and the R-R has another rate
3. Axis
Normally, QRS in leads I, II, III are positive (upwards ).

Right axis deviation: QRS in I is negative (downwards ); I and III facing.

Left axis deviation: QRS in II, III is negative (downwards ); I and III opponents.
Diagram showing how the polarity of the QRS complex in leads I, II, and III can be used to
estimate the heart's electrical axis in the frontal plane:

Lead I negative and aVF positive: Rt axis deviation / Lead I positive and aVF negative: Lt axis
deviation.
4. Bundle Branch Block:

Normally QRS in V1 is downwards , if in V1: QRS is upwards & wide: RBBB.

Wide QRS in V6 (M mountain): LBBB.
A mnemonic to remember ECG changes is WiLLiaM MaRRoW, i.e. with LBBB there is W in
V1 and M in V6 and with a RBBB there is M in V1 and W in V6

OSCE-guide-III.doc

Page 101 of 255

Emergency Medicine

5. ST segment:
Angina

STEMI AND Non-STEMI

6. Others:
Hypokalemia
ST segment depression, inverted T waves,
large U waves, and a slightly prolonged PR
interval.

OSCE-guide-III.doc

Hyperkalemia
1- Flat P wave
2- Wide QRS
3- Spiked T wave

Page 102 of 255

Emergency Medicine

Phone calls
THE SEIZING CHILD PHONE CALL
The mother is on the phone, panicked as her child is seizing for 3 minutes
Introduction
+ Reassurance

Stabilize

Analyze the event


During

After

Fever

Rule out
BINDE
PMH
FH
Counselling

- Good evening, I am Dr the physician in charge at the (clinic/ER), may


I know who is on the line?
- Before I proceed, I would like to know the name of your child, your
phone number and your address please! In case the line is disconnected,
hang on, and do not call me, I will call you back.
- What is your child weight?
- I know this is stressful time for you; however, I need your help here. I
will ask you some questions and give you some instructions.
- Is this his first time?
- Did you call the ambulance? I will ask the nurse to call for you
- Is your child alert/conscious? Is he breathing? What is his color?
- Can you support and monitor him, remove any object near him?
- Can you put him on his side?  analyze during
- Do not put anything in his mouth, do not fix him
- Monitor till he stops seizing  assess for focal symptoms  after
- Can you describe the seizure for me please!
- Is it the first time? Or did it happen before?
- Did you watch that?
- Did he lose conscious? Which happened first, seizing or LOC?
- Did he fall from height? Did he hit his head?
- Was he shaking? Certain part of the body or whole?
- Is he breathing? Did he turn blue?
- Any tongue biting? Did he wet himself? Roll up his eyes?
- How long did it last? How did it stop? Spontaneously?
- Did he regain consciousness? After he regains consciousness; is he
able to recognize you? Able to talk? To move? Confused?
- Does he feel any weakness, numbness?
- Does he have fever? When did it start? Any medical advice? Any
diagnosis? Treatment? Is he taking the medications? Any reason?
- Constitutional symptoms
Meningitis / pneumonia
Neurological deficits / diabetes mellitus
Febrile seizure / epilepsy
The seizing child counselling

Notes:

Febrile seizure vs. meningitis: 1st time send the ambulance, 2nd time: send the ambulance if:
the seizure is > 15 minutes or > 2 attacks in 24 hours

OSCE-guide-III.doc

Page 103 of 255

Emergency Medicine

THE CHILD SWALLOWED MEDICATIONS / CAUSTIC MATERIAL


The mother is on phone, panicked as her child swallowed medication / caustic material at home
Introduction
+ Reassurance

- Good evening, I am Dr the physician in charge at the (clinic/ER), may


I know who is on the line?
- Before I proceed, I would like to know the name of your child, your
phone number and your address please! In case the line is disconnected,
hang on, and do not call me, I will call you back.
- What is your child weight?
- I would like to ask you to take a deep breathe to calm down; I will need
your help here. I will ask you some questions and give you some
instructions, this is important for the sake of your child, ok.
Primary survey
- Is your child alert?
+ Stabilize
Yes
NO
- Good to hear that
- Do you know how to do CPR?
- Yes, start it now
- No, guide her:
- If < 8 yrs 5 / 1
- If > 8 yrs 15 / 2
ABC - Is he breathing? Talking to you?
- Can you check the child lips color?
- Does he recognize you?
- Can you check his mouth, and if there is any of the medications can
you take it out?
- If there are any remnants of the chemical on his mouth or face, can
you wash it with some water?
- Did you call the ambulance? I will ask the nurse to call the ambulance
and we will send it to your home now
- You need not to give him/her any thing by mouth
- You do not need to do anything; you can just hold him/her?
Analyze the event - Can you tell me what happened?
- Which medication did he take? If does not know; who is taking this
medication? Why for? Was it recently renewed? Is there vitamins?
Sleeping pills? Aspirin? When ambulance arrives, please give them any
bottle that the child might have taken medications from?
- How much did he take?
- For how long was he/she left alone? Any reason?
Is it the first time? - If yes, analyze the event
Or did this happen - Are there other children at home? Any history of repeated illnesses at
before?
home? Repeated visits to ER?
BINDE
- B: Was he a full term? Did he need special attention? Any congenital
anomalies?
- I: Are his/her shots up-to-date?
- N: Is he a fussy baby?
- D: Is he a difficult baby?
PMH

OSCE-guide-III.doc

Page 104 of 255

Emergency Medicine

EMERGENCY TRAUMA PHONE CALL


A nurse is calling you from a remote rural medical center; she has a patient of trauma after a car
accident, BP 90/60 and HR 120. Manage over the pho ne.
A/B: Mouth / O2 / inspect chest, neck / listen to lungs, heart
- I am Dr , the physician in charge in the emergency room; may I know who is on the
line?
- Can you describe the case / situation for me!
- Now, I would like to get your phone number (and address if reasonably far), and patient
name, and if by any chance the line gets disconnected, do not try to call me, just hang the
line and I will call you back, ok.
- Are you a nurse? Do you have someone else with you to help?
- Is the patient alert?
- Does he have a neck collar?
- Open his mouth, check it for any FB or blood, and make sure it is clear. Can you feel air
coming from patient mouth?
- What is his O2 saturation? Can you give him O2 please, 4 L/min via a nasal canula
- Can you open the neck collar window and check his trachea and neck jugular veins, is the
trachea central or not? Are the JV engorged?
- Can you look to his chest; is it bilateral symmetrical movements of the chest wall?
- Can you listen to air entry on both sides?
- I am suspecting tension pneumothorax, I would like you to insert a wide bore needle (16
or 14 G) into the second intercostal space, in the MCL (on the upper margin of the third
rib)
o If she said, I did not do it before, or I am afraid I can not do it, or who is
responsible for this: encourage her and tell her that you are responsible
- Did you get a gush of air?
o Yes, that is great, you have just saved the patient life
- Can you re-check the trachea centrality, jugular veins and HR again for me please
o If stable: can you please fix a chest tube in the 5th intercostal space, anterior
Axillary line
C: Vital signs / fluids / withdraw blood samples / D:
- Can you please insert two wide bore (16 G) IV lines in the patient ante-cubital fossae,
and give fluids (2 L NS bolus) and then withdraw samples for
- Can you find any source of bleeding?
- D3: Give universal antidotes:
o Give 100 mg thiamine
o If RR  naloxone
o If blood sugar dextrose
- Can you tell me his Glasgow coma scale! Check his papillary reaction (D1).
- I would like also to intubate the patient
What are the requirements to transfer patient from a center to another center?
- Accompanied by two trained medical personnel (paramedics, nurses, physicians)
- Intubated and on ventilator
- Secured IV lines and fluids
- pre-arrangement with the place that will receive the patient

OSCE-guide-III.doc

Page 105 of 255

Physical Examination

Physical Examination

OSCE-guide-III.doc

Page 106 of 255

Physical Examination

Medical Physical Exam

Always be nice and smile


Use the alcohol anti-septic
Introduction
Drape / cover the patient properly
Warm your hands before touching the patient
Warm your stethoscope
If you are going to make painful manoeuvres: explain before / apologize for the pain / do
not repeat
Make sure you put the patient in the proper position at ease
Whenever you notice the patient is in pain, tell him that you will give pain medication as
soon as you can
If you ask the patient to walk, make sure he will not fall, surround him
Make sure you talk to the patient: I am going to look, feel, press, tap, listen (use
simplified non-medical language)
Report to the examiner every thing in medical terms, by inspection , by palpation,
percussion,
Cover the patient after you finish
Thank the patient and the examiner

Introduction:
- Good evening Mr , I am Dr I am the physician working in the clinic today / I am the
physician in charge in the ER now. I understand that you are here because you have been
having For the next few minutes I am going to do physical exam for your and I will
need to ask you questions during my exam. Also, I will be asking you to do some
movements and manoeuvres, if you feel any discomfort or pain, please do not hesitate to
let me know and stop me
- If you have any questions or concerns please feel free to ask me / to bring it up
- If SOB: during my exam, if you feel that you can not continue, please stop me
If there is history taking and then physical exam:
- I will be asking you some questions, then I will do physical exam. Hopefully towards the
end, we reach a working plan
Vital signs:
- If vitals are given: based on the vitals, the patient is stable, I would like to proceed. Or
the patient is unstable! Or comment: with mil fever
- If the vitals are missing one; e.g. the temperature: ask about it specifically
- Vitals are not gives:
o I would like to get the vitals before I start!
o I am going to start my exam by measuring your vital signs that is your blood
pressure, heart rate. And I will start by measuring your heart rate

OSCE-guide-III.doc

Page 107 of 255

Physical Examination

Abdominal examination:
 Introduction / Vital signs / General inspection of the patient: pt is sitting comfortably
- Inspection
- Auscultation: bowel sounds / bruits (aortic / renal / iliac)
- Percussion
- Palpation: superficial / deep / special tests
Respiratory examination:
 Introduction / Vital signs / General inspection of the patient: pt is sitting comfortably
- Inspection: face / hand / neck / chest / back
- Palpation: tenderness / tactile fremitus / chest expansion
- Percussion: dullness / percussion note / diaphragmatic excursion
- Auscultation: regular / special tests
o Then end with cardiology exam
Cardiac examination:
 Introduction / Vital signs / General inspection of the patient: pt is sitting comfortably
- Inspection: face / hand / neck / chest / heart (PMI)
- Palpation: apex / left para-sternal areas for heaves / valvular areas for thrills
- Auscultation: in Z format A-P-T-M
o Leg exam for edema
o Lung bases
- If full CVS exam  peripheral vascular assessment: abdominal bruits / legs pulses
palpation / chest exam
Musculoskeletal examination:
 Introduction / Vital signs / General inspection of the patient: pt is sitting comfortably
- Inspection: SEADS (scars / erythema / atrophy / deformity / swelling) / specific findings
(bulk of muscles / bony symmetry)
- Palpation: (TTC) tenderness / temperature / crepitus / effusion
- ROM: active (if normal, NO need to do the passive) / passive / against resistance
- Special test: mechanical (shoulder / elbow / hip / knee / ankle)
o To complete my exam, I would like to do:





Check the pulses of the limb (upper or lower)


Brief neurological examination of the limb
One joint above and one joint below examination
The other side joint

Neurological exams:
 Introduction / Vital signs / General inspection of the patient: pt is sitting comfortably
- Orientation: what is your name sir? Where are you? Time? Place?
- Cranial nerves
- Upper and lower extremities:
o Inspection
o Palpation / bulk
o Tone
o Motor power
o Sensory
o Reflexes
- Gait / Romberg test
- Cerebellar signs / Coordination
- Cortical sensations: two points discrimination

OSCE-guide-III.doc

Page 108 of 255

Physical Examination

Abdominal examination:
- Introduction
- Vital signs
- General inspection of the patient:
o By general inspection, pt is lying down comfortably, no obvious distress
o Can I take a look at your eyes, would you please look downwards? No jaundice.
Upwards please? No pallor
o Would you please open your mouth: no signs of dehydration or vomiting
o Can I take a look at your hand?
 Temperature is fine / and skin is moist
 Normal capillary refill (< 3 seconds)
 No obvious clubbing
- I am going to drape you now!
o Bed flat
o Can you please put you hands to your sides
o Allowing the patient to bend his/her knees so that the soles of their feet rest on
the table will also relax the abdomen!
-

Inspection; I will take a look at your abdomen:


o By inspection: abdomen is flat, not distended, normal contour, umbilicus is
inverted, abd is moving with respiration, no scars from previous surgeries, no
bruises, no obvious bulge or mass
o Would you please look to the left side, can you please cough
 Check pt face for tenderness  there is cough tenderness
 If no  cough another time  no herniation along the middle line.
Thank you, you can look back if you wish

Auscultation; I am going to listen to your abdomen, warm my stethoscope:


o McBurneys point: normal bowel sounds (hyperactive / decreased)
o 2 inches above the umbilicus  no aortic bruits
o 2 inches above / 2 inches bilateral from umbilicus  no renal bruits
o 2 inches below / 2 inches bilateral from umbilicus  no iliac bruits

Percussion: now, I am going to tap on your abdomen, can you point to your painful are.
I am going to start away from there:
o Percuss in 2 X 2 lines, and percuss to side  for ascites
o No percussion dullness / normal tympanic percussion note / no percussion
tenderness / no ascites

Palpation:
o I am going to feel your abdomen. Start away from the painful area:
 I am checking (name the 4 quadrants or the 9 areas of the abdomen);
(NO) tenderness, guarding or rigidity
o I am going to apply more pressure now: no obvious masses, no organomegaly
o I am going to feel your kidneys now (bimanual)  no enlargement, no
tenderness of the kidneys
o I am going to do some special tests:
 Murphys sign (Rt costal margin)  can you take a deep breath
 Rebound tenderness: I am going to press and release my hand, can you
tell me which causes more pain! (any point except McBurneys)

OSCE-guide-III.doc

Page 109 of 255

Physical Examination




Press at McBurneys point  is it painful


Press left iliac area, tell me if there is pain:
Not painful
Pain to the left side: left iliac tenderness / negative Rovsing's
sign
Pain to the right side  positive Rovsing's sign
/+/+/+/+/+/+/+/ COVER THE PATIENT ABDOMEN /+/+/+/+/+/+/+/
 Psoas sign: Rt LL extended; I am going to press on your thighs down, can
you press up against my hand (actively flex thigh at the hip)
 Obturator sign: Rt hip flexed, knee flexed; can you relax yourself, I will
rotate your leg (int and ext rotation), if pain  +ve sign

Can you please sit up now; I am going to:


o Tap on your back, check the costo-vertebral angle for renal tenderness
o Listen to the back of your lungs  no crepitation
Can you please lie down; thank you.
I would like to complete my exam by doing pelvic and vaginal exam / and digital rectal
exam
o Inguinal LNs
o Pelvic, vaginal and bimanual exam; looking for:
 Bleeding and/or discharge
 Adnexal masses / tenderness
 Cervical motion tenderness
 To obtain samples for culture and sensitivity
o Digital rectal exam; looking for:
 Bleeding
 Piles / haemorrhoids
 Fissures

How to differentiate between spleen and kidney by palpation:


- Spleen changes position with respiration (goes down with inspiration)
- You can feel the lower border of the spleen
- Spleen does not ballot
If there is history of Crohns disease:
- Inspection:
o General: truncal obesity / moon-like face
Red eyes
Mouth: ulcers
o Hands: nail changes / clubbing / skin rash / psoriatic changes / joint tenderness
o Abdomen: stria / scars
o Lower limbs: erythema nodosum
o DRE: fissures / fistulae
- When he sits up  press on the sacro-iliac joints  sacroiliitis

OSCE-guide-III.doc

Page 110 of 255

Physical Examination

Liver Examination
Patient vomited blood 20 minutes ago, perform focused examination for liver cell failure
- Because the patient is vomiting blood, I would like to ask for protection for me and my
team please (gloves / gowns / masks / goggles)
- Introduction / Vital signs
- Orientation: I am going to ask you some questions which are part of my physical exam.
Do you know where you are now? What is the time? Do you know why you are here? 
Patient is oriented to people, time and place
- General:
o
o
o
o
o

o
o

Inspection:
o

2 X 2 lines tap
Liver: MCL (from above downwards and from down upwards) liver span
Spleen: ant axillary line  last intercostal space / ask pt to take deep breathe in / then repercuss for the spleen
Ascites: from midline to the side, no dullness  so there is no need to perform the
shifting dullness (to be clinically palpable: ascites > 500 ml)

Palpation:
o

Listen to liver: no hepatic rub / hum / or bruits


Listen to spleen: no rub / no hum

Percussion:
o
o
o

No bruises / normal hair distribution / no caput medusa / no dilatation of collateral veins /


no obvious ascites / no scratch marks

Auscultation
o
o

Patient is lying comfortably no signs of obvious distress


Eyes  sclera: no jaundice
Mouth: no fetor hepaticus / no clots / no vomitus
Face: no parotid gland enlargement
Hands: no clubbing / nail changes / palmar erythema / Dupuytrens contracture / atrophy
of thenar and hypothenar muscles / look for IV marks / stretch your hands please
(flapping tremors)
Can you please lower your gown: no gynecomastia / no spider nevi
Check lower limbs edema

Liver: start from the right iliac fossa and go upwards, while the patient is breathing in and
out (push during inspiration, do not move your hand from the patient)  margin of liver
is not palpable, not tender, and not nodular.
Spleen: patient elevates his LEFT side 45, support from left back. Start from above the
umbilicus towards the spleen  spleen is not palpable

I would like to complete my exam by doing:


o Digital rectal exam: for hemorrhage / piles
o Check for testicular atrophy






DD of hepatitis: (alcoholic / viral / drug-induced) / cirrhosis / liver cancer


Worst prognostic signs: ascites / encephalopathy
Investigations: AST:ALT > 2:1, GGT, albumin, INR, CBC
Treatment of encephalopathy: treatment of precipitating causes / Lactulose (15-30 ml tid) /
Rifaximin (550 mg PO bid)

DD of ascites: liver cirrhosis / renal failure / heart failure / TB / malignancy /


Hypoalbuminemia
Treatment of ascites: Sodium restriction (20-30 mEq/d) / Diuretic therapy / Therapeutic
paracentesis may be performed in patients who require rapid symptomatic relief for refractory
or tense ascites

OSCE-guide-III.doc

Page 111 of 255

Physical Examination

Nasal bleeding  Hematological Examination


-

Because the patient is bleeding, I would like to ask for protection for me and my team
please (gloves / gowns / masks / goggles)

Introduction
Vital signs

General:
o Patient is lying comfortably no signs of obvious distress
 Nose: open the speculum antero-posteriorly
 Use the otoscope for ENT (nose / ear / mouth)
o Look for bruises / petichae  if you find them  continue hematological exam

I ma going to feel your glands:


o Occipital / post-auricular / pre-auricular / sub mandibular / sub mental / anterior /
posterior cervical / supra-clavicular
o I would like to check the axillary LNs (ant / post / med / lat / apical)
o I would like to check the inguinal LNs:
 Superficial (transverse, along the groin crease)
 Deep:
Transverse: along the groin crease
Longitudinal: medial to the femoral artery
(Lat  Med: NAVAL Nerve Artery Vein LNs)
o I would like also to check the popliteal LNs
Check for bony pains: sternum and 3-5 spots on the vertebral column (I am going to press
on your back)

CAN YOU LIE DOWN PLEASE?


- Listen to heart / lungs
- Abdominal exam:
o Inspection
o Auscultation
o Percussion: liver / spleen
o Palpation: liver / spleen
o I would like to: perform vaginal and pelvic exam
-

Finally, I would like to do:


o Fundoscopic retinal exam
o Orientation
o Brief neurological exam:
 Hands: pull/push/ do you feel the same
 Legs: pull/push/ do you feel the same
Patient is grossly neurologically free
N.B. focal neurological deficits to DD between TTP and HUS.

OSCE-guide-III.doc

Page 112 of 255

Physical Examination

Questions:
- Diagnosis:
o ITP (Immune Thrombocytopenic Purpura)
 Most common cause of isolated thrombocytopenia
 Diagnosis of exclusion (i.e. isolated thrombocytopenia with no clinically
apparent cause)
- Investigations:
o CBC: thrombocytopenia
o Peripheral blood film: decreased platelets, giant platelets
Bleeding time: increased / PT and aPTT: normal
Anti-platelets antibodies
o Bone marrow: increased number of megakaryocytes (critical test to rule out other
causes of thrombocytopenia for age > 60 years; e.g. myelodysplasia)
o Markers of hemolysis: increased unconjugated bilirubin, increased LDH,
decreased haptoglobin
o Kidney function tests (urea / creatinine for HUS)
- Treatment:
o Steroids (methylprednisolone 1 g/d for 3 days, then prednisone 1.5 mg/kg/day)
o Immunoglobulins (if low platelet count): IVIG 1 g/kg/d X 2 days
o Splenectomy
o Vaccination (pneumococcus, meningococcus, HIB)
- DD:
o ITP (Immune Thrombocytopenic Purpura)
o TTP (Thrombotic Thrombocytopenic Purpura)
o HUS (Hemolytic Uremic Syndrome):
ITP
TTP
HUS

Remitting / relapsing

Predominantly adults

Predominantly children
course

Thrombocytopenia

Severe thrombocytopenia

Mild fever

Micro-angiopathic

Micro-angiopathic
hemolytic anemia
hemolytic anemia

Splenic discomfort
(MAHA)
(MAHA)
(mild engorgement)

Renal failure

Renal failure

Neurological symptoms
(headache, confusion,
focal deficits, seizures)

Fever

CBC and blood film: decreased platelets and schistocytes
Investigations
(both TIP, HUS)  PT, aPTT, fibrinogen: normal

Markers of hemolysis: increased unconjugated bilirubin.
increased LDH, decreased haptoglobin

Negative Coombs' test

Creatinine, urea, to follow renal function

Stool C+S (HUS)

Plasmapheresis steroids
Management
(both TIP, HUS)  Platelet transfusion is contraindicated (increased microvascular thrombosis)

Plasma infusion: if plasmapheresis is not immediately
available

TTP mortality 90% if untreated

OSCE-guide-III.doc

Page 113 of 255

Physical Examination

Chest Examination
- Introduction
- Vital signs; especially: tachypnea / temperature
- General inspection of the patient; comment on respiratory distress
- Inspection:
o General:

Neck; can you lower your gown please

Trachea is central, no accessory ms used for breathing / LNs

Chest:

Face: symmetrical, no nasal flaring / laboured breathing


Eyes: jaundice / pallor
Mouth / can you flip your tongue please: no central cyanosis / dehydration / exudates
or secretions
Hands: no nicotine staining / clubbing / peripheral cyanosis / normal capillary refill

Chest is symmetrical, no accessory ms used for breathing, no intercostal retraction


Look from the side: no increase in the A/P diameter of the chest / no barrel chest

Palpation; I am going to feel your chest


o Tenderness:

Check the ant chest wall: no tenderness

Tactile fremitus: can you say 99 for 4-5 times, whenever you feel my hands on
your chest  normal tactile fremitus

Any lung pathology  tactile fremitus ( conduction)


Any pleural pathology  tactile fremitus (insulation

o Chest expansion: normal chest expansion


/+/ I will continue to examine you from the back, then I will check the front again /+/ Can
you cross your arms please
- Inspection: chest is symmetrical, no scars, swellings, or deformity
- Palpation; I am going to feel your chest
o Tenderness: no tenderness
o Tactile fremitus: can you say 99 for 4-5 times, whenever you feel my hands on
your back
- Percussion; I am going to tap on your chest
o Apex (2) / back (6) / sides (2)  normal percussion note:
- Auscultation; now, I am going to listen to your back
o Whenever you feel the stethoscope touching your back, can you please take a
deep breathe in and out from your mouth
o Normal vesicular breathing sounds bilaterally, no rhonchi, no wheezes
- Now, I am going to demonstrate the special tests that should be done if there is
consolidation, with auscultation:
o Can you say letter E  egophony (normal: ee / over affected area: ay)
o Can you say 99  vocal fremitus
o Whisper 1/2/3: whispering pectorology ( in audibility)
/+/ I will now go to examine you from the front again /+/
- Percussion; I am going to tap on your chest on both sides
o Apex (2) / MCL (6) / sides (2)  normal percussion note:
- Auscultation; now, I am going to listen to your lungs
o Mid clavicular line both sides
o Normal vesicular breathing sounds bilaterally, no rhonchi, no wheezes
- Cardiology exam: Auscultation

OSCE-guide-III.doc

Page 114 of 255

Physical Examination

Pneumonia

CURB 65 score Pneumonia Clinical Prediction Tool


Confusion
Urea: > 7 mmol, BUN > 19
Respiratory rate: > 30 breaths / min
Blood pressure: systolic < 90 and diastolic < 60
65: or older in age
Management
01
Can treat as outpatient
23
Consider hospitalization
45
Consider ICU

Mortality
<5%
<10%
<30%

Pneumonia severity index: another clinical index (scored on age, resp rate, co-morbidities ),
used to determine whether to admit the patient to the hospital or not.

IDSA / ATS Community Acquired Pneumonia Treatment Guidelines 2007:


(IDSA: Infectious Diseases Society of America, ATS: American Thoracic Society)
Setting
Circumstances
Treatment
Medications
Outpatient Previously well
Macrolide
 Azithromycin 500 mg PO once,
OR
THEN 250 mg qDay for 4 day
Doxycycline
(total of 5 days)
 Clarithromycin 500 mg PO bid
for 7-14 days
Comorbidities, or Beta-lactam + Macrolide
 Levofloxacin 750 mg PO q24h
use of antibiotics OR
for 5 days
in last 3 months
Respiratory fluoroquinolones OR
 Amoxicillin 1000 mg PO tid
Inpatient
Ward
+ Clarithromycin 500 mg PO
bid
ICU9
3rd gen cephalosporin +
Ceftriaxone 1 g IV q24h
Macrolide
+ Azithromycin 500 mg IV q24h
OR
for 5 days (step to oral when
Beta-lactam + Macrolide
tolerated)
Other medications for the management of pneumonia:

Susceptible for pseudomonas / recent use (within 3 months) of antibiotics or cortisone:


piptazo (piperacillin / tazobactam); 3.375 gm IV q6h)

MRSA: Vancomycin 1 gm IV q24h


Pneumococcus vaccination:

Elderly patients > 65 yrs old

COPD

CHF

Cirrhotic

Cancer

Immunocompromised: steroids / HIV / DM / splenectomy

Leukemia / lymphoma
9

Especially in the presence of risk factors: group home / hospital infection / immunocompromised

OSCE-guide-III.doc

Page 115 of 255

Physical Examination

Case: HIV positive man C/O: SOB for 1 week / cough / fatigue

? Pneumonia (? PCP)

DD of unilateral lobar reticular pattern on CXR: PCP (pneumocystis pneumonia) / Kaposi


sarcoma in the lung / Lymphoma / Atypical pneumonia (mycoplasma / chlamydia)
DD of bilateral hilar infiltrates: PCP / TB / CMV
Signs of HIV: check the entire skin for Kaposi sarcoma / mouth and pharynx for thrush or oral
hairy leukoplakia / LNs / check the abdomen for liver and spleen
Investigations:
o O2 saturation / ABG
o CBC / differential / CD4 count
o LDH ( in 95% of PCP cases)
o Blood culture
o Sputum for cytology / culture / gram stain / acid fast stain
o Bronchoscopy: bronchial washing
Treatment of PCP:
o Septra DS (trimethoprim / sulfamethoxazole: 160mg/800mg): 2 tablets q8h x 14 days then
continue prophylactic 1 tab od
o Severe illness (PO2 < 70 mmHg):
 Admit
 O2
 Septra
 Steroids: prednisone 40 mg bid x 5 d then 40 mg od x 5 d then 20 mg od x 5 d
o If patient is allergic to TMP/SMZ:
 For treatment: IV pentamidine or atavaquone
 For prophylaxis:
Dapsone or atavaquone
HIV routine prophylaxis:
o Based on CD4 count:
 CD4 < 200  prophylaxis for PCP: TMP/SMZ or Dapsone
 CD4 < 100  prophylaxis for toxoplasma: TMP/SMZ or Dapsone
 CD4 < 50  prophylaxis for mycobacterium avium complex: azithromycin
(1/week) or clarithromycin (2/d)
o Regardless the CD4 count:
 PPD testing for TB, considered positive if > 5 mm induration  INH for 9 months
 Vaccinations for pneumococcus / influenza / hepatitis B

OSCE-guide-III.doc

Page 116 of 255

Physical Examination

Cardiac Examination Essential HTN case


+ 65 years old gentleman, with high blood pressure for 20 years, for follow-up visit
+ 35 years old gentleman with newly diagnosed high blood pressure, please assess
+ If 10 minutes case: vitals + examination
+ If 5 minutes: only examination
Analysis

Impact

Red flags

Introduction
HR and BP (lying down)
Sit up, relax for 2 minutes

HR and BP (sitting up)


General

Neck
Heart

Carotid
JVP
Inspection
Palpation

Auscultation

For how long? Medications?


How frequent do you check your BP? Is it controlled?
End organ damage? Proteinuria / visual changes / headaches /
confusion?
Other cardiovascular risk factors: smoking / DM / high
cholesterol / history of MI in the family

Can you please lie down


Get the systolic by pulse palpation, then increase 30 mmHg, then
auscultate

Orientation

Mouth for dehydration

Eyes for pallor

Lung bases auscultation

Sacral edema

Lower limb edema
No difference

Asses the radial pulse bilaterally

Hands: capillary refill / clubbing / temperature

Fundoscopic examination: disc edema / retinal hge

Carotids: bruits / palpate

JVP: measure / Kussmaul sign / hepato-jugular reflux

Symmetrical / PMI

Apex

Heaves (Lt para-sternal / with knuckles)

Thrills (A-P / with palmar aspect of distal phalanges of fingers)
Do timing by simultaneously checking the carotid pulse.

Areas A-P-T-M: normal S1, S2, no murmurs.

Look for S3, S4 at the apex (with the bell)

Aortic insufficiency murmur

Lung bases
Pulsation / Bruits (aortic / renal / iliac)
Inspection / Palpation (temperature / pulsations) / LL BP

Abdominal exam
Lower limbs exam
- Introduction
- Vital signs; especially: BP / HR
o Ask the patient to lie down, I am going to assess your blood pressure twice, one
while lying down, the other after you sit up
 I will start by measuring HR for 10 seconds; your HR today is which is
normal
 Do the BP: get the systolic by pulse palpation, then increase 30 mmHg, then
auscultate  your BP today is
o Can you please sit up on the edge of the bed, (feet dangled)? I need you to relax
for 2 minutes, before I proceed to other measurements, during which I am going
to ask you some questions:

OSCE-guide-III.doc

Page 117 of 255

Physical Examination








Measure the sitting HR and BP  no difference in readings  no orthostatic


hypotension (no changes: no 20 mmHg in systolic, no 10 mmHg in diastolic,
no 20 in HR)

General exam:
o Comment on the general status / respiratory distress of the patient
o Assess the radial pulse bilaterally: pulse is normal, regular, equal on both sides,
with no delay
o Hands: normal capillary refill / no clubbing / warm moist skin
o I would like to do fundoscopic examination (I am going to shine a light in your
eyes to examine the back of your eyes): looking for disc edema / cotton wool
exudates / retinal hemorrhage / nipping of the veins
I am going to put the bed in 45 position; examine your neck:
o Carotids:




Auscultate for bruits  use the bell


If no bruits  palpate the carotid arteries (ONE AT A TIME)
Pulse is regular, normal volume and contour

JVP:



Orientation: Do you know where you are? Why? Date?


Mouth for dehydration
Eyes for pallor
Can I listen to the back of your lungs (lung bases)  no crackles
Press against the sacrum for sacral edema
Press on the lower limbs, for edema

Measure the JVP


Kussmaul test: take a deep breathe and hold it, measure the JVP:
Normally it will  negative Kussmaul test
If JVP  positive Kussmaul test (restrictive cardio-myopathy,
constrictive pericarditis)
Press on the liver for 10 seconds  normally the JVP will for few seconds
then will  if it does not  hepato-jugular reflux

Chest inspection:
o Chest is symmetrical, no obvious pulsations
o PMI (point of maximum intensity) is not obvious / or is obvious
Palpation:
o Palpate the apex and locate it  apex is palpable at in the MCL, not
hyperdynamic (vol ovrld), not sustained (pres ovrld), not displaced
o Palpate the left para-sternal area by knuckles: no Rt ventricle heave
o Palpate the Aortic / Pulmonary areas by finger tips  no thrills
Auscultation:
o In Z format A-P-T-M: normal S1, S2, no murmurs
o Flip the bell; check the apex for S3, S4
o While listening to the left para sternal area: can you please take a deep breathe in,
exhale it out and hold, and lean forward  no enhancement of aortic
insufficiency murmurs
o Listen to the lung bases  no basal lung crackles
I am going to put the bed flat now:
o Abdominal exam: inspect for pulsations / auscultate for bruits
o Lower limb exam:




OSCE-guide-III.doc

Inspection: symmetry / SEADS / nails hypertrophy / skin changes


Palpation: temperature / pulsations / radio-femoral delay
I would like to measure the blood pressure in lower limb

Page 118 of 255

Physical Examination

Secondary Hypertension
A 25 years old young man with HTN;
Cardiac

Coarctation of the aorta

Cocaine (nasal septum)

Renal

Poly-cystic kidney

Renal artery stenosis

Endocrine

OCPs

Cushing disease / syndrome

Pheochromocytoma

Primary hyperaldosteronism

Hyper / hypo thyroidism

Patient disrobed to underwear, draped below waist


Introduction
Can you please lie down
Vitals

There is systolic and diastolic HTN

I want to rule out orthostatic hypotension (pheochromocytoma)

I would like to compare UL and LL (coarctation of aorta)

No tachycardia (pheochromocytoma / hyperthyroidism)

No bradycardia (hypothyroidism)
General

Orientation (brain edema)

Truncal obesity / cervical fat pad / moon face (Cushing)

Check for proximal ms weakness (Cushing)

Check the eyes from the side for proptosis, lid lag test
(hyperthyroidism)

Asses the radial pulse bilaterally

Hands: capillary refill / temperature / skin dry or wet (thyroid).
Stretch your hands please  fine tremors (hyperthyroidism)

Fundoscopic examination: disc edema / retinal hemorrhage
Neck
Thyroid

No masses

Palpate for the thyroid

Percuss for the retro-sternal thyroid dullness
JVP

JVP
Chest

Palpate the apex

Look for S3, S4 at the apex (with the bell)

Listen to lung bases
Abdominal exam

Inspection: no obvious masses, no stria, no caf au lait spots

Auscultation: bruits (aortic / renal)

Palpation: feel for renal / supra-renal mass
Lower limbs exam

Edema

Rule out femoral / radial delay
Brief neuro exam

Knee or ankle reflex

Notes:

Watch for labile white coat HTN


Any HTN in middle age  secondary HTN  most likely kidney disease

OSCE-guide-III.doc

Page 119 of 255

Physical Examination

Hypertension


Predisposing Factors
Family history

Sedentary lifestyle

Excessive salt intake /


Obesity

Smoking
fatty diet
Alcohol consumption

Male gender

African American
Stress

Age >30

Dyslipidemia
Diagnosis:

Visit ONE:
o
If hypertension urgency or emergency (sBP > 210 or dBP > 120) or end organ damage
(e.g. confusion)  diagnose HTN
o
Else (provided 2 more readings during same visit)
 Search for target organ damage: history (cardio-vascular risk factors) / examination
 Investigations:

CBC / Na+, K+ / fasting blood sugar / lipids (total cholesterol, HDL, LDL, TG)

Kidney function tests / Urinalysis / Renal Doppler

ECG / Echocardiogram

For secondary HTN: TSH / Plasma aldosterone / renin levels / 24 hours urine
for metanephrines / VMA
 Life style modifications ( salt / alcohol / cholesterol / exercise)
 Follow-up visit within 4 weeks

Visit TWO; within 4 weeks


o
If (target organ damage OR diabetes mellitus OR chronic kidney disease OR blood
pressure > 180/110 mmHg)  diagnose HTN
o
Else (BP: 140-179 / 90-109 mmHg)  24 hours BP monitor (diagnose if mean awake
sBP >135 mmHg and/or dBP > 85 mmHg or mean 24 hours sBP >130 mmHg and/or
dBP > 80 mmHg)
Management:

Target BP is < 140/90 mmHg, < 130/80 if DM or chronic kidney disease

Life style modifications (initial management):


o
Smoking cessation and decrease alcohol consumption
o
Diet: salt / cholesterol and saturated fats / follow Canada's Guide to Healthy Eating
o
Weight: maintain healthy BMI (18.5-24.9)
o
Moderate intensity dynamic exercise: 40-60 minutes, 4-6 times/week

Pharmacological:
o
First line: Diuretics; e.g. hydrochlorothiazide 12.5 25 mg PO od Except:
 DM:
ACEIs; Ramipril 2.5 5 mg PO od
 Gout:
Amlodipine (5 mg PO od) OR Candesartan (4 8 mg PO od)
 Elderly (especially if IHD):
ACEIs
-blockers:
metoprolol 25 mg bid
Especially if CHF / EXCEPT: asthma / bradycardia
 Pregnant:
Hydralazine: 10 mg PO qid for few days then 25 mg PO qid
OR -methyl dopa: 250 mg PO bid
 If > 3 cardiovascular RF: statins / ASA
o
If partial response to standard dose monotherapy, add another first-line drug
 Do NOT give -blockers and Ca ch blockers  may cause heart block
 Do NOT give ACE and ARBs  both K+,
 Available combinations: Altace plus (ramipril + diuretic) / Diovan H
o
Notes on ACEIs:
 Contraindications of ACEIs: Angio edema / Bilateral renal artery stenosis
 ACEIs are nephroprotective except in acute renal injury  nephrotoxic
 If patient on ACEIs developed cough  switch to ARBs

HTN emergency: Hydralazine: 20 40 mg IV or IM, repeated as necessary, decrease the dose in


case of renal impairment

OSCE-guide-III.doc

Page 120 of 255

Physical Examination

SOB shortness of breathe


Patient who had a car accident 24-48 hours ago developed SOB.
Complications: 1st day: atelectasis / fat embolism. 3rd day: DVT / PE

Brief cardio-pulmonary
hx
Vitals
General

Neck
Chest

Trachea
JVP
Inspection

Palpation

Percussion
Auscultation

Lower limbs exam

Common causes: Cardiac (CHF) / Lungs (asthma / COPD /


pneumonia / PE)
Life threatening conditions: MI / PE / pneumothorax / aortic
dissection
I would also like to rule out orthostatic hypotension (? hypovolemia)

General assessment of the patient

Orientation (SOB  hypoxia  confusion)

Face: eyes / no signs of respiratory distress / mouth (no signs of
dehydration / central cyanosis)

Hand: temperature / capillary refill / skin / no peripheral cyanosis
/ no nicotine staining / comment on both pulses

Sacral edema

Trachea centrality

JVP: not engorged

Symmetry

Accessory muscles breathing / intercostal retraction

Obvious pulsations / PMI

If car accident (bruises on chest wall)

Tenderness

Tactile fremitus

Apex

Para-sternal heaves

Chest back for any dullness

Heart (ask the patient to turn slightly to left)

Lungs: 8 spots scan for air entry

Lung bases

Inspection: SEADS

Palpation: temperature / capillary refill / peripheral edema /
pulses / press for pain in calf muscle area

Special tests:
- Measure leg circumference 10 cm below tibial tuberosity;
difference should be < 2.5 cm between the two calves
- Homans' sign10: there is pain in the calf or popliteal region
with examiner's abrupt dorsiflexion of the patient's foot at the
ankle while the knee is flexed to 90 degrees

Indications for intubation: ABG showing poor PO2 (60s) / elevated PCO2 (80s) / acidosis / GCS
score < 8

10

A positive Homans' sign does not positively diagnose DVT (poor positive predictive value), and also negative Homans' sign does
not rule out the DVT diagnosis (poor negative predictive value), and there is theoretical possibility of dislodging the DVT.

OSCE-guide-III.doc

Page 121 of 255

Physical Examination

DVT
Clots clinical probabilities / risk factors (Virchow's Triad):
Vascular injury (endothelial damage); e.g. recent surgery
Venous stasis; immobilization (post-MI, CHF, stroke, post-operative, obesity, long
travel or flight, chronic venous insufficiency) inhibits clearance and dilution of
coagulation factors
Hyper-coagulability; aging, surgery, trauma, malignancy, antiphospholipid antibody
syndrome, hormone related (pregnancy, OCP, HRT, SERMs)
DD: muscle strain or tear, lymphangitis or lymph obstruction, venous valvular insufficiency,
ruptured popliteal cysts, cellulitis, and arterial occlusive disease

Wells Clinical Score for Deep Venous Thrombosis


Clinical Parameter
Score
Active cancer (treatment ongoing, or within 6 mo or palliative)
+1
Paralysis or recent plaster immobilization of the lower extremities
+1
Recently bedridden for >3 d or major surgery < 4 wk
+1
Localized tenderness along the distribution of the deep venous system
+1
Swollen unilateral superficial veins (non-varicose)
+1
Entire leg swelling
+1
Calf swelling >3 cm compared with the asymptomatic leg
+1
Pitting edema (greater in the symptomatic leg)
+1
Previous DVT documented
+1
Alternative diagnosis (as likely or greater than that of DVT)
-2
A Wells score can be interpreted in a binary (likely vs. unlikely) or ternary (low, moderate, or
high probability) fashion.
Ternary interpretation
Binary interpretation
3
High probability
score 2 DVT likely
1 or 2
Moderate probability
score < 2 DVT unlikely
0
Low probability
In the unlikely group (low-to-moderate risk of DVT)  D-dimer assay (ELISA):
Negative  rule-out DVT
Positive  duplex ultrasonography
o Negative  rule-out DVT
o Positive  treat for DVT
In the likely group (moderate-to-high risk of DVT)  duplex ultrasonography
Negative  repeat clinical evaluation and ultrasonography in 1 week
Positive  treat for DVT
When discordance exists between the pretest probability and the duplex ultrasonographic study
result, further evaluation is required.
Other non-invasive tests include MRI.
Venography is the gold standard, but is expensive, invasive and higher risk

OSCE-guide-III.doc

Page 122 of 255

Physical Examination

Initial treatment of DVT:


Unfractionated heparin (UFH): requires bolus (7500-10,000 ill), followed by
continuous IV infusion (1000-1500 IU/h). Advantages: rapidly reversible by
protamine. Disadvantages: must monitor aPTT
Low molecular weight heparin (LMWH): administered SC. Predictable dose response
& fixed schedule
Long-term treatment:
Warfarin: standard treatment;
o Should be initiated with heparin overlap dual therapy for at least 5 days.
Discontinue heparin after INR >2.0 for two consecutive days.
o Warfarin should be dosed to maintain INR at 2-3. Monitor INR twice weekly for
1-2 weeks, then weekly until INR stable, then every 2-4 weeks
o Duration of anticoagulant treatment (with warfarin unless otherwise noted):
First episode DVT with transient risk factor: 3 months
First episode DVT with ongoing risk factor (e.g. cancer, antiphospholipid
antibody) or > 1 risk factor: consider indefinite therapy
Recurrent DVT (2 or more episodes): indefinite therapy
IVC filters: useful in those with contraindications to anticoagulant therapy, recurrent
thromboembolism despite adequate anticoagulation, recurrent embolism with
pulmonary HTN, or those who require emergent surgery without time to initiate
anticoagulation
Special considerations:
o Pregnancy: treat with LMWH during pregnancy, then warfarin for 4-6 weeks
post-partum
o Surgery: avoid elective surgery in first month after venous or arterial
thromboembolic event
Preoperatively: IV heparin may be used up to 6 hrs pre-operatively;
warfarin should be discontinued for at least 4 days pre-op. Surgery safe
when INR <1.5
Postoperatively: IV heparin or LMWH can be started 12 hours after
major surgery
Wells Prediction Rule for Diagnosing Pulmonary Embolism
For Predicting Pretest Probability of PE (N.B. the guideline notes that the Wells rule performs
better in younger patients without comorbidities or a history of venous thromboembolism)
Clinical Characteristic
Score
Previous pulmonary embolism or deep vein thrombosis
+ 1.5
Heart rate >100 beats per minute
+ 1.5
Recent surgery or immobilization (within the last 30 d)
+ 1.5
Clinical signs of deep vein thrombosis
+3
Alternative diagnosis less likely than pulmonary embolism + 3
Hemoptysis
+1
Cancer (treated within the last 6 mo)
+1
Score
0-1
2-6
7

Clinical Probability of PE
Low
Intermediate
High

OSCE-guide-III.doc

score > 4
score 4

Simplified Wells
PE likely
PE unlikely

Page 123 of 255

Physical Examination

Evaluation of suspected PE:


Low clinical probability of PE  D-dimer:
o Negative: ruled out
o Positive  CT scan with contrast:
Negative: ruled out
Positive: ruled in
Intermediate OR high probability:
o CT scan:
Negative: ruled out
Positive: ruled in

Notes:
1. Use D-dimers only if low clinical
probability, otherwise, go straight to spiral
CT or V/Q
2. If using V/Q scan (CT contrast allergy or
renal failure):
Negative V/Q scan rules out the diagnosis
Inconclusive V/Q scan requires leg US
duplex to look for DVT (q2d)
High probability V/Q scan only rules in the
diagnosis if have high clinical suspicion

CXR of PE: may be normal / wedge-shaped infiltrate / unilateral effusion / raised hemidiaphragm
Treatment of PE:
Admit for observation (patients with DVT only are often sent home on LMWH)
Oxygen: provide supplemental O2 if hypoxemic or short of breath
Pain relief: analgesics if chest pain narcotics or NSAIDs
Acute anticoagulation: therapeutic-dose SC LMWH or IV heparin start ASAP
o Anticoagulation stops clot propagation, prevents new clots and allows
endogenous fibrinolytic system to dissolve existing thromboemboli over months
o Get baseline CBC, INR, aPTT renal function liver function
o For SC LMWH: dalteparin 200 U/kg once daily or enoxaparin 1 mg/kg bid no
lab monitoring avoid or reduce dose in renal dysfunction
o For IV heparin: bolus of 75 U/kg (usually 5,000 U) followed by infusion starting
at 20 U/kg/hr aim for aPTT 2-3 times control
Long term anticoagulation:
o Warfarin start the same day as LMWH/heparin start at 5 mg PO od overlap
warfarin with LMWH/heparin for at least 5 days and until the INR is in target
range of 2-3
o LMWH instead of warfarin for pregnancy; active cancer, high bleeding risk
o Duration of long-term anticoagulation treatment:
If reversible cause for PE (surgery, injury, pregnancy, etc.): 3-6 months
If PE unprovoked OR ongoing major risk factor (active cancer):
indefinite
IV thrombolytic therapy:
o If patient has massive PE (hypotension or clinical right heart failure)
o Hastens resolution of PE but may not improve survival or long-term outcome
Interventional thrombolytic therapy (massive PE is preferentially treated with
catheter directed thrombolysis by an interventional radiologist, works better than IV
thrombolytic therapy and fewer contraindications)
IVC filter: only if recent proximal DVT + absolute contraindication to
anticoagulation

OSCE-guide-III.doc

Page 124 of 255

Physical Examination

Peripheral Arterial Disease Examination


Introduction
Vitals
General
Inspection

Palpation

Abdomen

Neurology

Burger test

Special tests

Based on the vitals, the patient is stable, I would like to proceed


Drape the patient / expose the lower limbs (triangular)

I would like to take a look at your feet, can you please remove the socks; do
you want me to help you?!

SEADS

No signs of arterial insufficiency: no hair loss / no shiny tight skin / no
hypertrophic nails

Temperature

Capillary refill (< 3seconds)

Pulses: dorsalis pedis / posterior tibial / popliteal / (to examiner) I would like
to check the femoral arteries

Drape the patient / I would like to examine your abdomen / can you please
uncover your abdomen

Listen for bruits (aortic / renal / iliac)

Check for light touch, here is a piece of cotton, this is how it feels; can you
please close your eyes! Tell me when you feel it touching you! Check both
lower limbs from distal to proximal

If light touch is ok, do not proceed with more tests
I would like to raise your legs, for 1-2 minutes, if you feel any pain / numbness /
tingling please let me know, check the color of the foot. Then dangle the feet and
check the color  no pallor on elevation, no rubor on dependence  Burger test
is negative.
I would like to arrange for ankle / brachial index


Investigations:

CBC

Fasting blood sugar / lipid profile

ECG

Angiography (side effects: nephrotoxic / allergy / aneurysm risk)

Doppler U/S study of the arterial tree both lower limbs


Treatment:

Life style modifications (refer to HTN)

Foot care

Graded exercise

Surgery (if severe disability)

OSCE-guide-III.doc

Page 125 of 255

Physical Examination

Diabetic Foot
Diabetic patient with long hx of diabetes, has an ulcer for few days
Introduction
Vitals
General
Inspection

Based on the vitals, the patient is stable, I would like to proceed










Palpation





Abdomen




Neurology








Burger test
Special tests
Balance (if
DM focused
exam case)

OSCE-guide-III.doc

Drape the patient / expose the lower limbs (triangular)


I would like to take a look at your feet, can you please remove the socks; do
you want me to help you?!
Describe the ulcer: location (in the sole at base of 1st metatarsal), shape
(round, irregular), size ( cm), margins not elevated, no active
bleeding or oozing
No other ulcers in the same foot / check the other foot / check in
between toes  no evidence of infection in between toes / in nails
No pigmentation around the medial and lateral malleoli
SEADS quadriceps wasting / swollen joints
No signs of arterial insufficiency: no hair loss / no shiny tight skin / no
hypertrophic nails
Temperature
Capillary refill (< 3seconds)
Pulses: dorsalis pedis / posterior tibial / popliteal / (to examiner) I would
like to check the femoral arteries
Drape the patient / I would like to examine your abdomen / can you please
uncover your abdomen
Listen for bruits (aortic / renal / iliac)
Check for light touch, here is a piece of cotton, this is how it feels; can you
please close your eyes! Tell me when you feel it touching you! Check both
lower limbs from distal to proximal
If light touch is ok, do not proceed with more tests
Light touch sensation is absent distal to the level of cm above ankle
Proprioception: I will move your toe, close your eyes please, tell me is
it up or down. Then move to the next joint. Start with head of
metatarsal, medial malleolus, tibial tuberosity,
Vibration: tuning fork, here is the sensation you will feel, tell me when
it stops  intact / decreased / absent
Monofilament test: to distinguish between the light touch and pressure
sensation / 10 points on the foot (9 on the sole, and one on the dorsum
above the big toe meta-tarso-phalangeal joint)
Ankle reflex; if you have time: knee reflex and Babinski

I would like to arrange for ankle / brachial index





Gait ataxia
Romberg test: positive (with open eyes: pt can balance himself by vision;
while with closed eyes: pt loses balance

Page 126 of 255

Physical Examination

Neurological Examination
o
o
o
-

Introduction
Vital signs
General inspection of the patient: pt is sitting comfortably

Orientation: what is your name sir? Where are you? Time? Place?
Cranial nerves
Upper and lower extremities:
o Inspection
o Palpation / bulk
o Tone:
 Just relax please, let me do everything for you. I am going to check the
tone in your Rt arm
 Tone is normal, no hypo or hyper tonia
o Motor power (5  0)
 5
full power
 4
less than full power (like Lt hand in Rt handed person)
 3
can do the movement against gravity
 2
can do the movement with the gravity eliminated
 1
muscle twitches, not able to initiate movements
 0
no power no movements
o Sensory:
 Light touch:
Pin prick or piece of cotton
First check on forearm or sternum
Can you close your eyes please
Distal to proximal
Bilateral  sensation is equal bilaterally
 Posterior column (B12 deficiency / alcohol / syphilis):
Vibration sense: tuning fork / test on sternum / tell me when it
stops / start distal / if intact move on / if not intact go proximal
on the next joint
Proprioception: eyes closed / start with the big toe or thumb / is
it moving or not? / is it up or down?
o Reflexes:
 0
absent
 1
weak (hyporeflexia)
 2
normal
 3
hyper reflexia
 N.B. Babinski reflex: I am going to tickle the bottom of your foot:
Planter flexion: normal response
Big toe dorsiflexion and toes fanning: UMNL (e.g. stroke)

OSCE-guide-III.doc

Page 127 of 255

Physical Examination

Gait ATAXIA
o Can you take few steps for me please?
o Protect the patient, surround him with your arms, and walk with him
Romberg test
o Can you put your legs together!
o Can you close your eyes please!
o Watch (protectively) for few seconds!
 Ataxia due to peripheral neuropathy (B12 deficiency / DM / syphilis):
with eyes closed
 Cerebellar ataxia: no with closed eyes (always on)
Cerebellar signs (stroke / alcohol / tumours / para-neoplastic / ):
o Nystagmus:
 Can you follow my finger please (move it side to side)
Physiological: transiently then corrected
Central: horizontal or vertical
Peripheral: horizontal only. Conditions: benign positional vertigo
/ acute labyrinthitis / drugs
o Finger to finger:
 Patient hand must be extended
 Move the examiner hand
 Check both upper limbs
o Finger to nose test: lesion in the cerebellum on the same side.
 Intentional tremors
 Loss of coordination
o Heel to shin: lesion in the cerebellum on the same side
Cortical sensations: two points discrimination
Mini-mental status exam

OSCE-guide-III.doc

Page 128 of 255

Physical Examination

Cranial Nerves Examination


-

Vital signs
Comment on the patient general condition

CN I:
- Do you have problems with smells?
Can you please close your eyes?
- What is that? Coffee / ammonia
- What is that? Ammonia / coffee
CN II:
The optic nerve:
- Visual acuity: Do you wear glasses? reading / color (Snellen chart at 1 foot distance 35
cm)
- Visual fields: eye by eye / by confrontation (when you see my fingers wiggling)
- Pupillary reflex: I am going to shine light in your eyes, please look straight to the wall,
each eye: direct and consensual (afferent: CN II, efferent: CN III)
- I would like to do fundoscopy examinations, looking for: disc edema, retinal hemorrhage,
neovascularisation, nipping of the veins
CN III, IV, VI:
- By inspection; both eyes are symmetrical, no deviation, no nystagmus, no head tilting, no
ptosis (CN III: opens, CN VII: closes)
- I would like you to follow this pen, without moving your head please, and if you see
things double or blurred at any time, please let me know  move the pen in large Hshaped manner, then conversion
- Normal extra-ocular muscles movements, no nystagmus or double vision
CN V:
- Motor:
o By inspection: no atrophy of the temporal or masseter area
o Can you please clench, feel the temporalis and masseter
o Can you open your mouth against my hand?
- Sensory:
o This is a piece of cotton, and this is how it feels, I am going to touch your face,
and whenever you feel it, please tell me. Can you close your eyes please?
o Touch the face in symmetrical areas; cover the ophthalmic, maxillary, and
mandibular areas. Does it feel the same?
o Facial sensation of the trigeminal nerve is intact and equal on both sides
- Reflexes:
o Corneal reflex (afferent: CN V, efferent: CN VII)

OSCE-guide-III.doc

Page 129 of 255

Physical Examination

CN VII: (mainly motor)


- Motor:
o Face is symmetrical, no deviation of the angle of the mouth, normal naso-labial
folds, no drooling
o Can you please copy me:
 Raise your eye brows, can you frown (wrinkle your forehead) please?
 Can you close your eyes, and do not let me open them?
 Can you blow your cheeks? Whistle?
 Can you show me your teeth?
o If there is a mouth angle deviation, it is deviated to opposite side of lesion
o UMNL vs. LMNL:
 UMNL: intact upper face muscles (the nucleus receives double sided
innervation)
 LMNL (Bells palsy)  ptosis; all face muscles are affected
- Sensory:
o To complete the facial nerve examination, I would like to check the sensations in
the outer 2/3 of the tongue
o I would like to check the ext auditory canal and the tympanic membrane to rule
out herpes zoster of the facial nerve
- Reflexes:
o Jaw reflex
o Corneal reflex (afferent: CN V, efferent: CN VII)
CN VIII
- Check by whispering (ABC CBA), while rubbing fingers in front of the other ear OR
by rubbing your fingers
- Because the hearing is normal, I am going to skip Weber and Rinne tests
o Rinne: place the tuning fork in front of ear, then on the mastoid process
o Weber: place the tuning fork on the forehead
CN IX, X:
- Patient voice is normal, no hoarseness
- Can you swallow a sip of water please? Normal swallowing
- Can you open your mouth please? Soft palate is symmetrical, uvula is central
o Uvula deviates to the opposite side of the lesion
- To check the reflexes: I need to do the gag reflex
CN XI:
- Can you please shrug your shoulders?
- Turn your head to the right, and to the left. I am going to resist you. I feel for the opposite
side sterno-mastoid
CN XII:
- Can you please open your mouth? Can you stick your tongue out?
o Tongue is central, no deviation. No fasciculations or atrophy of tongue.
o If there is a lesion, the tongue deviates towards the lesion side
- Can you please move it to the right and to the left? Can you stick it against your cheeks?
 Normal movements of the tongue

OSCE-guide-III.doc

Page 130 of 255

Physical Examination

Tremors
? Parkinson disease
Introduction
Vital signs
General comment
Inspection

Shaky right hand


I would like to rule out any orthostatic hypotension
Pt is sitting comfortably , with Rt hand tremors

Tremors

Right hand tremors, not obvious on the left hand


Count from 10 to 1 please  tremors with mental activity 
consistent with Parkinson disease, and rules out anxiety

Stretch your hand plz / no fine tremors  r/o hyperthyroidism

No flapping tremors  rule out liver failure

Finger to nose / no intentional tremors  r/o cerebellar dis
Patient tremors consistent with Parkinson disease, resting tremors,
beads rolling, and limited to Rt hand (). No head nodding
(No) limited facial expression, decreased eye blinking, drooling
Face
Palpation
Check the wrist and elbows:
Rigidity

(No) cog wheeling
Positive with parkinsonism

(No) lead pipe rigidity

(No) clasp knife spasticity
Positive with stroke
Standing / walking

Would you please stand up! Do you need help  patient finds
Postural instability
difficulty in standing up

Can you walk few steps for me please: comment with + or - Stooped posture
- Shuffling (festinating) gait
- Decreased arm swinging
- Patient turns in blocks
Special Tests

Rapid alternating movements (hand supination & pronation /
oppose thumb to fingers)  dysdiadochokinesia

Can you please repeat British constitution  monotonous

Can you write a sentence for me  micrographia

Can you draw a spiral parallel to this (draw spiral on paper)
I would like to do the mini-mental status exam


Treatment of Parkinsonism:
 Pharmacologic

Mainstay of treatment: Sinemet (levodopa / carbidopa). Levodopa is a dopamine


precursor, carbidopa decreases peripheral conversion to dopamine
o Levodopa related fluctuation: delayed onset of response (affected by mealtime),
end-of-dose deterioration (i.e. wearing-off), random oscillations of on-off
symptoms
o Major complication of levodopa therapy is dyskinesias

Treatment of early PD: DA agonists, amantadine, MAOI

Adjuncts: DA agonists, MAOI, anticholinergics (especially if prominent tremors),


COMT inhibitors
 Surgical: thalamotomy, pallidotomy, deep brain stimulation (thalamic, pallidal, subthalamic),
embryonic dopaminergic stem cell transplantation

OSCE-guide-III.doc

Page 131 of 255

Physical Examination

Thyroid Exam
Introduction
Vital signs
General

Thyroid Exam
Inspection
Palpation

Percussion
Auscultation

11
12

BP, HR
Can you stretch your hands:
- Fine tremors
- Palms for sweating
- Nail changes
- Hair loss (hypothyroidism)

Examine the eyes:
- Exophthalmos stand by the patient (stand behind the right shoulder
and look from above)
- Lid lag (can you follow my finger without moving your head from
above downwards)

Proximal muscle weakness:
- Can you shrug your shoulders (bilaterally against my hand) please

Knee reflex: brisk11 reflex

Peritibial myxedema: indicates hyper-thyroidism
Patient is sitting on a chair
Can you swallow12 please?  no apparent thyroid enlargement

Thyroid gland:
- From behind the patient, bi-manually
- Then while swallowing a sip of water  thyroid movement is normal,
I do not feel any masses, nodules, and no tenderness

Lymph nodes:
- Sub-mandibular and cervical

DIRECT percussion on upper part of sternum

Checking for retro-sternal extension (no retro-sternal dullness)

BOTH lobes

For thyroid bruits


Reflexes grades:
0 absent
1 hypo
2 normal
3 hyper (brisk)
4 hyper with clonus (ankle)
Whenever you ask the patient to swallow, give a sip of water, it is difficult to swallow on an empty mouth

OSCE-guide-III.doc

Page 132 of 255

Physical Examination

Dermatomes

OSCE-guide-III.doc

Page 133 of 255

Physical Examination

Neck Examination
Patient complaining of pain in the neck
Vitals
General
Inspection

Palpation










ROM






Powers

Special tests




Can I get the vital signs please


Comment on patient general condition: patient is sitting comfortably
I would like to take a look at your neck; can you lower your gown please?
From the back: SEADS (Swelling / Erythema / Atrophy / Deformity / Scars)
From the side: normal cervical and thoracic curvatures
Can you cough please, any pain
- No neck pain with Valsalva manoeuvre
I would like to feel you back please
- Temperature is normal
- Tenderness: pressing on the spinal processes and para-vertebral muscles,
trapezius and sternomastoid
Thyroid: can you swallow for me please?
Check the LNs: no enlarged LNs
Can you touch your chest by your chin?
Can you look to ceiling?
Can you turn your head to the right? And to the left?
Can you tilt your head? To the right and to the left?
- Normal flexion, extension, rotation and lateral flexion
Test again against resistance
- Neck pain is not associated with muscle contractures
Spurling test: stretching the nerve  reproduces the pain
Upper limb examination and neurological screening

Neurological
screen

Part of my exam is to check your upper extremities, can you roll up your sleeves please!
Inspection
Upper extremities are symmetrical, normal bulk, no atrophy / SEADS
Palpation
I am going to feel your shoulder; deltoid, biceps, triceps, forearm, thenar,
hypothenar  are symmetrical / no deformity / no atrophy
Motor
Power
Deltoid C5
Biceps C5/6
Triceps C7/8
Sensory
C4: deltoid
C5: biceps lateral aspect
Test light
C6: thumb
C7: middle finger
C8:
touch
little finger
T1: elbow medial aspect
Reflexes  Biceps and brachio-radialis C5/6

Triceps C7/8
Radial pulse
Pulse
Post encounter: what is the level of the lesion?
C6 nerve root lesion (C5-C6): weak biceps, weak biceps reflex
C7 nerve root lesion (C6-C7): weak triceps, weak triceps reflex
Diagnosis: Osteoarthritis of the cervical spine at level
X-ray findings: osteophytes of the cervical vertebrae / narrowing of disc space / subchondral
sclerosis (increased bone formation around the joint), subchondral cyst formation
Management:

Soft neck collar

NSAIDs / acetaminophen

Physiotherapy

If worsening: neurosurgery consult / CT myelography / nerve conduction studies

OSCE-guide-III.doc

Page 134 of 255

Physical Examination

Carpal Tunnel Syndrome


Hand pain for 6 months
History
Analysis
CC  Os Cf D / PQRST /

Radiation: how about your elbow? Your fingers?

Triggers: is it related to time? To work? What do you do?
AS  Weakness, numbness, tingling

Swelling, redness, warm

How about the other hand?
Impact

How did this affect your life? Your work?
Red flags

Constitutional symptoms
Differential

Hx of neck pain / injury / trauma
diagnosis

Hx of diabetes / symptoms

Hx of thyroid disease / symptoms of hypothyroidism

Hx of autoimmune disease / Rh arthritis / symptoms

Headache / visual disturbances / shoes are getting tight

Pregnancy / LMP
PMH / FH
Physical examination
Vital signs
Inspection

SEADS (thenar / hypothenar ms)

No nail changes, no nodules / no deformity
Palpation

Temperature: is normal

Tenderness: palpate distal radial bone, styloid process, joint line, styloid
process, distal ulnar bone, base of the thumb, carpal bones, metacarpal
bones, digits
ROM

Flexion / Extension /+/ move your hand to the right, to the left

Can you make a fist / fan your fingers  there is no obvious damage to
the nerves / muscles / and tendons of the hand

Thumb movements:
- Touch base of your little finger (thumb opposition)
- Move it all the way to opposite direction
- Point to the ceiling (with hand supine, flat)
- Touch the tips of your fingers
Power

Like ROM but against resistance

Thumb 90, DIP flexed: do not let me straighten it

Biceps ROM / against resistance / biceps reflex (C6)
Sensory

Check with cotton tip,

For the ring finger: check both sides: ulnar / radial
Special tests

Phalen's test,

Tinel's sign / tap on the carpal tunnel
Investigations: EMG / nerve conduction studies
Treatment:

Modify nature of work

NSAIDs

Wrist splint

Local corticosteroids injection

Surgical decompression

OSCE-guide-III.doc

Page 135 of 255

Physical Examination

Hand Laceration / Wrist Laceration


Physical examination
Introduction
Gloves / vital signs
Inspection

If big wrist laceration (BIG band aid): DO NOT remove the band aid,
should be removed under anesthesia

If small laceration, remove the band aid  describe the laceration: linear /
size / position / elevated margins / active bleeding / oozing

Other ulcers / other hand

SEADS (Swelling / Erythema / Atrophy / Deformity / Scars)

No nodules / no deformity
Palpation

Temperature: is normal, is hand pink and warm?

Tenderness: palpate distal radial bone, styloid process, joint line, styloid
process, distal ulnar bone, base of the thumb, carpal bones, metacarpal
bones, digits

Capillary refill

Feel the pulse  radial ulnar

Allens test
ROM

Do NOT ask patient to apply force against resistance as this may rupture
a partially severed tendon. Test active ROM only

Median nerve: thumb abduction (thumb to ceiling)

Ulnar nerve: finger spread (fanning)

Radial nerve: wrist extension

Tendon examination of the hand (flexor tendons):
- DIPs; flexor digitorum profundus: hold the MCP / PIP joints in
extension, and ask the pt to flex DIPs.
- PIPs; flexor digitorum superficialis: hold all fingers except one you
are testing in extension and ask pt to flex the remaining finger
- MCP joints: intrinsic (lumbricals)
Power

Do NOT ask patient to apply force against resistance as this may rupture
a partially severed tendon. Test active ROM only
Sensory

Check with cotton tip or pin-prick, for light touch

Check two point discrimination on either side of each digit

For the distribution of the three nerves: ulnar / radial / median
Neuro-motor examination of the hand
Median
Sensory Radial aspect of index finger
pad
Motor
Flex DIP if index finger (FDP)
extrinsic
Motor
Thumb to ceiling with palm up
intrinsic (abductor pollicis brevis)

Ulnar
Radial
Ulnar aspect of little finger pad Dorsal web space of thumb
Flex DIP of little finger (FDP), Extend wrist and thumb,
extensor carpi radialis
(extensor pollicis longus)
Abduct index finger (first
dorsal interosseous)

Structures lacerated
Diminished ulnar territory sensation
Allen test shows (no) refill from the ulnar circulation
FDS weakness in little finger and ring finger

Ulnar nerve
Ulnar artery
Flexor retinaculum, ulnar two divisions of FDS

Management: clean and explore wound under local anesthesia and sterile conditions. Consult plastic
surgery for micro-vascular repair. If at night, may suture the skin and arrange for pt to be seen by plastic
surgeon next day.

OSCE-guide-III.doc

Page 136 of 255

Physical Examination

Back Pain
Acute




Acute on top
of chronic
Chronic

Herniated disc
Muscle spasm

Pain at end of day shoots to legs, increases when leaning


forward:

Disc herniation

Degenerative disease (OA)
Pain related to position, increases when extension (or
leaning backwards), walking down the hill (arching the
back)  spinal canal stenosis
Pain wakes patient in morning, improves at end of day,
stiff  Ankylosing spondylitis  limitation in all
directions

Neurological exam

Mechanical exam

Red flags for back pain:


BACK PAIN

B
bladder or bowel dysfunction

A
anaesthesia (saddle)

C
constitutional symptoms  malignancy

K
chronic disease





P
A
I
N

OSCE-guide-III.doc

parathesia
age > 50 years old
IV drug user
neuro-motor deficits

Page 137 of 255

Physical Examination

Acute Back Pain


Acute back pain: pain for 3 days
Acute on top of chronic: pain for 3 hours old patient
Introduction

Analysis

Impact
Red flags

DD
PMH
FH
SH

OSCE-guide-III.doc

How do you feel?


Do you prefer to remain standing, sit, or lie down?
That is ok, whenever you want to lie down or rest, feel free to do so
Os Cf D

What were you doing? What do you do for work?

Did you hear any crack sound?

Were you able to stand up and continue? Were you able to
move? Did anyone help you?

Is this the first time?

How about at rest? During night?

Recently did you have any discomfort? Milder back pain?
How did it affect you?
PQRST

Does it shoot to your thighs? Toes?

Which is bothering you more; your back or your legs?


How about lying down? Stretch your back? Coughing?
Moving? Leaning forward or backward?

Any medication? Did it help?

How does it affect you?

Mets to lungs / liver / brain

Constitutional symptoms

Overweight
In addition to your pain, did you notice any other symptoms:

Weakness, numbness, tingling

Difficulty with balance, falls

Any difficulty passing urine? How about bowel
Cauda equine
movement? Did you find that you soiled yourself?

Any numbness in the buttocks area?

Do you have morning erection? Any sexual dysfunction?
Cancer prostate / bladder
Cancer prostate

Page 138 of 255

Physical Examination

Chronic Back Pain


Pain for 3 months
Introduction
Analysis

Do you prefer to remain standing, sit, or lie down?


That is ok, whenever you want to lie down or rest, feel free to do so
Os Cf D

What were you doing? What do you do for work?
PQRST

Impact
Red flags

DD
PMH
FH

Quality: stiffness
Timing: is it worse in morning? Improves with time? Or is it worse
at the end of the day?

Does it shoot to your thighs? Toes?

Which is bothering you more; your back or your legs?


How about lying down? Stretch your back? Coughing? Moving?
Leaning forward or backward?

Any medication? Did it help?
AS

Pain other joints (knees / hips / hands) / distribution? Osteoarthritis
Sero
Pain other joints (knees / hips / hands) / distribution? Ankylosing
negative

Eye pain, redness / mouth ulcers
spondylitis
symptoms

Skin changes / nail changes / hx of psoriasis

Repeated attacks of abd pain / diarrhea

Urethral discharge

How does it affect you?

Mets to lungs / liver / brain

Constitutional symptoms

Overweight
In addition to your pain, did you notice any other symptoms:

Weakness, numbness, tingling

Difficulty with balance, falls

Any difficulty passing urine? How about bowel movement? Did you Cauda equine
find that you soiled yourself?

Any numbness in the buttocks area?

Do you have morning erection? Any sexual dysfunction?









Trauma
Injury to back
Osteoarthritis
Ankylosing spondylitis
Other rheumatic disease

SH
Osteoarthritis: older patient / worse at evening
Ankylosing spondylitis:

Morning stiffness improves by time

LSS x-ray: sacroiliitis OR fusion of SI joints

ESR:

HLA-B27 tissue antigen: positive


Associated symptoms: inflammatory arthritis / Uveitis / psoriasis / IBD / pericarditis / aortic regurgitation
Management:

No cure

Regular therapeutic exercises to prevent deformity (swimming / back extension exercises)

NSAIDs: Indomethacin (50 mg PO bid) or Naproxen (250 mg PO bid)

In severe cases: total joint replacement

OSCE-guide-III.doc

Page 139 of 255

Physical Examination

Back Joint Examination

Neurological screen

Physical examination
Introduction
Can you stand up please?
Vital signs
Inspection

Gait / balance / stance

Ask the patient to stand up from sitting position

Posture: normal cervical, thoracic, lumbo-sacral curvatures

Adams forward bend test (if scoliosis: the scapula will be higher)
- No scoliosis or kyphosis

SEADS
Palpation

Temperature

Tenderness: spinal processes, para-vertebral muscles, sacro-iliac joints
(medial to dimples of Venus)
ROM

Can you touch your toes with your fingers? Without bending knees

Can you arch your back? Without bending knees (stand supported by the
bed foot: will not fall, less possibility of knee bending)

Slide your arms on both sides (Rt and Lt)? (stand against wall, normally
the tips of finger travel > 10 cm)

Cross your arms? Turn to the Rt and Lt (pt sitting on bed)

Modified Schober's test: (midline, between the dimples of Venus) + 5
cm below + 10 cm above  bend forward  N> 6 cm diff.
Special tests

Occiput-to-wall distance (tragus & nose same level): normally zero

Straight leg raise (irritation of the roots of sciatic n: L4/L5/S1/S2):
elevate the lower extremity straight, when it is painful  where it does
hurt?  straight leg test positive

Decrease the angle, try to dorsiflex foot  Lasgue sign

Cross straight leg raise test: elevate the other LL  trigger pain

Fabers test (figure 4 test): to check sacro-iliac joint pathology

Femoral nerve stretch (done for patients c/o pain radiating to the anterior
aspect of the thigh): patient prone, knee flexed,
Motor

Hip flexion (L1/L2/L3) / extension (S1/S2)

Knee flexion (L5/S1/S2) / extension (L2/L3/L4)

Ankle dorsiflexion (L4/L5) / plantar flexion (S1/S2)

Can you walk on your heals?
Normal L4/L5 muscles

Can you walk on your toes? Normal S1/S2 muscles
Sensory
S1: little toe
L5: first web
L4: medial malleolus
L3: knee med
L2: thigh ant
L1: groin
T10: umbilicus
Reflexes
Knee (L2/L3/L4 mainly L4) / Ankle (S1/S2 mainly S1) / I would like to do
the Babinski reflex (positive in UMNL)
Pulse
Dorsalis pedis
Other clinical examinations: DRE; to rule-out cauda equina (sphincter weaknesses, reduced anal
tone)
N.B. dimples of Venus correspond to PSIS

OSCE-guide-III.doc

Page 140 of 255

Physical Examination

Ankle Twist
Young man comes with ankle twist; history and physical examination are normal, no fractures,
and no lacerations. In the next 10 minutes counsel him about the treatment
History

Mechanism of trauma / injury


Noise heard at the time of trauma

Where you able to walk after the injury?
Symmetry / SEADS

Temperature

Tenderness: med malleolus / lat malleolus / ant joint line (between malleoli)
/ lat ligaments (slightly ant and inferior to the lat malleolus) / med ligaments
(inferior and slightly ant to the med malleolus) / tendon Achilles / navicular
head (medial bony prominence) / fifth meta-tarsal head (lat) / around the
meta-tarsal heads / origin of the plantar fascia (inf med aspect of calcaneus )

Crepitus
Patient is sitting on the side of the bed

Dorsi-flexion (point toes up) and plantar flexion (point toes down)

Inversion (point the bottoms of your feet towards each other) and eversion
(the opposite direction)
ROM against resistance / after stabilizing the ankle with your other hand

Talar drawer sign: 1 cm is significant, indicates ant talo-fibular lig rupture

Talar tilt: lat calcaneo-fibular lig rupture (inversion), med lig rupture
(eversion)

Squeeze test (squeezing calf  ankle pain); if negative  NO ankle
fracture



Inspection
Palpation

ROM

Power
Special tests

Investigations: x-ray

Ottawa ankle rules; for ankle series:
o Pain in the malleolar zone and any one of the following:
 An inability to bear weight both immediately and in the emergency
department for four steps
 Bone tenderness along the tip of the medial or lateral malleolus

Ottawa foot rules; for foot series
o If there is any pain in the mid-foot zone and any one of the following:
 An inability to bear weight both immediately and in the emergency
department for four steps
 Bone tenderness at the base of the fifth metatarsal
 Bone tenderness at the navicular bone
Management:

Complete tear should be evaluated by orthopedics  stat orthopedics consult

RICE: rest (and crutches) / ice for 20 min QID x 3 days / compression (by tensor bandage) /
elevation

Pain medication: NSAIDs; e.g. Ibuprofen 400 mg, PO, q6h.

Show him how to wrap it, remove the wrap, and ask him to wrap it again (to make sure he
knows how to). Remember: from distal to proximal and 1/3 width overlap.

Show him how to use the crutches.

OSCE-guide-III.doc

Page 141 of 255

Physical Examination

Shoulder Joint
History
- Trauma to shoulder / neck? X-ray done? What is your occupation?
- Neurological deficits? How does it affect your life?
Vital signs
General
Patient condition (restlessness, discomfort, willingness to move)
Inspection
- Both shoulders symmetrical / clavicle level / scapula level / deltoid
- SEADS (Swelling / Erythema / Atrophy / Deformity / Scars)
Palpation
- Temperature: compare
- Tenderness: sternal notch / sterno-clavicular joint / clavicle / acromioclavicular joint / deltoid / long head of the biceps / insertion of the rotator
cuff muscles / spine of the scapula / medial border of scapula / spinal
processes of the cervical spine
- Crepitus
ROM
- Active ROM: can you copy me please:
- Abduction  and comment on painful arc test
- Adduction  and comment on drop arm test
- Forward flexion (180) /+/ Backward extension (60)
- External rotation /+/ Internal rotation
- Another faster way to check:
- Hands behind your neck (abduction / ext rotation)
- Hands behind back (adduction / int rotation) between shoulder blades;
touch the tip of the contra-lateral scapula.
- Passive ROM: If patient is unable to complete the whole range of
movements actively, complete the ROM passively and comment (in
inflammation: passive ROM is > active ROM)
Power
- Like the ROM, but against resistance
Special tests
1 Painful arc (between 60 and 120)
All these tests are done to
2 Drop arm test  complete tear of supratest for subacromial
spinatous tendon
impingement of supra3 Neers test
spinatous
4 Hawkins test
5 Jobes test (empty can test)
6 Lift-off test: try to push my hand away from
Sub-scapularis
your back
Yergasons test; palm face up test: shake
For bicepital tendinitis
hands, try to let your palm face upwards, I
will resist you, and press on your shoulder
8 Speeds test: supine, semi-flexed, do not let
me push your arm down
9 Stability testing:
For joint stability
+ Push ant / post
+ Pull down  sulcus sign
10 Apprehension test (ant and post): for
dislocation
To complete my exam, I would like to do:
- Check the pulses of the upper limb (radial / ulnar / brachial)
- Brief neurological examination of the upper limb
- One joint above and one joint below examination (cervical spine / elbow)
- The other shoulder examination
7

OSCE-guide-III.doc

Page 142 of 255

Physical Examination

Rotator cuff muscles:


Supra-spinatous
Infra-spinatous
Teres minor
Sub-scapularis

Abduction
External rotation
Internal rotation

Lift-off test

Impingement syndrome:
- The most common symptoms in impingement syndrome are pain, weakness and a loss of
movement at the affected shoulder
Treatment:
- Mild: RICE / NSAIDs / PT. Rest (cessation of painful activity), ice packs and NSAIDs
may be used for pain relief. Physiotherapy (PT) focused at maintaining range of
movement and avoiding shoulder stiffness.
- Moderate: therapeutic injections of corticosteroid and local anesthetic may be used for
persistent impingement syndrome
- Severe: surgery
Investigations:
- U/S
Possibilities:
- Normal shoulder exam
- Frozen shoulder
-

Bicepital tendinitis
Repeated ant dislocation
Rotator cuff tear
o Complete tear
o Partial tear
Rotator cuff tendinitis
Sub-deltoid bursitis

stiff, with limited active and passive ROM (ttt:


physiotherapy, NSAIDs, steroids)
+ve palm face up test / speed test
positive apprehension test
drop arm test
(ttt: surgery)
pain with initiation of movement / +ve empty can test (ttt:
physiotherapy, NSAIDs, steroids, surgery)
similar to partial tear / +ve impingement test

Elbow
ROM: flexion / extension /+/ pronation / supination
Tennis elbow (lateral epicondylitis)

With the elbow fully extended, there are points of tenderness over the lateral epicondyle
(origin of the extensor carpi radialis brevis muscle).

Cozen's test: pain with passive wrist flexion and resistive wrist extension.[

X-rays are used to confirm and distinguish possibilities of existing causes of pain that are not
related to Tennis Elbow, such as fracture or arthritis.
Golfer's elbow (medial epicondylitis)
The common tendinous sheath is inserted into the medial epicondyle of the humerus

Treatment:
o NSAIDs: ibuprofen, naproxen or aspirin /+/ Heat or ice
o A counter-force brace or "elbow strap" to reduce strain at the elbow epicondyle, to
limit pain provocation and to protect against further damage.


OSCE-guide-III.doc

Page 143 of 255

Physical Examination

Hip Joint
+ Middle-age male with septic arthritis
+ Elderly female with osteoarthritis
Vital signs
General

Patient condition (restlessness, discomfort, willingness to move)


May I ask for full exposure please?
Inspection
Hip joint is deeply seated joint, I am looking for the surroundings
SEADS (Swelling / Erythema / Atrophy / Deformity / Scars)
Scoliosis / kyphosis / pelvic tilt (level of both iliac crests)
Gait: no wide stance, shuffling, drop foot, or antalgic gait
Balance: Trendelenberg sign; standing on one leg (while the patient is putting his
arms on the examiner shoulders), the pelvis drops
Palpation
- Temperature: compare
- Tenderness: ASIS, iliac crest, PSIS, sacro-iliac joint, greater trochanter of the
femur.
I would like to check symphysis pubis and inguinal ligament.
- Crepitus: over femoral head (lat to femoral art, below inguinal lig)
ROM
Active ROM, each one followed immediately by passively stressing (increasing) the
ROM while patient is lying
- Forward flexion (120)
- Internal rotation (30) /+/ External rotation (45)
- Create space, stabilize the contra-lateral hip with your left hand: Abduction (45)
/+/ Adduction (30)
- Backward extension (while lying prone): stabilize the lower back by your left hand,
can you lift your thigh (20)
Power
- Resisted isometric testing (patient lying supine)
Special tests
- Figure 4 test (Patrick or Fabers test)  the leg of the examined side flexed and
externally rotated with the ankle resting on the patella of the contra-lateral leg. The
examiner applies counter-pressure at the opposite hemi-pelvis, and applies gentle
downward force on the knee. Post hip pain indicates sacro-iliac joint pathology,
while ant lat hip pain may suggest hip joint pathology
- Thomas test  put your hand under pt LSS, and try to max flex the contra-lateral
knee
- True leg length  from ASIS (anterior superior iliac spine) to medial malleolus
on both sides
- Apparent leg test  from umbilicus to medial malleolus
To complete my exam, I would like to do:
- Check the pulses of the lower limb (dorsalis pedis / posterior tibial / popliteal)
- Brief neurological examination of the lower limb
- One joint above and one joint below examination (LSS / knee)
- The other hip examination
Septic arthritis:
Physical exam: fever / very painful joint / +ve trendlenberg test / restricted movements on all directions
DD: Septic arthritis / Osteoarthritis / Osteomyelitis
One diagnosis: septic arthritis / One diagnostic test: arthrocentesis (joint aspirate)
Management:

IV antibiotics, empiric therapy, (based on age and risk factors; oxacillin [2 g IV q4h for 4 weeks], or
vancomycin [if suspecting MRSA; 20 mg/kg IV q8h, for 8 wks], combined with ceftriaxone for gram
ve, if suspecting Gonococcal: ceftriaxone; IV for 2 wks then oral for 2 wks), adjust pending C&S

For small joints: needle aspiration, serial if necessary until sterile

For major joints such as knee, hip, or shoulder: urgent decompression and surgical drainage

OSCE-guide-III.doc

Page 144 of 255

Physical Examination

Knee Joint
Inspection

Palpation

ROM

Power
Special tests

Vital signs, General


- Gait and stance: normal; no antalgic (painful) gait
- Bilateral joint exposure (quadriceps)
- SEADS (Swelling / Erythema / Atrophy / Deformity / Scars)
- No genu varum (bow legs) and no genu valgum (knock-knee) deformities
- Temperature: compare
- Extended knee: tenderness over patella /+/ Lateral movement of patella /+/
quadriceps muscle / quadriceps tendon / patellar ligament / Tibial tuberosity
/ popliteal fossa and popliteal artery
- Flexed knee: tibial plateau / bilateral joint lines /+/ Collateral ligaments /+/
Femoral condyles /+/ patellar crepitus
- Popliteal fossa /+/ Cuff muscles [slightly flexed knee]
Knee effusion:
- Fluid wave or bulge sign (or milking test): for small amount of effusion;
from below and med to upward and lat. Then immediately sweep hand
down the lateral aspect pushing the fluid back
- Fluid ballottement test: for moderate amount of effusion
- Patellar tap: for large amount of effusion
Patient lying down:
- Flexion (130) and extension (180)
- Internal and external rotations: while knee is flexed 90, point your toes in &
out please
- Patellar movement: medial and lateral
- Patellar compression test: tight your thigh please  rough or painful
movement: patello-femoral syndrome or osteoarthritis
- Flexion and extension, while the knee is flexed 90
- Anterior drawer test  for anterior cruciate ligament tear
- Posterior drawer test  for posterior cruciate ligament tear
- Lachman test: hip / knee semi flexed (30)  ACL tear
- Check for the medial / lateral collateral ligaments (stability of knee); while
flexed at 30  no laxity nor pain
- McMurrays test (for medial and lateral meniscus tears)  feel for
crepitus / patient feels pain
-

For medial: maximally flexed knees, externally rotated foot  extend while
applying varus force (from inside outwards)
For lateral: maximally flexed knee, internally rotated foot  extend while
applying valgus force (from outside inwards)

To complete my exam, I would like to do


-

Painful clicking  cruciate, meniscus


Knee lock  torn meniscus
Instability  cruciate 

DD
Tenderness
Investigations
Treatment

OSCE-guide-III.doc

ACL
- Knee giving way
- Inability to continue activity

MCL
- Can not descend
stairs

OsgoodSchlatter disease
Chondromalacia patellae

Pain on tibial tuberosity ( by kneeling)  Pain on lateral movement of patella
X-ray (AP / LAT / skyline)

Benign self-limited condition

Non-impact activities

Continue activity as tolerated

NSAIDs

NSAIDs

Physiotherapy

Physiotherapy

Surgery for refractory cases

Page 145 of 255

Physical Examination

Obstetrics and Gynecology

OSCE-guide-III.doc

Page 146 of 255

Physical Examination

History taking OB-GYN


Introduction
CC
Analysis of CC

HPI

Os Cf D
COCA Blood

Associated
symptoms
DD
Menstrual
M
Gynecological
G
Obstetric
O
Sexual
S

PMH
FH
SH

OB/GYN cases
History taking:
- Vaginal discharge
- Vaginal bleeding
- Amenorrhea
- Infertility
Counselling:
- OCPs
- HRT
- C-section (wants to have c-section or wants to have vag delivery after c-section)
- Abortion
- 22 years old pregnant  anti-natal counselling
- 39 years old found she is pregnant, counsel her
- 30 yrs old pregnant (36 wks), HTN/+++ ptn in urine  counsel for pre-eclampsia
- PAP smear; 16 years old wants to arrange for a PAP smear
- PAP smear: 38 year old had abnormal PAP smear

OSCE-guide-III.doc

Page 147 of 255

Physical Examination

MGOS history questions:


Menstrual:
-

When was your LMP? First day? Was your LMP similar to the previous ones?
Are they regular or not? How often do you have periods?
How long does it last? How many days?
How about the amount? Is it large / small? How many pads/day? Any blood clots?
Are your periods painful? [not painful  anovulatory (PCOS/infertility)]
Any spotting / bleeding between periods?
When was your first period? Was it regular? For how long it was not regular? Normal to be
irregular for up to 18-24 months.

Gynecological:
-

Do you have history or were diagnosed with any gynecological disease (e.g. polyps)?
Do you have history of pelvic surgery or instrumentation (e.g. D&C)?
Do you use contraception? What method? Since when? When was the last time?

Screening:
-

Have you ever had Pap smear before? When was the last time? Any reason (if long time)? What
was the result?
(>40 yrs) have you had mammogram done before? When? (Is it painful doctor? Could be; we
need to apply pressure on the breast to get better image)
(>65 yrs) have you had your bone mineral density (BMD) done? Any reason?

Obstetrical  GTPAL:
-

Have you ever been pregnant before? Any abortions (termination)? Or miscarriages (spontaneous
abortion)?
Number of babies you delivered? Any twins? Any children with congenital abnormalities?
For each delivery: was it full term or pre-term? Vaginal or CS? Any complications like high blood
pressure / high blood sugar?
Family history of: repeated abortions / CS / congenital anomalies / twins

Sexual history:
-

With whom do you live?


If (alone / with family): are you in any relationship? Are you sexually active? Have you ever been
sexually active?
If with partner: how do you describe the relationship? Is it stable? Are you sexually active? Do
you practice safe sex, and by that I mean using condoms every time? For how long you have
been together? (> 6 months  stable). And before that, were you sexually active?
When did you start sexual activity?
How many partners have you had for the last 12 months? For the last month?
What is your sexual preference? Men/ women/ both? What type of sexual activity?
Have you screened or diagnosed before with STIs? HIV? Vaginal discharge?
How about your partner? Any fever? Discharge? Burning sensation?
Do you feel safe in this relation?

What if the male partner does not like condoms? Is it ok to consider it safe sex? Yes, provided
that:
- Scan the partner for STIs first
- Strict monogamy relation (no extra-marital affairs)
- Use alternative reliable contraception (e.g. OCPs)

OSCE-guide-III.doc

Page 148 of 255

Physical Examination

History of pregnant lady third trimester


Are you doing regular ante-natal follow-up visits?

NO

Yes

Social issue

Deal with the social issue

Last visit history / pre-eclampsia





Make sure the mother is stable





Make sure the baby is stable


U/S







When was your last f/u visit?


What was your BP? Was there any headache?
Was there leg swelling? Weight gain?
Any abdominal pain? Cramps?
Vaginal bleeding? Discharge?
Any gush of water?
Is your baby kicking like before? > 6 in 2 hrs
Have you done your U/S? How many times? When
was the last time?
Number of babies?
Location of the placenta?
Amount of fluids?

N.B. to make sure the mother and baby are stable: ABCDE

Activity of the baby

Bleeding

Contractions / pain

Dripping / Discharge

EDD (expected date of delivery)

OSCE-guide-III.doc

Page 149 of 255

Physical Examination

Vaginal Discharge
Teenager / 5 minutes case
CC
How can I help you?!
Analysis of CC
Os Cf D
COCA Blood / color / fishy odour?
-

- Related to periods
- Related to sexual intercourse (bact vaginosis: discharge post-coitus)
- LMP / regular / how often / similar to previous ones?
M
Same system
HPI
- Any pain? With intercourse?
AS
- Itching? Redness?
? Candida
DD
- Any blisters / warts / ulcers13?
- Inguinal swellings?
- Urine changes? Dysuria, frequency?
Nearby systems
- Bowel movements changes? GIT symptoms
- Abdominal pain  OCD / PQRST /
- ? PID  Adnexal tenderness / fever
- Dissemination to liver (pain Rt upper abd)
DD
- Constitutional symptoms
- Sore throat? Mouth ulcers? Red eyes?
- Joint swelling/pain? Skin rash? Reiters
G
- IUD
- PAP smear!
- History of STI / PID?
O
Complete sexual history for both partners
S
PMH
- Any medications? Recent use of antibiotics
- Allergies
- DM
FH / SH
- How do you support yourself?
- HEAD SS / SAD
Conclusion: STI because of risky sexual behaviour

Physical examination including pelvic, speculum exam / PAP smear / swabs for C&S including those
for Chlamydia & Gonorrhea / saline slide microscopy / KOH / Whiff test

DD: Gonorrhea, Chlamydia, Candidiasis (whitish), Bacterial vaginosis (thin gray, clue cells),
Trichomonas (frothy yellowish / greenish discharge, motile organism).

Treatment:
o Gonorrhea: Ceftriaxone 250 mg IM single dose
o Chlamydia: Azithromycin 1g orally single dose
o Candidiasis: Miconazole 200 mg vag supp, 1 vag supp od qhs x 3 d
o Bacterial vaginosis: Metronidazole 500 mg PO bid x 7 d
If pregnant: Amoxicillin 500 mg PO tid x 7 d
o Trichomonas: Metronidazole 500 mg PO bid x 7 d

Follow up with in 4 weeks

Her partner(s) to be notified and to come for treatment, ask about sexual health (fever, discharge)

Advice regarding safe sex (condoms, multiple partners, STIs)

Chlamydia and Gonorrhea are reportable diseases

HIV testing and other STIs screening if high risk sexual behaviour

Advise regarding PAP smear regularly, vaccination against HPV
13

Blisters: HSV (Herpes Simplex Virus) / warts: HPV (Human Papilloma Virus) / ulcers: syphilis

OSCE-guide-III.doc

Page 150 of 255

Physical Examination

Vaginal Bleeding Non-Pregnant / Not-Known Pregnant


Analysis of CC
Timing: OS Cf D When did it start to be continuous?
COCA / fresh blood vs. clots  if large amounts  impact  anemia /
dehydration symptoms (pallor, SOB, dizziness, fainting, heart racing)
-

- Related to sexual intercourse (if yes: cervical*)


- For the last few weeks, how do you distinguish between your regular
M
periods and the bleeding? Related to periods
- LMP / regular / how often / how much / similar to previous ones?
- First menstrual period? Regularity?
HPI
- Any chance you are pregnant? How do you know for sure?
Same
AS
system
- Any nausea / vomiting / breast engorgement? Frequency?
DD
- Any pain? With intercourse?
- Itching? Redness?
- Any blisters / warts / ulcers?
- Grape like tissue (hydatiform mole)
- Urine changes? Urinary symptoms?
Nearby
- Bowel movements changes? GIT symptoms
systems
- Abdominal pain: OCD/PQRST (? PID / ectopic)
- Pelvic fullness / heaviness?
- Constitutional symptoms
DD
- Bleeding disorders/ tendencies? Any bleeding elsewhere
(nose, gums, with stool, easy bruising)?
- Blood thinners? Aspirin?
- Thyroid problems? Symptoms?
- Polyps, fibroids, endometriosis, gyn cancer?
G
- Hx pelvic surgeries? Instrumentations?
- Contraception history; OCPs / IUD / HRT / mammogram
- PAP smear! Was it normal?
- Any previous pregnancies? Abortions? How many? Route
O
of delivery?
- Number of partners / safe sex / when did you start activity?
S
- Hx of STIs
PMH
- Breast cancer, mammogram (if > 40 years), HTN
FH
- Gynecological cancer / Breast cancer
SH
- How do you support yourself?
- SAD / HEAD SSS (if teenager)
Vaginal bleeding cases:
-

Middle age / risky behaviour / old abnormal Pap smear  cervical cancer.
A 48 years patient with vag bleeding and all symptoms will be negative  dysfunctional uterine
bleeding (DUB); intermittent / lose track of periods / no pain with periods.
A 52 years pt / constipation / OCP/HRT / no pregnancies14  endometrial cancer / constitutional
A 62 years pt with intermittent bleeding / small amount with secretions  atrophic vaginitis;
(menopausal symptoms / dyspareunia / itchy vulva)  Rule out cancer (endometrial biopsy) 
estrogen vaginal cream
DD: Fibroid / Cervical polyp / hyper/hypo-thyroidism / Trauma / Coagulopathy / PCOS

Investigations: pregnancy test -HCG / progesterone challenge test / hysteroscope / PAP / U/S /
endometrial biopsy / TSH
14

Cervical cancer for prostitutes (risky behaviour) and endometrial cancer for nuns (no pregnancies)

OSCE-guide-III.doc

Page 151 of 255

Physical Examination

Vaginal Bleeding Pregnant / Ante-Partum Hemorrhage






Patient usually sleeping in a left lateral position with oxygen, very anxious, crying
Reassure her & ask her to bear with you for a while to get to the bottom of her problem
Ask about her feelings & empathize (impact on her and her partner)

Analysis of CC

HPI

M
AS
DD

G
O
S
PMH

FH
SH

Timing: OS Cf D
first time or happened before
COCA / fresh blood vs. clots  if large amounts  impact  anemia /
dehydration symptoms (pallor, SOB, dizziness, fainting, heart racing)

- LMP / regular / how often / how much / similar to previous ones?


- How many weeks? date based on LMP
Same
- How did you know that you are pregnant? Was it planned?
system
- Did you have regular antenatal care? F/U visits? U/S?
- What is the result of U/S?
- Any pain / discomfort? (OCD / PQRST)? Contractions
- Water gush / grape like tissue (hydatiform mole)
- Fetal movement
- Urine changes? Urinary symptoms?
Nearby
- Bowel movements changes? GIT symptoms
systems
- Abdominal pain: OCD/PQRST (? PID / ectopic)
- Pelvic fullness / heaviness?
- Constitutional symptoms
DD
- Bleeding disorders/ tendencies? Any bleeding elsewhere?
- Headache, hand or feet swelling
- Trauma, sexual intercourse
- Polyps, fibroids, endometriosis, gyn cancer?
- Hx pelvic surgeries? Instrumentations?
- PAP smear! Was it normal?
- GTPAL
- Any previous pregnancies? Abortions? How many? How
many weeks? Route of delivery?
- Hx of STIs
- Breast cancer, mammogram
- HTN, DM, kidney disease, blood gp & Rh
- Medication / allergies / hospitalization / surgeries / blood transfusion.
- Gynecological cancer / abortions
- How do you support yourself?
- SAD / HEAD SSS (if teenager)

Threatened abortion
Separation of part of the placenta
Will do physical and obstetric examination. Vaginal exam only after U/S excludes placenta previa
- 50/50% chance to keep or loose the baby
- Admit to the hospital
- Investigations: continuous vitals monitoring / CBC / INR / PTT /
- Arrange for continuous fetal heart monitoring, U/S
fibrinogen / Rh status / blood grouping and cross matching / US /
and biophysical profile
fetal monitoring
- Management: O2, IV fluid, LLP, if fetus is still in
- Management:
distress, arrange for C/S
- If she is improving / the fetus is not distressed: she will go home,
Placenta previa
Placenta abruption
resume normal activity, come back if more blood or more pain
Painless
Painful (contractions)
- Otherwise: bed rest / steroids / fetal monitoring / Rhogam / platelets
Incomplete abortion: 3 findings on vag exam would confirm the diagnosis: (1) Cervix dilated, (2)
Ruptured membranes, (3) Product of conception passed

OSCE-guide-III.doc

Page 152 of 255

Physical Examination

Abnormal Uterine Bleeding (AUB)

Dysfunctional Uterine Bleeding (DUB):


Abnormal bleeding not attributable to organic (anatomic / systemic) disease. DUB is a
diagnosis of exclusion. Anovulatory AUB often used synonymously with DUB.

Causes:
o
progesterone: luteal phase defect (estrogen-dependent DUB)
o
PCOS
o
Endocrinal ( TSH / prolactin)
o
Stress, weight loss, exercise
o
Liver and kidney disease

Investigations for AUB:

o
o

o
o
o

o
o

Beta-hCG
CBC, serum ferritin
Coagulation profile (esp. adolescent): rule out von Willebrand's disease
TSH, free T4
Prolactin if amenorrhea
FSH, LH
Day 21 (luteal phase) progesterone to confirm ovulation
Serum androgens (especially free testosterone)
Pelvic U/S: detect polyps, fibroids; measure endometrial thickness (postmenopausal)
Sonohysterogram (SHG): very sensitive for intrauterine pathology (polyps, submucous
fibroids}
Hysterosalpingography (HSG)
Pap test
Endometrial biopsy: women > 40 years are at higher risk of endometrial cancer
 Must do endometrial biopsy in all women presenting with postmenopausal bleeding
to exclude endometrial cancer
D&C: not for treatment; diagnosis only (usually with hysteroscopy)

OSCE-guide-III.doc

Page 153 of 255

Physical Examination

Treatment of AUB:
Treat underlying disorders / if anatomic lesions and systemic disease have been ruled out,
consider dysfunctional uterine bleeding (DUB)

Medical:
o
Mild DUB







o

NSAIDs
Anti-fibrinolytic (e.g. Cyklokapron) at time of menses
Combined OCP
Progestins (Provera) on first 10-14 days of each month if oligomenorrheic
Mirena IUD
Danazol (pseudo-menopause)

Acute, severe DUB





Replace fluid losses, consider admission


Medical treatment:
(a) estrogen (Premarin) 25 mg IV q4h x 24h with Gravol 50 mg IV/PO q4h or
(b) Ovral15 1 tab PO q4h X 24h with Gravol 50 mg IV /PO q4h
Taper Ovral: 1 tab tid X 2d  bid X 2d  OD
After (a) or (b), maintain patient on monophasic OCP for next several months or
consider alternative medical treatment

Clomiphene citrate: consider in patients who are anovulatory and who wish to get
pregnant
Surgical:
o
Endometrial ablation; consider pre-treatment with danazol or GnRH agonists
o



o

If finished childbearing
Repeat procedure may be required if symptom recurrence

Hysterectomy: definitive treatment

Post-coital bleeding / middle age:

STIs

Cervical (Cervicitis / Cervical polyp / Cervical cancer / Ectropion; if OCPs)

Bleeding tendencies

Trauma
Management:

Vaginal and cervical swabs; C&S and gram stain

Polyp: Sonohysterogram

Biopsy

If ectropion: stop OCPs, cautery (silver nitrate)


Cancer uterus:
Risk factors:

Early menarche
Nulliparity
Weight gain
HRT / estrogen therapy
Unopposed estrogen

Management:

Endometrial biopsy; if positive


o
Total abdominal hysterectomy and bilateral salpingo-oophorectomy
o
Adjuvant chemotherapy
15

Ovral is progestin (levonorgestrel) and estrogen (ethinyl estradiol) combination OCP

OSCE-guide-III.doc

Page 154 of 255

Physical Examination

Amenorrhea
CC

Did not have periods for 6 months?!


Did you seek medical attention? Any recent changes?
Analysis of CC
During these 6 months; any irregular bleeding? Spotting?

When was your first period? What age? Was it regular? For how long it was
M
regular / not regular? How often? How much? LMP?

When it was regular; was it painful? (painless  anovulatory)

Did you use any contraception? When did you stop? Why?
HPI
AS

Any chance you are pregnant? How do you know for sure?

Any nausea / vomiting? Breast engorgement? Frequency?
DD

For how long have you been trying to get pregnant?

Any previous pregnancies? Abortions?

Constitutional symptoms?

Are you under stress?
Hypothalamus

Excessive exercise?

Any concerns about your weight? (anorexia)

Any headache? Vomiting in morning? Visual changes? Pituitary
Difficulty seeing to sides? Milk secretions from breast?

History of thyroid disease? Heat/ cold intolerance?
Bowel movements? Moist/ dry skin?

Do you have excessive hair growth? Acne? Did you
Ovarian
notice any weight changes? Hx of DM / thirsty /
frequency? Fm Hx of PCOS?

Hx of chemotherapy? Radiotherapy? Hot flushes?
Vaginal dryness? Soreness?

Any change in your voice? Muscle bulk?

Any repeated surgical procedures? D&C?
Uterine
G

Pelvic surgeries? Instrumentations?

PAP smear!

Any previous pregnancies? Abortions?
O
S

Hx of STIs
PMH
- Any medical conditions? Psychiatric illness?
- Any medications? Recent use of antibiotics
FH
- Family hx of PCOS / infertility?
SH
- How do you support yourself?
- SAD

Investigations: -HCG / progesterone challenge test / hormonal assay (estrogen / progesterone / FSH /
LH / prolactin / thyroid-TSH / serum testosterone; total and free) / US / CBC

Ovarian causes of amenorrhea: PCOS /+/ Premature ovarian failure /+/ Androgen-producing tumours

PCOS (poly-cystic ovarian syndrome):

History: previous pregnancy / contraception hx

To diagnose PCOS: must have 2 of 3 criteria: (1) oligomenorrhea / irregular menses for 6
months, (2) hyper-androgenism (hirsutism or blood level), (3) PCOS by US

Investigations: -HCG / US / High LH:FSH ratio > 2:1 / Fasting blood sugar

Treatment:
o Lifestyle modification ( BMI, exercise) to peripheral estrone formation
o Metformin 500 mg PO tid
o Clomiphene citrate; if she wants to become pregnant
o Tranexamic add (Cyklokapron); for menorrhagia only
o OCPs; if she does not want to become pregnant

Dysmenorrhea:

DD: PID, fibroid, endometriosis

Investigations: U/S to exclude other conditions

Treatment: NSAIDs (ibuprofen 400 mg tid), OCPs.

OSCE-guide-III.doc





Page 155 of 255

Physical Examination

Infertility
Transitional statement before going in details with the history:
In order for a couple to achieve pregnancy, both partners should be capable of having children
and relatively healthy. For that reason, I am going to ask some questions about your health and
your partner health; some of these questions are personal, but it is important to ask. And I would
like to assure you that all the conversation is strictly confidential and I will not release any
information without your permission!
Introduction
CC
Analysis of CC
For how long have you been trying to conceive?
- Anovulation (irregular cycles / painless / no pre-menstrual syndrome)
HPI DD
- PCOS: Do you have excessive hair growth? Acne? Did you notice any
weight changes? Hx of DM / thirsty / frequency? Fm Hx of PCOS?
- Tubal occlusion: surgeries / STIs and PID / IUD
- Endometriosis: dysuria / dyspareunia / dyschezia / back pain
- History of chemotherapy and radiotherapy
- Do you have hx or were diagnosed with any gyn disease (e.g. polyps)?
Gynecology
- Did you use contraception? What? Since when? When was the last time?
Hx
- Have you ever had Pap smear before? When was the last time? Any
reason (if long time)? What was the result?
- Have you ever been pregnant before? Any abortions (termination)? Or
Obstetric Hx
miscarriages (spontaneous abortion)?
- Family history of: repeated abortions / CS / congenital anomalies / twins
Coital history - For how long are you in this relation? For how long have you been
trying to achieve pregnancy?
- How frequent? Regularity?
- Are you aware of sexual cycle and ovulation (both you and your partner)
- Was he sexually active before? Did he father kids from another partner
Partner
before?
history
- Did he receive any chemo or radio therapy?
- Does he complain of any testicular problem; varicose veins,
inflammation?
- Does he complain of any penile discharge?
- Did he go through any investigations; e.g. semen analysis?
PMH
- Allergies
- DM
FH
Infertility
SH
Investigations:

Semen analysis

Ovulation documentation (mid-luteal phase progesterone; d 21-22 / US)

Tubal patency (HSG / laparoscopy)


When to start investigations:

If the woman is < 35 years  after 12 months of trying to conceive

If the woman is 35-40 yrs  after 6 months of trying to conceive

If the woman is > 40 years  start investigations after 1 months of trying to conceive

OSCE-guide-III.doc

Page 156 of 255

Physical Examination

Counselling pre-eclampsia
36 weeks pregnant lady comes for f/u visit, BP 160/110, +++ protein in urine, Manage.
Like the B12 results case
Introduction
I will discuss results with you
Ethical challenge: travel permission
History
Last visit history / pre-eclampsia
Make sure the mother and baby are stable
U/S
Obstetric history / Gynecological history
PMH / Social history
Counselling
Explain what is pre-eclampsia
Serious concerns with pre-eclampsia
Management
Hospitalize
If insisting to leave  sign a LAMA
Introduction
- Good afternoon Ms I am Dr I understand that your blood pressure was measured and urine
test was done, I have the results with me and I will discuss it with you. However, because this is
my first time to see you, I need to ask you some questions, to get a better understanding of your
health condition, is that ok with you?
- Is this you first time to have these checks during your pregnancy?
- Are you under regular follow-up?
o Yes  proceed to history
o No  any reason? My husband had a car accident! I am sorry to hear that; was he hurt?
Was anyone else hurt? When was that? It must be difficult, how did this affect your life?
Ethical challenge: travel permission
o Actually I am here to get a note.
- What type of notes?
o Travel note, I really need to travel.
- It looks like it is an important trip for you; usually pregnant ladies do not travel during this time of
pregnancy!
o It is a business trip that would save our financials.
- I see it is important for you, however, before we proceed, let me check your health condition first,
and I will start by asking you some questions:
History
Last visit history / pre-eclampsia
- When was your last f/u visit?
- What was your BP? Was there any headache?
- Was there leg swelling? Weight gain? Did they do urine test?
- How about before being pregnant? Any hx of high blood pressure?
Make sure the mother and baby are stable: ABCDE
- Activity of the baby, is your baby kicking like before?
- Bleeding
- Contractions / pain
- Dripping / Discharge
- EDD (expected date of delivery)
U/S
- Have you done your U/S? How many times? When was the last time?
- Number of babies?
- Location of the placenta?
- Amount of fluids?
Obstetric history: any pregnancy before / any similar conditions? Gynecological history
PMH: high blood pressure Social history: SAD / support / home environment

OSCE-guide-III.doc

Page 157 of 255

Physical Examination

Counselling
Explain what is pre-eclampsia
- Your blood pressure is 160/110, which is high, and the urine test shows protein in large amount
(+++) which is not normal, the most likely diagnosis is a medical condition called pre-eclampsia
OR pregnancy-induced hypertension.
- I would like to ask more questions to see how it affected you!
o My dad had HTN, and lived with it, I am ok.
- These are different conditions; your dad had HTN, but you have pregnancy-induced HTN,
which is a serious condition, with very serious and may be fatal consequences.
o Have you had hx of headache?  OCD / PQRST (not detailed)
o Nausea / vomiting
o Change in your vision? Flashing lights? Flying objects?
o Any abdominal pain in your upper right part of your abdomen?
o Any bruises? Yellowish discoloration / itching / dark urine / pale stools?
o Any chest pain / heart racing / SOB?
o Any weakness / numbness?
o Any swelling in your body / face/eyes? Did you feel your shoes tight?
o Did you gain weight?
o Any changes in the urine? Frothy? Burning sensation?
- Based on all this, the most likely explanation for your increased is pre-eclampsia; and this is a
very serious condition, we need to admit you to the hospital to monitor you. Then, the obstetrician
will assess you and may consider delivering the baby now.
o But doctor, I need to travel, just 2 days and I will come back.
- I understand your concern about traveling, but we have a serious situation here.
- We do not know exactly why patients have pre-eclampsia. We believe it is imbalance of
hormones, or it might be related to placenta, however the only treatment is delivering the baby.
Serious concerns with pre-eclampsia
- What happens is that there is a narrowing of blood vessels, this leads to the amount of blood
reaching the baby, subsequently the amount of oxygen and nutrients. On the long term this will
lead to some injury and even damage to the baby AND the mother.
o This includes your heart and blood vessels, that is why you have BP,
o This includes your kidney, that is why you have +++ protein in urine,
o This includes your liver, that is why you may have abdominal pain,
o This includes your brain, that is why you have headache, visual changes,
o This includes your baby, that is why he is not kicking like before
This is not because of your pregnancy; all of these are due to this condition.
- The concerns we have is that we can not predict the outcome, without the proper medical care,
patients having pre-eclampsia will end up going to the next stage which is eclampsia; do you
any idea what is e0clampsia?
- A condition in which, the patient will start to seize, lose conscious, will not be able to breath and
turn blue. The only resolution for this is delivering the baby.
- Imagine that I give you the note, and they allow you to take the trip, 2 hours later while you are in
the plane, you start to fall down and seize. What will happen? Nobody will be able to help you.
- By this you endanger your life and your babys life.
Management
- What we need now is to admit you to the hospital and arrange for obstetrical assessment.
- If insisting to leave  sign a LAMA (leaving against medical advice)
- Suggest solution for her business travel, like giving a doctor note that she needs to be hospitalized.
Treatment Plan
- Assess severity including good history and physical exam focusing on heart, lungs, reflexes, fetus,
urine analysis and BW (important CBC, liver function tests, Uric Acid)
- If all above are stable, consider daily check, urine dips and fetal kick counts as outpatient. If any
of above unstable may need to hospitalize as inpatient for close monitoring
- Measure L/S ratio of the baby, give corticosteroids for lung maturation
- MgSO4 and delivery
- Blood pressure controlled often with labetalol, Ca channel blockers

OSCE-guide-III.doc

Page 158 of 255

Physical Examination

Caesarean Section Counselling wants to have CS


Young 18-20 years old pregnant lady would like to have CS, counsel her.
Introduction
Any reason you want to have CS?
Social issue
History
Last visit history / pre-eclampsia
Make sure the mother and baby are stable
U/S
Obstetric history / Gynecological history
PMH / Social history
Counselling
Address patient concerns
Why not caesarean section?
Management
Refer to obstetrician
Spend some time to think / stabilize
Introduction
Any reason you want to have CS?
- I understand that you are here to discuss the possibility of CS; we will discuss this in details, but
before that I would like to ask you is there any reason you would like to have CS?
o I do not want to have this severe pain!
- How do you know it is painful?
o I had previous abortion
OR
o I attended my sister delivery and it was very painful experience
- When was that? Did you attend?
o Congratulations! How is your sister doing? How is the baby?
o I understand that you saw her in pain, but people differ! And within few minutes I will be
explaining different options to control labour pains!
History
- Let me ask you some questions to assess the condition first!
o How do you feel? How is your mood?
o How about this pregnancy, was it planned? How do feel about your pregnancy? How is
the feeling of your partner?
o Are you under regular follow-up? NO! Any reason?
There may be social issue here.

Empathy: it looks like you are doing through difficult times! How are you coping?

Offer social support: being pregnant lady without support, you have priority and
there are a lot support and resources in the community. I will make sure to connect
you with social worker who will help you with proper support (housing / financially /
for both of you and the baby)
Last visit history / pre-eclampsia
- When was your last f/u visit?
- What was your BP? Was there any headache?
- Was there leg swelling? Weight gain? Did they do urine test?
- How about before being pregnant? Any hx of high blood pressure?
- Did you have your blood sugar checked? How about before pregnancy?
Make sure the mother and baby are stable: ABCDE
- Activity of the baby, is your baby kicking like before?
- Bleeding
- Contractions / pain
- Dripping / Discharge
- EDD (expected date of delivery)

OSCE-guide-III.doc

Page 159 of 255

Physical Examination

U/S
-

Have you done your U/S? How many times? When was the last time?
Number of babies?
Position of the placenta?
Amount of fluids?

Obstetric history: GTPAL


- Any pregnancy before? Any abortions or miscarriages?
- What were the circumstances? How many weeks?
- How did you feel about it? How did you cope with that?
Gynecological history: Fibroid, Genital herpes
PMH: Medications / allergy / blood transfusion
Social history: SAD / support / home environment
Counselling
Address patient concerns
- I know that you are here to talk about CS. But first let me explain some facts about delivery. The
natural route for delivery is the vaginal delivery, and if there is no real indication for CS, we
would prefer to go for vaginal delivery.
- However, I appreciate you concerns, if your concern is the pain there is a lot of options to control
it.
o We can start by learning some relaxation techniques
o And then on the delivery day, when you go there, there are a lot of support groups; one of
them is called doolas; they attend with you and they provide a lot of emotional support
o Finally, when you start your labour, we have good measures; and I mean what we call
epidural anesthesia; this is very effective and safe. Where the anesthesiologist puts a
needle into your back and injects a freezing substance that helps you to go through the
delivery without pain (this is like the dentist freezes your mouth before doing painful
procedures). It might however cause headache, bleeding, and less likely infection.
Why not caesarean section
- I would like to ask you; what is your understanding of CS?
- It is commonly used obstetrical intervention, used when there is a problem or contraindication for
vaginal delivery and if there is an emergency situation that necessitates immediate delivery; and in
these cases it is life saving; for both the mother and the baby!
- However, it is a major surgery, has the risks of bleeding, higher risk of infection, DVT, you stay
longer in the hospital, and it will leave scar in your abdomen.
Management
- After all, I am not the person who makes the decision; this should be decided by the obstetrician.
- I am going to refer you to the obstetrician; who will perform further and detailed assessment then
discuss the results with you.
- Meanwhile, I would recommend you spend some time to think about what I told you, try to
stabilize yourself emotionally.
-

I will give you some brochures and web sites so that you can read more about that.
I will connect you with the social worker.
And if at any time you have any questions or concerns, you can come to see me.

OSCE-guide-III.doc

Page 160 of 255

Physical Examination

Caesarean Section Counselling does not want to have CS


Middle aged pregnant lady (36 weeks) is here to have her file. Three years ago, she had urgent CS
for cord prolapse. Now she would like to deliver at home with the midwife.
Introduction
Any reason you want to have your file?
Concern
Deal with the patient concern
What was the type of your CS?
History
Last visit history / pre-eclampsia
Make sure the mother and baby are stable
U/S
Obstetric history / Gynecological history
PMH / Social history
Counselling
What is CS? The two types of CS
Risks of vaginal delivery post CS
Management
Prepare a copy of the file
Speak with your midwife
Introduction
Any reason you want to have your file?
- I understand that you are here to have a copy of your file for the urgent CS you had 3 years ago.
This is your right, and I will ask someone to prepare a copy for you.
- But first I would like to ask you some question, is it ok with you!
- Is there any reason you would like to have your file?
o Yes, I would like to have delivery at home this time. A lot of my friends did it at home
with the midwife and they say it is much easier and relax.
- Are you seeing obstetrician? Are you doing any regular follow-up visits? Any reason for that?
o No, I am going to follow up with the midwife.
o I did not like last time when they did CS at the hospital!
- What happened last time?
o They told me the baby had cord prolapse!
- Were there any consequences?
o How is the baby doing? How old is he? What can he do? Walk? Talk?
o How about you? Any complications? Infections? Scars?
Deal with the patient concern:
- What is your understanding of cord prolapse?
- It is a condition where the umbilical cord goes into the birth canal before the head, and then gets
stuck and squeezed by the head.
- This is a very serious condition. The cord delivers blood and nutrients to the baby. If blocked for
long time, the baby will suffer from brain damage.
- That is why they had to do urgent CS, which was a life saving procedure for the baby, and it had
to be done immediately.
- Do you know what the type of your CS was?
Classical!
History
- I understand your point of view, but first let me ask you some questions about your health and
your pregnancy!
Last visit history / pre-eclampsia
- When was your last f/u visit?
- What was your BP? Was there any headache?
- Was there leg swelling? Weight gain? Did they do urine test?
- How about before being pregnant? Any hx of high blood pressure?
Make sure the mother and baby are stable: ABCDE
- Activity of the baby, is your baby kicking like before?

OSCE-guide-III.doc

Page 161 of 255

Physical Examination

Bleeding
Contractions / pain
Dripping / Discharge
EDD (expected date of delivery)

U/S
- Have you done your U/S? How many times? When was the last time?
- Number of babies?
- Location of the placenta?
- Amount of fluids?
Obstetric history: GTPAL
- Other than the pregnancy that you had CS 3 years ago; any pregnancy before? Any abortions or
miscarriages?
- What were the circumstances? How many weeks?
- How did you feel about it? How did you cope with that?
Gynecological history
PMH: Medications / allergy / blood transfusion
Social history: SAD / support / home environment
Counselling
What is CS? The two types of CS
- I would like to ask you; what is your understanding of CS?
- It is commonly used obstetrical intervention, used when there is a problem or contraindication for
vaginal delivery and if there is an emergency situation that necessitates immediate delivery; and in
these cases it is life saving; for both the mother and the baby!
- There are two types of CS:
o The transverse (done at the lower segment of uterus); it is the most common type; its
advantages include: smaller scar and better healing.
o The classical or vertical type; it is done less common; as we cut through the muscle fibers
of the uterus it produces weaker scar; but it is indicated and actually needed in urgent
case, like yours. As it allows quick access and fast delivery, because in some cases (like
cord prolapse) we can not afford even few minutes more.
Risks of vaginal delivery post CS
- Due to the scar formed after the CS procedure; it is always recommended to deliver by CS, to
avoid the tearing pressure of the uterine contractions during vaginal delivery.
- If you decide to go for vaginal delivery, my concern is that the scar might undergo severe tearing
pressure and might rupture, which will lead to massive bleeding. This is an obstetrical emergency
that necessitates immediate intervention. Because you may end up losing your life and/or losing
your baby.
- I do not want to scare you, but the risks of having uterine rupture after classical CS is 12%, of
which 10% of cases end up losing their lives.
- For that reason: once classical CS, it is always CS.
In case of counselling transverse CS:
- Risks of having uterine rupture after transverse CS is 1%.
- Even though, if you want to try vaginal delivery, we can not take the risk to try this at home, we
can try this in the hospital, so that just in case any emergency might happen, we can intervene in
the proper time.
Management
- I will ask someone to prepare a copy of your file
- Speak with your midwife:
- I am sure that your midwife is highly trained and qualified, and we share the same guidelines. I
would recommend that you take your file and speak with your midwife, and I am sure she will
explain the situation to you.
- I will give you some brochures and web sites so that you can read more about that.
- And if at any time you have any questions or concerns, you can come to see me.

OSCE-guide-III.doc

Page 162 of 255

Physical Examination

OCPs / Contraception Counselling


XX is a 16 years old girl. She would like to get information about OCPs. For the next 10 minutes
talk to her
Introduction
Concerns

History

Exclude
pregnancy
M
G

O
S

Do you have any concerns?


Why do you need / think about contraception at this point?
Have you used contraception before? What is your previous
experience? Why did you stop it?
Any chance you are pregnant? How do you know for sure?
Any nausea / vomiting? Breast engorgement? Frequency?
Painful periods? / Irregular? / Heavy bleeding?
Last PAP / any abnormal PAP
Previous D&C
GTPAL / IUD is not recommended in nullipara
DETAILED
If risky behaviour: OCPs will not protect against STIs
Risky behaviour or previous STI/PID: NO IUD

Definition: birth control is an umbrella term for several techniques and methods used to prevent
fertilization

Hormonal (OCPs / implants / injections)

IUD (contragestion: prevents the implant)

Barrier methods (condoms / diaphragms) spermicidal

Behavioural (fertility awareness/timing) / coitus interruptus

Post-coital contraception

Sterilization (male / female)


OCPs
IUD
Mechanism of

OCPs are hormones: estrogen and

Mechanically prevents the implantation of


action
progesterone
the fertilized ovum

OCPs prevent ovulation, increase thickness Available forms: Copper / hormoneof cervical secretion
coated

21 tab + 7 sugar pills  28 days

Structural uterine anomalies


Contraindications HTN / CAD / Cerebro-vascular disease /
DVT

History of ectopic pregnancy

Breast or gynecological cancer (past


personal and family hx)

Undiagnosed vaginal Bleeding

Undiagnosed vaginal Bleeding

History of PID(s)

Active liver disease

Risky behaviour

Smoker > 35 yrs

Migraine
Benefits

Regulate periods

Longevity

Independence to coitus or compliance


ABCD:

Improves anemia ( bleeding)

benign breast lesions

ovarian cysts and cancer

risk of uterine cancer

dysmenorrhea

Available
methods

OSCE-guide-III.doc

Page 163 of 255

Physical Examination

Side effects

Does not prevent STIs

Heavy periods
risk of DVT / CAD

If hormonal coated: prog side effects:


TRANSIENT:
headache / wt gain / mastalgia

Breast tenderness

Weight gain

Headache

Nausea / vomiting

0.1% 5%

0.5% 2%
OCPs
IUD

Will do physical and pelvic examination, and document blood pressure, pap smear

Brochures and information

Follow up within 1 month

How we will start:

First day of next menstrual cycle

Put package in obvious place that you see


frequently

Take the pill in the same approximate time


every day

Use another method of contraception for the


first TWO months

If you miss a pill: see below

Failure rate
Plan

NOTES


N.B.: if woman with risky behaviour:


- Counsel for safe sex
- OCPs do not protect against STIs

Failure rates are very high with behavioural methods:


- Because it depends largely on the knowledge, experience of user and the usage technique;
perfect-use versus typical-use failure rates.

What happens if I missed 1 or 2 pills (OCPs)?


- 1 missed pill  take 2 pills the next day
- 2 missed pills  take 2 pills a day, for the next 2 days
- If missed more than 2 consecutive pills: use a backup method of contraception
simultaneous to finishing up packet of pills (2 pills a day) until next menses

Woman comes to request sterilization (tubal ligation)


1- Discuss various other alternatives, make sure she knows everything about all of them
(OCPs, IUDs, diaphragm, condoms, vasectomy)
2- Counsel the patient on the permanent nature of the procedure, the operative risks, and
the chance of failure (1 in 200)
Counsel about the risks of regret for the decision (young age, recent emotional trauma,
family coercion)
3- Bring the patient back after 1 month for the bilateral tubal ligation procedure,
preferably immediately after menstruation to decrease possibility of pregnancy

OSCE-guide-III.doc

Page 164 of 255

Physical Examination

OCPs

IUD

IM
Injection

Advantages
- Regulate periods
ABCD:
- Improves anemia (
bleeding)
- benign breast lesions
- ovarian cysts and cancer
- risk of uterine cancer
- dysmenorrhea
- Longevity
- Independence to coitus or
compliance

- Dysmenorrhea
- Effective
- Independence

Contraindications
Pregnancy
Un-dx vag bleeding
Cerebro-vascular dis / CAD
Active liver disease
Hormone-dep cancer
Smoker > 35 yrs
Hx of DVT / PE
Migraine

Pregnancy
Un-dx vag bleeding
Structural uterine anomalies
Hx of PID(s)
Risky behaviour
Hx of ectopic pregnancy

Condoms
Diaphragm
Behavioural

OSCE-guide-III.doc

Available
Combined pills (E+P):
low dose estrogen (20,
35, 50)

Failure rate
0.1% 5%

TRANSIENT:
- Breast tenderness
- Weight gain
- Headache
- Nausea / vomiting
- Heavy periods
- Copper
- If hormonal coated: prog - Hormone coated
side effects: headache /
wt gain / mastalgia

0.6% 2%

- NO absolute contraindications

- Irregular bleeding
- Amenorrhea after 1 year
- Delayed post-use
fertility

Every 3 months

< 1%

- Irregular bleeding
- Headaches
- Mood changes

Every 5 years

< 1%

Depoprovera
SC implants - Longevity
- Effective
Nor-plant
- Independence to coitus or
compliance

Side effects
- Does not prevent STIs
- risk of DVT / CAD

Un-dx vag bleeding


Acute liver disease
Thrombophlebitis
Thromboembolic disease

14%
20%
- High failure rates up to 25%
- To decrease the failure rate, can combine 2 methods

Page 165 of 255

OB-GYN

HRT counselling
Introduction / overview
History
General

How do you describe your general health?


Support
DETAILED
D&C / OCPs / PAP / mammogram / BMD
GTPAL
Dyspareunia

Irregular menstrual cycles

Vasomotor symptoms: sweating / hot flashes (hot flushes) /
palpitations

Uro-genital symptoms: vaginal dryness / soreness /
superficial dyspareunia / urinary frequency and urgency

Neurologic symptoms: mood changes / sleep disturbance /
depression / anxiety
Breast Cancer
Uterine Cancer
Ovarian Cancer

Early menarche

Obese

Late menopause

Diabetic

OCPs

Nullipara

No breast feeding  PCOS

Age

Past medical history of cancer or biopsy

Family history

Screen the first major risk factors for osteoporosis (see
osteoporosis counselling case)
What is your understanding about HRT?

Local preparations: creams / pessaries / rings

Systemic formulations: oral drugs / trans-dermal patches and
gels / implants



M
G
O
S
Menopausal
symptoms

Risk factors for


CANCER

Risk factors for


Osteoporosis
Counselling
Common forms / products
of therapy

Indications for HT




Contraindications to HT
Pre-treatment evaluation
Adverse effects and risks
Any concerns?

OSCE-guide-III.doc

Estrogen alone
Combined estrogen and progestogen
Selective estrogen receptor modulator (SERM)
Menopausal symptoms For SHORT term only, 1 2 years
Osteoporosis

Hx / PE / baseline investigations
Media spoke that HRT increases incidence of stroke, heart attacks
and breast cancer, this was done by the (Women's Health
Initiative), on the other hand, smoking, obesity, cholesterol
increases the risk of these dis much more than HRT. In your case,
you do not have the risk factors for cancer, and it will be
beneficial for your hot flashes, vaginal dryness, and will protect
you against osteoporosis In medicine we always weight risk /
benefits

Page 166 of 255

OB-GYN

Introduction / overview:
- The reproductive years of a womans life are regulated by production of the hormones
estrogen and progesterone by the ovaries. Estrogen regulates a woman's monthly
menstrual cycle and secondary sexual characteristics (e.g. breast development and
function). In addition, it prepares the body for fertilization and reproduction.
Progesterone concentrations rise in a cyclical fashion to prepare the uterus for possible
pregnancy and to prepare the breasts for lactation.
- Toward the end of her reproductive years when a woman reaches menopause, circulating
levels of estrogen and progesterone decrease because of reduced synthesis in the ovary,
which may lead to several symptoms, the severity of which can vary widely.
- Hormone therapy (HT) involves the administration of synthetic estrogen and
progestogen. HT is designed to replace a woman's depleting hormone levels and thus
alleviate her symptoms of menopause. However, HT has been linked to various risks, and
debate regarding its risk-benefit ratio continues
Contra-indications of HRT:
No absolute contraindications of hormone therapy have been established. However, HT is
relatively contraindicated in certain clinical situations (similar to OCPs):
- Breast and/or endometrial cancer
- Undiagnosed vaginal bleeding
- Acute liver disease
- Thromboembolic disorders / DVT
- Endometriosis / Fibroids
- Diabetes, HTN, Heart disease
Required baseline investigations
- CBC
- Urinalysis
- Blood sugar levels
- Fasting lipid profile
- Electrocardiography
- Pap test
- Ultrasonography to measure endometrial thickness and ovarian volume
- Mammography
Possible adverse effects are as follows (similar to OCPs):
- Nausea / bloating
- Fluid retention
- Weight gain (equivocal finding)
- Mood swings (associated with use of relatively androgenic progestogens)
- Breakthrough bleeding
- Breast tenderness
- HT may slightly increase the risk for breast cancer
- There is association between HT and uterine hyperplasia and cancer
- There is increased risk of thromboembolism with HT

OSCE-guide-III.doc

Page 167 of 255

OB-GYN

Needle Stick Counselling HIV


You are about to see Mrs 33 years old female nurse, upset because she had just had needle
stick after she gave an IV injection to a patient. Counsel her.
Introduction
Concerns
History

Inform the
patient about
HIV

Plan

Workup

HIV infection / fatal disease / will impact her family


- Can you tell me what happened?
- Complete immunization record, including tetanus and hepatitis B
- Previous occupational exposure to body fluids
- Intravenous drug abuse
- Sexual history
- What is HIV? Major pathogens of concern!
First of all; let me tell you the transmission rates: (no accurate studies)
- Risk of blood transmission is: 0.3% for percutaneous exposure
- Risk of female to male transmission is: 0.03%
- Are you pregnant? Risk of intrauterine tx is: 3% with treatment and 30%
without treatment
- Advancement of HIV treatment
Address pregnancy concerns:
- Patient should receive ttt (not teratogenic)
- HIV positive mothers should not breastfeed their babies
- I will speak with the patient, explain the whole situation and ask him to
consent for HIV status
- If he agrees; we will know possibility of tx to you. If he is HIV negative,
NO post-exposure prophylaxis is needed
- If he refuses or if he is HIV +ve; we will have to assess what is called
exposure code and match it with HIV status code; to simplify this,
guidelines state we should assume you were exposed and give
prophylaxis treatment: 4 weeks of 2 anti-virals (the basic regimen)
- Hepatitis B vaccination 3 doses + immunoglobulins (immediately)
- Blood tests for the patient if possible and for the exposed
- I am going to speak with the patient now, and I will come back to you
with his decision.
- Any other questions or concerns?

What is HIV?
- Human immunodeficiency virus (HIV) is a blood-borne, sexually transmissible virus.
The virus is typically transmitted via sexual intercourse, shared intravenous drug
paraphernalia, and mother-to-child transmission (MTCT), which can occur during the
birth process or during breastfeeding.
- The major pathogens of concern in occupational body fluid exposure are HIV, hepatitis
A, hepatitis B, hepatitis C, and hepatitis D. These pathogens are viruses that require
percutaneous or mucosal introduction for infectivity. The major target organs are the
immune system (HIV) and the liver (hepatitis).

OSCE-guide-III.doc

Page 168 of 255

OB-GYN

Advancement of HIV treatment:


- It used to be fatal
- Currently, it is not cured, but controllable, may be in the future they can develop a cure to
it
- A lot of anti-virals were developed since 1990s till now, with efficacy and side
effects, we started by giving many pills q4h, now it is once or twice a day
- We follow the guidelines with monitoring of what is called viral load and cell counts
of the patient immunity cells. To decide when to start treatment
Workup:
- Source patient (if available)
o HIV
o Hepatitis B antigen
o Hepatitis C antibody
- Victim/health care worker
o HIV; testing now, at 1 month, and at 3-6 months
o Hepatitis B surface antibody / titre (if vaccinated)
o Hepatitis C antibody; testing now and after 4 and 8 weeks
- Prior to initiating retrovirals:
o Pregnancy test (stat) if she is not pregnant
o CBC count with differential and platelets
o Serum creatinine/BUN levels
o Urinalysis with microscopic analysis
o AST/ALT levels / Alkaline phosphatase level
o Total bilirubin level
The ethical questions that might arise:
- We can solve all this by calling the lab and adding HIV status check for the list of blood
works of the patient, we had just send his blood to the lab!
o You are right, this will save us the whole prophylaxis plan, however, we need to
speak to the patient first; we can not do this HIV status test with the patient
knowledge and consent.
o What I am going to do to help you is that after we finish, I will go to speak
with the patient, explain the whole situation to him, and ask his permission that
we do this HIV statue test.
- Even if the patient refused, we can ask Dr , his surgeon, may be he knows!
o May be Dr knows the patient status or may not, but if the patient refuses to let
us know, it is patient confidentiality, we can not ask Dr about this information
unless the patient consents we can ask the surgeon.

OSCE-guide-III.doc

Page 169 of 255

OB-GYN

Counselling PAP smear


CC

I understand you are here because you have some inquiries/worries about your
last PAP test, is this right? How can I help you today?
HPI
M

Previous Pap test? How many? How frequent? Any abnormal Pap test?
G
Any previous colposcopy?

Contraceptive history
GTPAL
O

RISK factors for cervical dysplasia:
S
- Early age of sexual activity
- Risky behaviour: unprotected sex / multiple partners
- Smoking
AS
Same system
- Any pain? With intercourse?
- Discharge? Itching? Redness?
- Any blisters / warts / ulcers?
- Inguinal swellings?
Nearby systems
- Urine changes? Dysuria, frequency?
- Bowel movements changes? GIT symptoms
- Abdominal pain  OCD / PQRST /
General
- Constitutional symptoms
PMH
Any allergy / medication / disease
FH
Gynecological tumours
SH
If teenager: HEAD SSS
COUNSELLING:
- What do you know about (LGSIL)? What would you like to know?
- Have you had any experience with in the past?
- Have you [read / talked to someone / searched the internet] about this issue?
Worried about PAP results
- PAP smear or test is done to screen for any changes that might happen in the cervix,
before it turns to serious disease (to early detect pre-malignant lesions).
- At the cervix there is transitional zone between two types of cells, it undergoes rapid
growth, if there is irritation due to HPV, it might turn malignant. It takes years from the
moment it begins to grow abnormally to the moment it becomes malignant, that is why
we do frequent PAP tests, to detect it before it turns into malignant tumour.
- The results come back from PAP test either ASCUS (Atypical squamous cells of
Undetermined Significance) / LG-SIL (low grade squamous intra-epithelial lesion) / or
HG-SIL (high grade squamous intra-epithelial lesion)

OSCE-guide-III.doc

Page 170 of 255

OB-GYN

For ASCUS:
o Woman 30 yrs  HPV DNA testing
 If negative  repeat cytology after 1 year
 If positive  colposcopy
o Woman < 30 yrs  repeat cytology in 6 months
 If negative  repeat after 6 months  still negative  routine screening
 If ASCUS  colposcopy
For LG-SIL:
o Colposcopy
o Or repeat cytology after 6 months
 If negative  repeat after 6 months  still negative  routine screening
 If ASCUS  colposcopy
For HG-SIL:
o We send you for colposcopy
For colposcopy, we will refer you to the gynaecologist who will perform special
procedure, during which, the gynaecologist will take a biopsy, and send it for further
investigations;
o If the biopsy is negative, we will repeat the PAP after 6 months
o If the biopsy is positive, we will do more investigations to establish a diagnosis
and may need to do another larger biopsy called cone biopsy

Treatment options:
- Laser
- Cone biopsy
- LEEP (loop electrosurgical excision procedure)
Colposcopy
- Colposcopy is a magnification of the cervix (10-12 times), the procedure may cause some
discomfort but is not painful.
- The gynaecologist will insert a speculum (the same instrument used for Pap test), and
then she/he will use a special magnification device (the colposcopy) to visualize the
cervix.
- The gynaecologist will apply acetic acid (vinegar) that helps make the vascular patterns
more visible, application of this acetic acid may give an itchy sensation.
- Then if the gynaecologist suspects a lesion, she/he will need to take a biopsy, you will
feel a punching sensation, and you might experience a little discomfort and spotting for
few days.
- You need not to have anything inserted into your vagina for 24 hours before and 2 days
after the procedure (no vaginal intercourse, no douching), and you might need to take
some OTC medications (Advil) for few days after the procedure.

OSCE-guide-III.doc

Page 171 of 255

OB-GYN

Antenatal Counselling
o

Ms XX has missed her period for 2 wks; she did a home preg test which was positive. This is
her first experience. In the next 10 min, please talk to her and give her necessary advices
about her pregnancy.
A 38 yrs old pregnant lady came to you because she is concerned about problems during
advanced-age pregnancy, counsel

Pregnancy






M
G
O
S
PMH









FH

Social Hx







Concerns

Was this planned pregnancy?


What is your feeling about that? And your partner feeling? congratulations
How do you describe your general health?
Symptoms suggesting pregnancy: any nausea / vomiting? Breast engorgement
/ heaviness? Frequency?
LMP, calculate due date
Pap smear?

OSCE-guide-III.doc

HTN, Diabetes, Kidney disease, Heart disease, Epilepsy, exposure to


chickenpox as a child, Rubella vaccine, HBV, HIV, Blood Group & Rh group
Allergy
Medications
FH of complicated pregnancies, abortions, MR, genetic & congenital
abnormalities
Home, violence, support
Environment: work
Activity: exercise / any pet
Diet
SAD
Address any problem you find while youre taking history

Page 172 of 255

OB-GYN

Counselling:


















To ensure healthy outcome of the pregnancy I need to see you on scheduled visits, every 4
weeks till the 28th week, then every 2 weeks till the 36th week, and then every week thereafter
and till delivery
Today well do physical examination including pelvic exam, Pap smear if more than 6
months, blood work including CBC, Lytes, INR/PTT, Urea, Creatinine, Blood Type, VDRL,
Rubella antibody, Hepatitis, HIV, Urine dip and microscopy, ECG.
Anatomy US at 20 weeks. Glucose challenge test at 24 weeks
Risks of Down syndrome are: 1/400 at 30 yrs of age, 1/200 at 35 yrs of age, and 1/100 above
40 yrs of age  we try to anticipate it by US and integrated prenatal screening then confirm it
by amniocentesis
U/S for nuchal translucency: at 12 weeks
IPS I: 11-14 wks /+/ IPS II: 15-18 wks (Maternal serum alpha-fetoprotein, -hCG, uE3
Unconjugated estrogen)
Amniocentesis (U/S-guided trans-abdominal extraction of amniotic fluid / for identification
of genetic anomalies): at 15-16 wks, 0.5% risk of spontaneous abortion and risk of fetal limb
injury
You need a well balanced diet; Canada's Food Guide to Healthy Eating suggests 3-4 servings
of milk products daily (greater if multiple gestation), a daily caloric increase of -100 cal/d in
the 1st trimester, -300 cal/d in the second and third trimesters and -450 cal/d during lactation.
If you do not consume an adequate diet, you can take daily multi-vitamins (avoid excess
vitamin A)
Important nutrients during pregnancy: folate; 0.4-5 mg per day / calcium; 1200-1500 mg per
day / iron: 1 mg/d in T1, 4 mg/d in T2 and > 6 mg/d in T3
Pregnant ladies tend to have constipation, you can take Lactulose for this, avoid raw or
processed meat
Haemorrhoids, back pain, heartburn and increased vaginal discharge are common
Will gain weight; 5-10 pounds in 1st half, 1 pound /week in 2nd half, total of 25-35 pounds in
average
Exercise is OK walking, swimming, avoid strenuous activities
Stay away from cats litter
No medication without asking your doctor, no x-rays
Smoking increase the risk of abortion, LBW, premature delivery
No safe level of alcohol during pregnancy, better to avoid it totally
Offer brochures, connect to support groups and classes for pregnant women

OSCE-guide-III.doc

Page 173 of 255

OB-GYN

Endometriosis
You are covering for your colleague Dr. Smith. You are about to meet Mrs. XX to discuss the
result of her laparoscopy & inform her that she has endometriosis. For the next 10 minutes, please
talk to her& address all her concerns.
-

Introduction: I would like to discuss the result of your laparoscopy but I need to get some
information.
o Why did you have laparoscopy done and what was your doctors concern?
o You have endometriosis:

This means implantation of the interior lining of the uterus somewhere in


other places outside the uterus including the ovaries, the supporting structures
of the uterus or on the intestine (draw a picture of the uterus and ovaries for
the patient).

During periods, this outside tissue also begins to bleed. This explains the
painful periods.

This may cause infertility in some people.

Sometimes it runs in the family


o I would like to ask about some symptoms (to fit everything together)

Pain: Analyze (OCD / PQRST). Relation to the period. Dysmenorrhea,


dysuria, dyschezia, dyspareunia. Do you need pain killer?

Infertility: I understand your frustration. For how long have you been trying
to conceive? Are you currently sexually active? How frequent?

Irregular vaginal bleeding  analyze

Frequency

Blood with stool, diarrhea

Menstrual history brief


Gynecological history
Obstetric history GTPAL
Sexual history brief

Past medical history: HTN, Diabetes, kidney disease, blood group & Rh. Allergies /
medications / hospitalization / surgeries / blood transfusion
Family history of abortion
Social history: smoking, alcohol, drugs / work / home environments / support

16

Conclusion: endometriosis
Plan:
o Will do physical and obstetric examination
o Give her treatment options

Medical:
 NSAIDs e.g. naproxen sodium (250 mg PO bid)
16
 Pseudo-pregnancy: OCPs trial for 6-12 months (Ovral 1 tab PO od)
 Pseudo-menopause (only short-term <6 months): Danazole (weak
androgen / Side effects: weight gain, fluid retention, acne, hirsutism,
voice change)

Surgical treatment:
 Conservative laparoscopy: laser ablation / resection of implants
 Definitive: bilateral salpingo-oophorectomy hysterectomy
o Brochure & support groups
Ovral is progestin (levonorgestrel) and estrogen (ethinyl estradiol) combination OCP

OSCE-guide-III.doc

Page 174 of 255

OB-GYN

Woman wanting an abortion


-

History
Pregnancy

Social

Abortion
Depression

Pregnancy (LMP, symptoms, how did you find out, Rh status) How do
you feel? How is the feeling of your partner about the pregnancy?

Gynecological history: contraception history, surgeries, infections, PAP

Obstetrical history: hx of previous pregnancies / GTPAL

Partner involvement? Abuse, rape?

Support; family, friends, spoken to any one else about this?

Education, Religious beliefs?

SAD

What are your thoughts regarding abortion?

Exclude depression: MI PASS ECG


PMH / FH / SH

Counselling

Pregnancy
Social
Abortion

Depression

OSCE-guide-III.doc

Be supportive and not judgmental


Health while pregnant:

Counsel for nutrition, exercise, and activity

Social support: it is important to communicate with support groups

Does not need to rush decision

Abortion options are available in Canada; most hospitals will do less


than 14-16 weeks, no problems with some up to 20 weeks. Private clinics
also provide the service

No legal gestational age where it can no longer be performed

Most commonly done via D&C under mild anesthetic

Discussion about contraception especially if not the first time (not


necessarily at this meeting)

Alternative choices: adoption / or rethink about keeping the baby with


the help of social support

Normalize the patient depressed mood and feelings in view of


circumstances

Close follow-up

May refer to psychiatrist

Page 175 of 255

OB-GYN

Osteoporosis Counselling / OR / Short Case


Age related decrease in bone mass, starts at 35 years old, accelerated post-menopause, leading to bone
fragility and an increased risk of fracture. May manifest later as kyphosis, loss of height and increased
abdominal girth
Risk factors for osteoporosis:
1- Personal history of fragility fracture (above the age of 40 years)
2- Maternal history of hip fracture
3- Malabsorption syndrome (celiac disease / inflammatory bowel syndrome)
4- Prolonged use of corticosteroids (> 7.5 mg/day for > 1-3 months)
5- Primary hyper-parathyroidism
678910-

Weight < 57 Kg
Current smoker
Premature ovarian failure (female on Tamoxifen for breast cancer / surgical menopause)
Male on androgen-deprivation therapy for prostate cancer
Heparin or anti-epileptic use or biologics (anti-cancer treatment)

Investigations:
- BMD:
Age group
When to do BMD
-

< 50 years
If > 2 of the first (5)
risk factors

50 65 years
If > 2 of any from the list
of risk factors

> 65 years
Always do BMD, screen
even there is no C/O

Blood work:
o Serum calcium and phosphate levels
o Alkaline phosphatase
o Creatinine
o SPEP (serum protein electrophoresis)
o PTH (para-thyroid hormone)
o Give vitamin D for 2-3 months, then assess the level, if > 0.75 nanogram  it is normal and
do not repeat it again

Treatment:
- Based on BMD, risk factors, age of patient  Fracture Risk Stratification  low, moderate, or high
LOW
MODERATE
HIGH
- Life style
If fragility fracture (in thoraco-lumbar x-ray) OR prolonged
- Life style
modifications use of corticosteroids
modifications
- F/U DEXA
Medical
NO
YES
after 5 yrs
treatment
- Life style modifications
- Life style modifications
- F/U DEXA after 2 yrs
- Medical treatment
Life style modifications
- Weight-bearing exercises (walking, jogging)
- Ca 1200 mg/d (including the dietary intake,
avoid Ca)
- Vit D 1000 IU/d if < 50 yrs, and 2000 IU/d if >
50 yrs
- Smoking cessation
- alcohol and caffeine

OSCE-guide-III.doc

Medical treatment
- Bisphosphonates
- SERM (Raloxifene): agonistic effect on bone
but antagonistic effect on breast and uterus
- Parathyroid hormone (PTH)
- Calcitonin (if back fragility fracture + pain)
- HRT

Page 176 of 255

OB-GYN

Pediatrics

OSCE-guide-III.doc

Page 177 of 255

OB-GYN

History taking Pediatrics


History taking Pediatrics



Introduction:
Chief complaint

1234567-

Analyze the CC
Impact
Rule out infection
Differential diagnosis
BINDE
Past medical history
Family history

1- Analyze the CC

- BINDE screening
- Mother attitude!
-

2- Impact

3- Red flags
Rule out infection

4- Differential
diagnosis
5- BINDE
6- Past medical history 7- Family history
0-6 years
BINDE
Pregnancy / Birth
Immunization
Nutrition
Developmental
Environmental

OSCE-guide-III.doc

In any pediatrics case: always


CONSIDER child ABUSE / NEGLECT
 report to CAS

TIME: Os Cf D: When did it start? How did it start? Sudden or


gradual?
- At that time, did your baby have any fever, flu-like symptoms?
- Is it continuous or on and off? How often? Day and night?
Character:
- PQRST
- If vomiting or diarrhea: COCA + BLOOD
- Timing: is it related to feeding / meals?
Factors: is it related to position? Meals?
Is he drowsy? Floppy?
Does he cry? Is it high pitched cry?
Did you notice his suckling is weaker than before?
Constitutional symptoms!
R/O infection: Did you notice if your child has fever or skin rash?
Cough / wheezes? Ear pulling or discharge? Runny nose? Foul
smelling urine? Abdominal distension? Diarrhea?
Rule out child neglect
- BINDE screening
- Mother attitude!
Scan for risk factors for child abuse / neglect
Hospitalizations / surgeries / blood transfusion
Illnesses (cancer) / infections
Medications / allergies.
Travel
Family history of similar disease in the family
6-14 years
School performance

14-18 years
HEAD SSS
Home
Education
Activity
Diet
Suicide
Sexual activity
SAD (smoking / alcohol / drugs)

Page 178 of 255

OB-GYN

BINDE
Pregnancy:
- Was your pregnancy planned? If no  social issues
- Were you having regular follow-up visits? How about U/S? Was it normal?
- During your pregnancy, did you have any illness? How about any fever or skin rash? Have
you ever been in contact with sick kids? Kids with skin rash or fever? Have you ever been in
contacts with pets?
- Did you take medications? Even OTC? Did you smoke? Drink alcohol? Have you ever tried
recreational drugs? What about before pregnancy?
- Were you screened for Hepatitis B virus? HIV? other diseases? There is screening test that
we do a vaginal swab at 36 weeks called GBS, did u have it?
- What is your blood group? What is your baby blood group?
Birth:
- Was your baby full term or not?
- Was it a vaginal delivery or c-section?
o If c-section  why? Was there any complication? Abnormality?
o If vaginal  was it difficult labour? Prolonged labour? How many hours? Was
there any early gush of water? How many hours? Did you need any help to make
it easier; e.g. vacuum?
- Did your baby cry immediately or not? Do you know what his Apgar score was? Did he need
special attention? When did you leave the hospital?
- Were there any bruises or swellings on your babys body?
- Were you told that your baby had any special features?
- After delivery, did you have any fever / discharge? Did you take any medications?
Immunization:
- Are your babys shots up-to-date?
o Yes  when was the last shot?
o No  any reason for that?
 Our religion prohibits vaccination: ok, that is fine
 We think vaccines cause autism: correct this info, vaccines are safe
 We were busy  neglect concern  what is baby weight?
Nutrition:
- Weight:
o
o
o
o

Weight calculation:
What is your babys weight today?
What was his weight at birth?
What was his highest weight?
Do you have access to growth charts?
 If below 3rd percentile: underweight
 If (at any time) he crossed (down)
two major lines: failure to thrive

At birth : x
5 months
1 year
2 years

Kg
:2x
:3x
:4x

Kg
Kg
Kg

More than 2 years old:


[(age X 2) + 8] Kg

Even low birth weight, catch up weight later, i.e. @ 1 year


they must be around 10 Kg, not only 3 x

OSCE-guide-III.doc

Page 179 of 255

OB-GYN

Height:
o

o
-

To calculate height:
 At birth
X cm
50 cm
 1 year
1 X cm
+ 25
75 cm
 2 years
1 X cm
+ 12.5
87.5 cm
 3 years
1 7/8 X cm
+ 6.5
94 cm
 4 years
2
X cm
100 cm
For each year: the baby gains () of the previous year increase, so the baby gains
X by the first year, X by the second year, 1/8 X by the third year.

Diet:
o

What do you feed your baby?


 Everything we eat. No restrictions!  that is fine
 Breast feed  if more than 4 months:
Any iron supplement?
Any Vit D supplement?
 Formula:
Since when?  if since birth: any reason that you chose formula
over breast feeding?
Which formula? Any recent change in formula?
For any case of chronic diarrhea?
 Do you give him solid food; biscuits / bread / cereal?
 Which started first? The diarrhea or this new food?
 How many diapers do you change per day? (normally 5 6)

Developmental:
Now I would like to ask you some questions about the kind of activities that your child can do,
and other questions to assess his development.
Gross motor
Fine motor
Sit alone / roll over
6 months
Draw line
15 months
Crawling
9 months
Draw cross
2 years
Standing / cruising
1 year
Draw circle
3 years
Walking
15 months
Draw square
4 years
Go upstairs holding
18 months
Draw triangle
5 years
Go downstairs 2 feet
2 years
Tricycle
3 years
Social
Social smile
Stranger anxiety
Separation anxiety
Says NO

6 weeks
6 months
9 months
2 years

Speech / verbal
Mama / papa
2 words beyond Ma, Pa
2-3 words phrases
Short sentences
Speaks fluently

9 months
1 year
2 years
3 years
5 years

N.B. (autism / Down syndrome / child abuse): there is no stranger or separation anxiety.

OSCE-guide-III.doc

Page 180 of 255

OB-GYN

Environment:
- How do you feel being a new mom? How do you feel about your baby?
o How is your mood? You look down for me, any chance you are being depressed?
Did you have depression before?
- With whom do you live? How is the relation between you?
o How is the relation between you and the baby?
o How is the relation between your partner and the baby?
- How do you support yourself financially?
- Do you live in home (basement: mold) or apartment? Is it an old building (lead)?
- Any other children in the house?
- Do you or any body in the home smoke? Drink? Use recreational drugs?
- Is anyone of your family seeing a psychiatrist? Has mental illness?
- In ABUSE cases: tell me more about your childhood
HEAD SSS
Home:
- With whom do you live?
- How is the relation between you? Are they supportive?
- Any siblings?
Education:
- Do you go to school? Do you like going to school?
- Which grade? Which subjects do you study?
- How about your marks, what marks do you get? What about in the past?
Activity:
- What kind of hobby do you have?
- Have you travelled recently?
- In EPILEPSY case: do you operate machines / drive / go hiking?
Diet:
- How about your diet? What do you eat? Do you follow special diet?
- What is your weight? What was your weight before?
Suicide:
- How is your mood?
- Any chance that you might hurt yourself?
Sexual activity:
- Are you dating? Are you in relationship?
- Are you sexually active? When did you start? When was the last time?
- How many partners do you have? Do you practice safe sex?
Smoking / Alcohol / Drugs:
- Now, I would like to ask you some personal questions, it is important to ask it, and it is
confidential, do you Smoke? Drink Alcohol? Have you ever tried recreational drugs?
- Sometimes people at your age might start to smoke, drink, or use recreational drugs. Do you
know any of your friends doing this? How about you? Have you tried that?
- For IV drugs: When was the last time? Did you share needles?

OSCE-guide-III.doc

Page 181 of 255

OB-GYN

Jaundice
A new born 5 days old, with jaundice since day 2
Introduction
Differential diagnosis of newborn jaundice
- Physiologic (usually days 2-7) 
CC
unconjugated
- Analyze the jaundice (OCD)
- Breast milk jaundice
- Impact / consequences
- Breast feeding jaundice
- Red flags / rule out infection
Pathologic
(anytime)
- DD
- Hemolysis (unconjugated)
- BINDE
- Infection  sepsis (conjugated or
- Birth  pathological
unconjugated)
- Nutrition  physiological
FH
Introduction:
Good morning Mrs , I am Dr , I am the physician in charge today, I understand that you are
here because your son has jaundice (or is yellow). In the next few minutes I will be asking you
some questions to help me figure out the condition, before I proceed, I would like to know the
name of your child? This is a nice name.
1- Analyze the CC:
- When did it start? Early in the 2nd day (or before: pathological) or late (pathological or
physiological)?
- Who noticed it? You or someone else? When? Where did you notice it? How about his eyes?
How about his feet? Is it spreading? Is it or ?
- How about his urine, is it darker? And stools, is it pale?
2- Impact / Consequences:
- Is he drowsy? Floppy?
- Does he cry? Is it high pitched cry?
- Did you notice his suckling is weaker than before?
3- Red flags / Rule out infection:
- Did you notice if your child has fever or skin rash? Cough / wheezes? Discharge from his
ears? Runny nose? Foul smelling urine? Abdominal distension?
- Any night sweats / chills? Any lumps or bumps in his body? Tender points?
4- Differential diagnosis:
Physiological
Pathological
How do you feed him? Breast milk? Formula? Infection should be ruled out or

Breast feeding jaundice: (or lack of


confirmed by now
breastfeeding jaundice): Not enough milk Hemolysis:
 dehydration

What is your blood group? Your

Breast milk jaundice: is more of a


baby blood group? Father blood gp?
biochemical problem (inhibition of

Rh incompatibility  IUGR
bilirubin conjugation leads to increased

Were you screened for infections
levels of bilirubin in the blood).
during pregnancy?
Treatment: substitute with formula

Biliary atresia

Hepatitis: neonatal

OSCE-guide-III.doc

Page 182 of 255

OB-GYN

5- BINDE
Birth:
Nutritional history:
- How do you feed him? Breast milk? Formula?
- Breast:
o How many times do you feed him?
o Do you use 1 breast or both of them? How long each?
o After feeding him, do you feel your breast engorged?
- Formula:
o Any reason to choose formula feeding?
o Which type of formula? Do you know how to prepare it?
Environment:
- Any other children? Did any of them develop jaundice after birth before?
6- PMH?!
7- FH:
- Jaundice
- Liver disease
- Blood disease
- Disease called cystic fibrosis
Diagnostic
workup:

When to
suspect
pathological
jaundice?
Treatment

Hemolytic workup: CBC / blood gp (mother and baby) / peripheral blood


smear / Coombs test / bilirubin (direct and indirect)
Septic workup: CBC / differential / blood & urine cultures / TORCH screen
TSH and G6PD screening
Liver enzymes / bilirubin / and coagulation profile
If in the first day (or early second day) of life
Bilirubin rises > 85 mol/L/day
Bilirubin level > 220 mol/L before 4 days of age
Conjugated (direct) bilirubin > 35 mol/L
Persistent jaundice lasting beyond 1-2 weeks of age
Ensure proper hydration and feeding
If sepsis: treat the underlying infection
Phototherapy: if total bilirubin is > 300 mol/L, and only for unconjugated
hyperbilirubinemia, it is contraindicated in direct hyperbilirubinemia
Exchange transfusion: if total bilirubin is > 400 mol/L

The mother has a concern: will he develop mental retardation?


1. This is a reasonable concern, I am glad you came here today to figure out
2. What made you think about that?
3. In order to be able to answer this, I first need to ask you some questions, examine him
and we may need to do some blood tests. However I would like to inform you that
jaundice in newborns is a common condition, and it is usually physiologic jaundice that
does not lead to mental retardation
Complications of Rh incompatibility: (1) kernicterus (brain damage  seizures), (2) Hydrops
fetalis (generalized edema), (3) Hypoglycemia

OSCE-guide-III.doc

Page 183 of 255

OB-GYN

IUGR
A newborn 3 hours old with IUGR, counsel the mother
Introduction

News
BINDE
Obstetrical history

Mother PMH

Good morning I understand that you just gave birth, my colleagues are
taking care of your baby. And I would like to ask you some questions
regarding your child health, but first tell me;
- How do feel right now?
- Have you seen the baby?
- Did you pick a name?
Your baby has just been diagnosed with a condition called intra-uterine
growth retardation or low birth weight For that reason; I would like
to ask some questions about your pregnancy!
-

GTPAL
Were you pregnant before? How many times? Any abortions?
Miscarriages?
Any history of chemo therapy or exposure to radiation
Any family history with congenital anomalies

+ The mother has a concern: the baby is green


 This means he passed meconium (baby stool) which means the baby had a stressful delivery
+ The mother has a concern: is it my mistake?
 This condition is related to multiple factors, some factors are related to pregnancy, and others
are related to the baby or the environment. On the other hand, there are NO safe limits for
smoking / drinking alcohol during pregnancy, so, in the future, if you decide to become pregnant,
it is important NOT to smoke / drink alcohol / use recreational drugs during your pregnancy
Possible causes:

Smoking / alcohol / cocaine during pregnancy, (cocaine during pregnancy 


microcephaly, IUGR, MR)

TORCH infection,

Extreme of age, esp. advanced age pregnancy


Risks for the next 48 hours: asphyxia / hypoglycemia

OSCE-guide-III.doc

Page 184 of 255

OB-GYN

Crying Baby
Introduction
CC
Analysis of CC

OCD / all the time / day and night?


Is he crying > 3 hrs/day for > 3 days/week for > 3 weeks

What initiates or increases the crying?


o Any chance the baby is hungry? What do you feed him?
o Any chance that he is too hot / too cold? Do you adjust the
temperature?
o Any chance that he is wet? How often do you change his diapers
daily? Is there any skin or diapers rash?

What improves or decreases the crying? When he cries, what do you do?
o Did you try to hug / hold / burp / sooth / play music / give him a
walk?
o Did you try to rock him? Shake him? What happened to him?

When he cries, does he pull his legs? Is he passing gases? Is his abdomen
distended? Is it related to feeding? How are you coping with this?

How does this affect your life? And your partner life? Are you able to go
to work?

Is he drowsy? Floppy?

Did you notice if your child has fever or skin rash? Cough / wheezes?
Discharge from his ears? Runny nose? Foul smelling urine? Abdominal
distension? Diarrhea?

Any infection (there will be other symptoms)  review of systems

Infantile colics (crying > 3 hrs/day for > 3 days/week for > 3 weeks),
between the age of 3 weeks and 3 months, without another explanation
 reassure

Child neglect

Feeding problems: overfeeding / hungry


Scan for risk factors for child abuse
Nutritional

How do you feed him? Breast milk? Formula?

What about his weight?


Environmental With whom do you live? How is the relation?

How do you support yourself financially? Do you get


enough support?

Any other kids? Any repeated visits to the ER?

Mental problem

Parent SAD

Impact

Red flags / R/O


infection
DD

BINDE

FH

Investigations (not including those for suspicious child abuse): CBC / urinalysis / stool analysis

OSCE-guide-III.doc

Page 185 of 255

OB-GYN

Chronic Cough Asthma


Child complains of cough for 6 weeks, post-pneumonia, father wants to renew antibiotics
Here, we have two issues:

The productive cough & fever 6 weeks ago  pneumonia

The intermittent / dry cough that presented after
CC
HPI

Analysis of the
CC
Cough

Impact
Red flags

Cough

Os Cf D /+/ COCA + B + Phlegm

Certain time of the day? Night?
Acute phase
Chronic phase
Continuous / productive /
Intermittent / dry cough / on and off /
fever / loss of appetite
no fever

Seen by a doctor? What  SOB, noisy breathing, wheezes,
diagnosis? Treatment?
chest tightness, nausea / vomiting

Anti-biotic history!

Does he cough to the extent of
vomiting or LOC

Did you renew it? From
the same doctor? Was

Pertussis vaccination?
he examined? Any xrays were done?
How did this affect his life? Daily activity?
Constitutional symptoms
Triggers of Asthma: any thing that this cough?

Chronic diarrhea  cystic fibrosis

Any allergy
Brief
Other allergic diseases: atopic dermatitis / allergic rhinitis
Allergic diseases: asthma / skin allergies



Differential diagnosis
BINDE
PMH
FH
Triggers

Infection

Medications 


Outdoor

Indoor

Stress

OSCE-guide-III.doc













Recent chest infection? Flu-like symptoms? Fever / chills?


How do you use puffers? Stored properly? Not expired?
Did you start new medication? -blockers? Aspirin? Any recent
in dose of these medications?
Exercise
Cold air
Pollens (is it seasonal?)
Dust: construction / smug (smoke/ fog/ exhaust)
Do you smoke? Anybody around you?
Do you have pets? People around you?
Fabrics related: carpets floor? Any change in linen? Pillows?
Blankets? Mattress? Curtains?
Relation to any type of food?
+ Perfumes
Do you live in a house (basement  mold)?
Any construction renovation? Exposure to chemicals?
Any new stressful situations?

Page 186 of 255

OB-GYN

Questions:
Diagnosis:
Investigations:
Treatment:

hyper-reactive airways disease


x-ray
steroids puffer for 4 weeks

Counselling:

The most likely explanation for that is a condition called: hyper-reactive airways disease.
It is a term used to describe asthma-like symptoms in infants (< 6 years old) that may
later be confirmed to be asthma when they become old enough to participate in asthma
tests (spirometry and bronchodilators).

This is a common problem, and is usually triggered by infection (acute bronchitis or


pneumonia), it may last up to 10 weeks after infection.

It may be self limited; however, we need to start treatment with puffer (steroids puffer
for 4 weeks).

When the child becomes older than 6 years, and if the condition is still persistent for
more than 10 weeks, we send the child for investigations (spirometry and
bronchodilators) to confirm the diagnosis of bronchial asthma.

If this condition happens in adults, we treat with puffer for 4 weeks, if no improvement;
we send to investigate for asthma (spirometry and bronchodilators then metacholine
challenge test).

Case: 9 weeks history of cough on Amoxil for 2 weeks

DD: asthma / bronchitis / cystic fibrosis / recurrent pneumonia

Investigations: CXR / CBC, differential, lytes / sweat chloride test

OSCE-guide-III.doc

Page 187 of 255

OB-GYN

Anemia
6-9 months, mother complains he is pale?
1- Analyze the CC
- Clarify CC: What do you mean he is pale? Is he yellow?
- Os Cf D
- Who noticed it? You or someone else? Is there any chance that
he had this pallor before and you were not aware of it?
2- Impact
- Is he drowsy? Floppy?
- Does he cry? Is it high pitched cry?
- Did you notice his suckling is weaker than before?
Signs of
- Is he active / playful like before? What can he do? Is he crawling?
anemia
- If he is doing activity, did you notice any SOB? Fainting?
- Is he gaining weight?
3- Red flags: rule out
- Constitutional symptoms!
infection
- Did you notice if your child has fever or skin rash? Cough /
wheezes? Ear pulling or discharge? Runny nose? Foul smelling
urine? Abdominal distension? Diarrhea?
4- Diff diagnosis:
- Rule out child neglect
- Bleeding disorders: nose / gums / coughing / vomiting / bruises

Iron def. anemia
on body / blood in urine / stools / joint swelling

Thalassemia
Leukemia: Constitutional symptoms / Bone pain [if he walks,

Hemolytic disorders
does
he limp? if you carry him, does he complain of tender

Bleeding disorders
points
in his body] / cough / repeated infection

Chronic diseases
Lead intoxication
Leukemia
5- BINDE



6- Past medical history

7- Family history

Scan for risk factors for child abuse / neglect


N: What are you feeding him? Breast milk? From the beginning?
Do you give him any iron supplements or iron fortified cereals?
B: was he term or not?
E: with whom do you live? How do you support yourself
financially?  offer social support
Where do you live, if old place, have you ever seen him eating
the paint scales?
Any heart / lung / kidney / liver disease?
Hospitalizations / surgeries / illnesses (cancer) / infections
Medications (Sulpha drugs G6PD deficiency) / allergies
Travel
Family history of similar disease in the family
Any bleeding disorder
Any repeated surgeries? (cholecystectomy / splenectomy)
Ethnicity: some blood diseases are more common in certain parts
of the world, that is why I need to ask you about your
background, what about your partner?
Are you related by blood to your partner?

Investigations: lab works; CBC / differential / lytes / serum iron studies (ferritin, TIBC) /
hemoglobin electrophoresis / KFTs / INR / PTT
Treatment: iron supplement

OSCE-guide-III.doc

Page 188 of 255

OB-GYN

Vomiting
The mother of (6 weeks 3 months) old baby came to the clinic complaining of childs repeated
vomiting.
Introduction
Chief complaint
1- Analyze
the CC

2- Impact

3- Red flags -

4- DD

When did it start? How did it start? Sudden or gradual?


At that time, did your baby have any fever, flu-like symptoms?
Is it continuous or on and off? How often? Day and night?
COCA + BLOOD: what do you feed him? Is he vomiting the entire amount? Is
it watery or curdy?
Character: is the vomiting forcible? Projectile?
Timing: is it related to feeding / meals?
After vomiting, do you fell he is still hungry? General condition after vomiting
Factors: is it related to position? When lying down?
Other GIT symptoms: diarrhea
Is he drowsy? Floppy?
Does he cry? Is it high pitched cry?
Did you notice his suckling is weaker than before?
Dehydration: do you feel his lips dry / skin dry? Does he tear? How many
diapers
Failure to thrive: what about his weight, do you know his weight? What was
his weight at birth? Do you have access to his growth charts?
Constitutional symptoms!
R/O infection: Did you notice if your child has fever or skin rash? Cough /
wheezes? Ear pulling or discharge? Runny nose? Foul smelling urine?
Abdominal distension? Diarrhea?

Differential diagnosis of 6 weeks vomiting


Rule out child neglect
- Pyloric stenosis
-

Any infection / meningitis


Brain tumour

GERD
Wrong formula OR not preparing
it well
Overfeeding OR NOT burping

OSCE-guide-III.doc

BINDE screening
Mother attitude!
Family history of pyloric stenosis
Appear at age of 2 4 weeks
Projectile / non-bilious / baby still
hungry after feeds
Continuous
Other symptoms / neurological:
weakness / neck stiffness / seizures
After feeds
Not all feeds
No in weight

Page 189 of 255

OB-GYN

5- BINDE

Scan for risk factors for child abuse / neglect


Nutritional history:
- How do you feed him? Breast milk? Formula? Which type? Do you know how
to prepare it properly? Any solid food?
- Which happened first, the vomiting or switching to the new formula?
- Weight analysis!
Environment:
- How do you feel being a new mom? How do you feel about your baby?
- How is your mood? You look down for me, any chance you are being
depressed? Have you ever been depressed before?
- With whom do you live? How is the relation?
- How do you support yourself financially?
- Do you get enough support?

6- PMH
7- FH
Pyloric stenosis DD: duodenal atresia / tracheo-esophageal fistula
Management plan:
- Investigations: lab works (CBC, lytes, ABG) / US
- If dehydrated: admission
- If suspicious child neglect: contact CAS
Potential risk factors for child abuse:
- SAD parents (smoking / alcohol / drug use)
- Pregnancy not planned
- Preterm baby
- Congenital anomalies
- Baby who needed special attention after delivery
- Separation from the child
- Difficult child
- Young couple
- Parents with history of abuse
- Stress or financial difficulties in the family
Investigations for child neglect
- Full blood work / CBC / albumin level
- Fundoscopy
- Skeletal survey

OSCE-guide-III.doc

CONTACT CAS

Page 190 of 255

OB-GYN

Diarrhea
Diarrhea
Failure to thrive FTT

What about his/her appetite?

What other associated symptoms? (Respiratory / Gluten)
Cystic fibrosis
Celiac disease
Milk protein
HIV
allergy
From cow milk
- Good appetite - Poor appetite
- Respiratory
- Gluten
Should not be
given < 1 year

NO FTT
Toddlers diarrhea
Infections
Lactase Deficiency
(lactose intolerance)

A 50 years old father comes with 9 months child with 6 weeks of diarrhea (CHRONIC)
1- Analyze the - Os Cf D
CC
- COCA + BLOOD + others:
- Watery / loose / bulky
- Any undigested food
- Difficult to wipe?
- Factors: Juice (Excess fruit juice)
- Identify FTT weight: What is weight today? At birth? Last visit? The
highest weight? Not gaining weight?
- Other GIT symptoms: vomiting
- APPETITE
2- Impact
- Is he drowsy? Floppy?
- Does he cry? Is it high pitched cry?
- Did you notice his suckling is weaker than before?
- Dehydration: do you feel his lips / skin dry? Does he tear? How many
diapers
- Failure to thrive: what about his weight, do you know his weight? What
was his weight at birth? Do you have access to his growth charts?
- Long period  malabsorption  anemia and rickets
3- Red flags:
- Constitutional symptoms!
(R/O infection) - Did you notice if your child has fever or skin rash? Cough / wheezes? Ear
pulling or discharge? Runny nose? Foul smelling urine? Abdominal
distension? Diarrhea?
4- Differential
DD for ACUTE diarrhea:
diagnosis
- Use of antibiotics
- Infectious:
- Camping / travelling
- Any body else at home with diarrhea?
- Does he go to day care?
DD for CHRONIC diarrhea without failure to thrive:
- Toddlers diarrhea: does he drink too much juice daily?
- Infectious parasitic / travellers diarrhea
- Lactose intolerance:
- Does he pass a lot gas?
- Does he have any redness / skin rash at his buttocks?

OSCE-guide-III.doc

Page 191 of 255

OB-GYN

5- BINDE

DD for CHRONIC diarrhea WITH failure to thrive:


- HIV (if susceptible): was he screened for HIV
- Milk protein allergy: cow milk given before 1 year
- Cystic fibrosis
- Celiac disease
Cystic fibrosis
Celiac disease
- Good appetite
- Poor appetite
- Repeated chest infections? Cough? - bulky foul-smelling stools
- When did he poo the first time?
- What do you feed your son? How
- Did he have yellow discoloration
many times?
after birth? For how long?
- Do you know how to prepare the
- Does he have any bulging through
formula? Was it changed
his buttocks?
recently?
- Did you start to give him solid
food? What type of food? How
about bread, cereals, biscuits?
Which started first, the diarrhea or
the new food?
- After you feed him, does he have
abdominal distension? Gases?
Pulling his legs?
- Does he have flat buttocks? Thin
legs and arms? Distended
abdomen?
- Scan for risk factors for child abuse / neglect
- Nutritional history
- Weight analysis

6- PMH
7- FH

Questions:
- What is your differential diagnosis:
o Cystic fibrosis
o Celiac disease
- If the biological mother called, want to know about her son, do you tell her or no?
o In order to determine whether I should release any information or no, I would
like first to know who has the legal custody (guardian) of this child. It might be
the adopting father, a social worker (case manager)
Notes:
- If the child was adopted, and you are speaking with one of the new parents:
o Are you the biological mother/father?
o Is this adoption or foster home?
o When was the child adopted? At which age? From where?
o What were the circumstances?
o Do you have information about the biological parents?
o Was he screened for HIV?

OSCE-guide-III.doc

Page 192 of 255

OB-GYN

Mother worried about her child weight


- Not gaining weight
- Not eating well
1- Analyze the CC

Weight analysis:
- Weight today, birth, last visit, highest
- When did you start to worry about that? Why?
- Who noticed it?
- Who is the primary care giver? For how long have you been with him?
- Do you have his growth chart? If no: if you do not mind, I need to
contact his family physician to take a look at his charts
Height analysis:
- Height today, birth, last visit

2- Impact

3- Rule out any


serious condition:
chronic inf/ dis /
malignancy
4- Differential
diagnosis

5- BINDE

6- PMH
7- FH

Is he drowsy? Floppy?
Is he playful? Active like before? Any limitations? Does he turn blue
with activity?
Constitutional symptoms!
Any congenital or long term disease?
Review of systems: cardiac/ chest/ GIT/ urinary/ MSK/ skin/ allergy/
pale/ bleeding
Rule out child neglect
Difficulty swallowing (CP, Cleft Palate)
Chronic loss: chronic Diarrhea (Celiac disease, CF, pancreatic
insufficiency) / chronic vomiting (pyloric stenosis in a younger child)
Diabetes mellitus (drinks too much water, pees a lot, tired)
Chromosomal abnormalities / inborn error of metabolism
Scan for risk factors of potential child abuse / neglect
Apgar score at birth
Diet in details: breast feeding/ formula/ cow milk? Any reason? For
how long? Any supplements? Any solid food?
Developmental milestones
Environment: with whom do you live? Who takes care of the baby? Is
he/she capable of doing this?
SAD during pregnancy and now

Under weight:
- Failure to thrive (FTT): weight decreases first then height will be affected later
- Endocrine causes: fat and short
- Congenital: everything is small / short, thin with small head
Failure to thrive
- Weight < 3rd percentile or falls across 2 majors percentiles
- Most common cause is inadequate intake

OSCE-guide-III.doc

Page 193 of 255

OB-GYN

Case: A 2 years old boy does not want to eat. The father carries a bag!

History:
o When you ask about the bag, he says it is for the boy lunch, it is full of candy and a coke.
o Details about breakfast, lunch, dinner and snacks
o Review of systems  will be negative

Plan; to improve the health and diet education of the father:


o Educate about nutritious food and supplementation
o Give brochures and Canada dietary guidelines
o Refer to dietician specialist or nurse
Recommend supplementation:
o Iron
o Multi-vitamins
Examine the baby
Blood works: CBC / lytes / serum iron studies (ferritin, TIBC)
Differential diagnosis:
o Stresses int the family
o Child abuse / neglect
o Failure to thrive

Case: A 6 years old developed severe allergy to peanut, child is now stabilized, counsel the
father.

Is it first time to eat peanuts? Any similar reaction before? Any known food allergy?

Review of systems  will be negative

Past history of asthma, allergic rhinitis, allergic skin reaction?


Family history of allergy? Asthma, allergic rhinitis, allergic skin reaction?
Other siblings with allergic reactions?

Management:

Will send the boy for allergic testing

Strict avoidance of allergens

Epi-pen

OSCE-guide-III.doc

Page 194 of 255

OB-GYN

Fever
Introduction
CC
1- Analyze the CC

2- Impact

3- Red flags

SKIN RASH

4- Differential
diagnosis:
Review of systems

FEVER
- Os Cf D
- Any flu at that time?
- Any diurnal variation? More at morning or night?
- Any special pattern? More every 2nd or 3rd day?
- Do you measure it? How many times daily? How do you measure
it?
- Did you try to give any medications to help? Did it help?
- Is it the first time?
- Other constitutional symptoms
- Other persons at home with the same symptoms?
- Is he drowsy? Floppy?
- Does he cry? Is it high pitched cry?
- Did you notice his suckling is weaker than before?
- The fever and constitutional symptoms are already analyzed
- Review of systems: DD
- Is he tired?
- Did you notice any skin rash?

OCD / distribution / color / do you feel it elevated?

Are his shots up-to-date?
- Buttocks / abdomen  henoch schonlein purpura /
Investigations: urinalysis Treatment: steroids
- Trunk  vesiculo-papular  chickenpox
- Face: measles / rubella
- Cheek: fifth disease
- Headache / drowsy / neck pain / rigidity / nausea / vomiting? Does
he recognize you? Talk to you?
- ENT
- Chest: cough / phlegm / SOB / wheezes
- Abdomen (pain, distension, diarrhea) / liver (yellow color, itching,
dark urine, pale stools) / urinary (urine changes, crying while
peeing, loin pain)
- Joints: pain / swelling / mouth ulcers
- Scan for risk factors for child abuse / neglect

5- BINDE
6- Past medical history
7- Family history

OSCE-guide-III.doc

Page 195 of 255

OB-GYN

Runny Nose / Flu / URTI


8-15 years child is coming to see you with his mom, c/o: runny nose / flu / URTI?
Introduction
- To BOTH the mother and the child
- During the encounter, distribute the questions and interaction between
both the mother and the child
1- Analyze the CC
- Os Cf D
- COCA
- What or
- Is this the first time? Or did it happen before?
2- Impact
- Is he playful? Active like before? Any limitations?
3- Red flags
- Constitutional symptoms
Review of systems:
- Rule out infection: Any recent flu-like symptoms? Do you feel tired/
fatigue? History of sinusitis / Pain in your face? Any sneezing? Red
eyes? Pain/discharge in ears? Any sore throat/ oral ulcers/ tooth pain?
Pale / bleeding
- R/O meningitis: Neck stiffness / pain? Headache? N/V?
- Cardiac / chest / GIT / urinary / MSK / allergy
- Skin rash
4- DD

5- BINDE

6- PMH
7- FH

Physical exam

OSCE-guide-III.doc

Allergic rhinitis: runny nose related to seasons, recurrent, no fever


Viral flu: respiratory symptoms / joints & muscles ache
Viral common cold
Scan for the risk factors of potential abuse
Immunization
School performance
Any congenital or long term disease?
Other members in the family with symptoms?
School contacts?

Mouth
ENT
LNs
Chest exam

Page 196 of 255

OB-GYN

Rash
Clinical Presentation
Fever, headache, parotitis (bilateral; pushes earlobes up and out), myalgia,
malaise
Measles
Appearance: erythematous maculo-papular rash; Koplik spots
Timing: 10-14 days incubation, rash 3 days after start of symptoms
Distribution: starts at hairline, spreading downwards; palms and soles
typically not involved
Rubella
Appearance: pink, maculo-papular rash.
Timing: 14-21 day incubation; rash 1-5 days after start of symptoms.
Distribution: starts on face spreading to neck and trunk.
Chickenpox
Appearance: macules  papules  vesicles  crusting; all stages apparent
(varicella)
at once (polymorphous rash)  very pruritic
Timing:

10-21 days incubation;

1-3 days prodrome: (fever and respiratory symptoms),

Then rash
Distribution: face, trunk, extremities, mucosa, palms and soles.
Erythema
Appearance: uniform, erythematous maculo-papular rash
Infectiosum
Timing: 4-14 days incubation, rash 10-17 days after symptoms
(fifth disease) Distribution: bilateral cheeks with circum-oral sparing, can affect trunk
Roseola
Appearance: pink maculo-papular rash (faint).
Timing: 5-15 days incubation; rash 3-5 days after symptoms.
Distribution: starts at neck and trunk spreading to face and extremities
Management: rest / anti-pyretics / fluids / good nutrition
Mumps

When does the kid go back to school?


Infectivity: 7 days pre-parotitis to 7 days
post-parotitis
No rash, no fever (infectivity: 4 days prerash)
Infectivity: 7 days pre-rash to 5 days postrash
Infectivity: 1-2 days pre-rash until vesicles
have crusted over

Infectivity: prior to onset of rash

Reye Syndrome:

Acute hepatic encephalopathy and non-inflammatory fatty infiltration of liver and kidney

Mitochondrial injury of unknown etiology results in reduction of hepatic mitochondrial enzymes, diagnosis by liver biopsy

Associated with aspirin ingestion by children with varicella or influenza infection.

40% mortality

OSCE-guide-III.doc

Page 197 of 255

Pediatrics

Delayed Speech
Introduction
Verbal assessment
Rule out any serious condition

BINDE
PMH
FH

Clarification: is it not gaining words, losing words OR


not speaking at all?
-

Hearing loss
Autism
Anatomical: tie tongue / cleft palate

- History of meningitis / jaundice


- Family history of deafness or hearing loss

VERBAL ASSESSMENT
- Would you please tell me more about that!
- When did you start to have concerns? Did you seek medical attention before?
- Is the child able to speak at all? How many words is your child capable of using? When
did he start to say it? Can he use many words in one sentence?
- Was he able to use more words (talk better) and lost them?
- How can he communicate with you? What does he do if he wants something?
I would like to ask you some questions in order to reach to the cause of this condition:
HEARING:
- How do you describe his hearing? Does he have hearing difficulties?
- If you call him, would he respond and reply? What if you are behind him? What if you
are in another room?
- Did you notice that he keep increasing the volume of the TV?
- Did he get repeated ear infections? Fluids in the ears? Discharge?
- Did he take any medications? Any antibiotics (aminoglycosides)?
- Was he ever screened for hearing test, when he was born?
AUTISM:
- Does he maintain eye contact? Does he show emotions?
- Is he aggressive? Does he play with other kids?
- Does he do repeated movements like rocking, or head banging?
- Does he have a favourite toy? How does he play with it? (train / spinning wheels)
- Any family history of autism?
BINDE:
- Start with the development: to rule out MR
Developmental (mile stones):
- What can he do? When did he start to sit? Crawl? Stand? Walk? Climb stairs?
- As a child, did strangers make him nervous?
- Does he control his urine / bowel movements?
Environment:
- Screen for neglect: how many hours you spend with him? Is he a difficult child?
- Family factor: how many languages do parents and other family speak at home?
Pregnancy / Birth:
- Did you have skin rash during pregnancy? TORCH infection? SAD during preg?
- Was it complicated labour? Apgar score?
- Did he have any special features? Congenital malformations? Cleft palate?

OSCE-guide-III.doc

Page 198 of 255

Pediatrics

Seizing child counselling


Refer to the seizing child phone call case in the emergency medicine section for analysis of the
event
History

Analyze the event


Fever
Rule out
BINDE
PMH
FH

Counselling

During / before / after


Analyze the fever
Rule out meningitis / pneumonia

Febrile seizure / epilepsy


Introduction
Febrile seizures

Introduction:

Based on what you have told me, the most likely explanation of your child seizures is a
medical condition we call benign febrile seizure

What do you know about febrile seizures? Do you want me to clarify some information
about it? In details?
Febrile seizures:

This condition usually affects kids 6 months to 6 years, it is not uncommon, and a lot of
children (around 3%) might have attacks.

We do not know exactly the reason for it, but it is related to fever and may be because the
children brain is not fully developed at that age, and can not tolerate high fevers.

Usually it is self-limited, benign, typical attack is less than 15 minutes, and will not recur in 24
hours. Most children will outgrow their condition after the age of 6 years.

Another attack(s):

From the studies we know it might happen again; for each 100 child who got 1
febrile seizure attack:
o 65 children will not have it again
o 30 children will have another attack
o 3 children will have many other attacks even without fever
o 2 children will develop seizure disorder

The best treatment for it is the prevention that is why it is important to make sure that
whenever he gets a fever, to seek medical attention and to decrease the fever ASAP
using Tylenol or cold foments. Then find the source of fever and treat.

In case it happens again:


Turn the child on his side / protect him from hitting any nearby object / do not
force objects into his mouth
Bring to ER if seizure does not stop within 15 minutes
Diazepam 5 mg PR suppository
If repeated attacks, we may consider prophylactic anti-convulsion therapy
Will do CT, EEG

I will give you some brochures and web sites in case you want more information.

Any other questions or concerns.

OSCE-guide-III.doc

Page 199 of 255

Pediatrics

ADHD counselling
The father comes to you saying that his son was diagnosed with ADHD two days ago and he has
concerns about ADHD and Ritalin. Counsel for 10 minutes.
Introduction
To diagnose ADHD:
- 2 settings (school / home)
Address concerns
- > 6 months duration

Diagnosis (symptoms of ADHD)
- < 7 years old child

Impact
Differential diagnosis:

Differential diagnosis
- ODD /+/ Conduct disorder
BINDE
- Specific learning disability
PMH
- Seizures (petit-mal epilepsy)
FH  ADHD / MR / autism / depression
- Depression
Conclusion
Introduction:
- Who diagnosed it? Usually teachers recognize it first (pick it), but to make a diagnosis a
psychiatrist, paediatrician, or a specialized nurse assessment is needed
Concern do you give Ritalin (which is amphetamine) to children?
- Actually yes. A lot of children use Ritalin, it is the first line of treatment for ADHD, and it is
effective and has been used for long time.
- It is not exactly amphetamine, it is the same family, it is called methylphenidate and it is
approved for this indication.
- In children, it helps them to focus as increases their concentration and channels their energy,
this is crucial for children, as it allows them to do better in schools. Even though we might
not be able to cure all children with ADHD, by we try to help them with education, so that
they can have career and live independently in the future, without problems with the law.
- It is generally a safe medication in children. It is not addictive, and we can stop it at any time.
However, like any other medication, it has its side effects, that include: insomnia (that is why
we give it early), abdominal pain, and not all children improve on it.
Before talking further about ADHD and Ritalin, let me first ask you some questions to see if your
child meets the criteria of ADHD or any other developmental challenge:
Diagnosis (hyperactive / inattentive / impulsive):
- Did the teachers complain that your child is full of energy? Spinning all the time? Refuse to
stand still? Talk all the time? Answers even if he is not asked? Does he stand in-line or does
he break the queues?
- Can he focus on one subject for > 30 minutes? Can he finish his tasks (e.g. the homework)?
Does he jump from one activity to another without finishing it? Does he lose his stuff? Does
he forget his belongings?
- Is this only at school or also at home?
- Did you notice that yourself?
- How much time do you spend with him? How about the mother, is she involved?
- How about before? Did anyone mention that or no?
IMPACT:
- Impact on functioning, school performance, relationship with peers

OSCE-guide-III.doc

Page 200 of 255

Pediatrics

Differential diagnosis:
ODD
Conduct
disorder
Learning
disability
Petit-mal
epilepsy
Depression
Autism
MR
-

Does he like not to follow the instructions?


Does he like to challenge his teachers and other family members?
Is he aggressive? Does he fight a lot with other children?
Does he have a pet? How does he treat his pet / or other pets?
Did you notice that he takes others belongings without telling them?
Does he tell the truth all the time?
Does he like to set fires?
Does he like to go to school?
Does he have specific difficulty in reading / writing / mathematics?
Does he have a history of seizures?
LOC? Abnormal movements?
Was he stressed recently? Any loss of a beloved one?
Is he sad? Crying? Nightmares? Losing weight?

BINDE: detailed developmental history


Conclusion:
- I am really sorry for this loss; it must be difficult for children in his age to go through all
that. How is he/she coping with that?
He has NO symptoms OR does not fulfill the criteria:
- Based on what you have told me, and the fact that it happened > 8 years old, it is less
likely that he has ADHD.
- On the other hand, his symptoms and behavioural changes could be related to the loss of
his mother, which might lead to depression. And in his age group, depression may
manifest as behavioural changes like what he is experiencing right now.
- We can refer your child to a child psychiatrist who can help him deal with that.
He has symptoms:
- Based on what you have told me, your child symptoms meet the criteria for diagnosis
with ADHD. However, this is not uncommon condition, and there is medical treatment
for it, in which the first line is Ritalin.
- Counsel on Ritalin.
Notes:
- Whenever you hear that one of the parents has passed away  show empathy.
I am sorry to hear that, it must be difficult for children in his age to go through all that.
How is he/she coping with that? How are you coping?

OSCE-guide-III.doc

Page 201 of 255

Pediatrics

Vaccination counselling
New comer to Canada, comes to you as she has some concerns about vaccinations
Introduction / welcome her / how do you feel?
Speak with enthusiasm (to
encourage) with three
Identify the language barrier
counselling sessions:
Identify concerns
- Pap smear
- Deal with concerns one by one
- Breast feeding
- Pose frequently and ask if she has any questions
- Vaccination
Candidacy for vaccination
Mother vaccination
Otherwise, speak neutrally

What are vaccines?

How do we vaccine?

Side effects of vaccines


Introduction / welcome her / how do you feel?
- Good evening Mrs vich, my name is Dr I understand that you are a new comer to Canada,
and you came to the clinic because you have some concerns about vaccinations. We will discuss
all you concerns. First of all, welcome to Canada, for how long have you been here? How do you
feel being here?
Identify the language barrier
- Before we proceed, am I clear, or do I need to talk slower? We can arrange for an interpreter or a
family member, if you would like to.
Identify concerns
- Now, can you tell me more about your concerns?
- Do you need general information, or do you have specific concerns?
o I heard that vaccines cause autism!
o I think we do not have these diseases in Canada, why should we give the vaccines for
diseases not common here?
Thanks for coming here to discuss your concerns with me.
Vaccines cause autism!
- What gave you this feeling? Concerns?
- There is misinformation among the general population that there is a connection between vaccines
and autism. And the origin of this misinformation is a study done in England many years ago, the
study found there is a connection between autism and 1 type of vaccines; namely the MMR.
- And because we take vaccines very seriously, a much larger study was done, in large number of
countries, including very large number of children. Now we found for sure that there is no
connection between vaccines and autism. The only relation is a coincidence between the age in
which parents start to notice autism symptoms and the age we start to give MMR.
- When we tried to figure out why the first study found the connection, the explanation of that was a
bias in the selection by the author and the study was conducted to favour this outcome. Another
theory to explain the connection was the preservative used in the vaccine (Thiomersal) and it
contains mercury. However Canadian vaccines do not contain it.
- I can assure you that there is no connection between vaccines and autism. Any questions till now?
Why to vaccine against diseases not preset in Canada?
- Even though we do not have these diseases commonly in Canada, and this is because we have a
successful vaccination program. But because the world is a becoming more and more a small
village, and people travel easily from a place to another, we do not want your child to be
vulnerable to preventable diseases if there is any outbreak anywhere. And I will give you some
examples:
o In England, people stopped to vaccinate their children for MMR, they started to have
measles infections again, and some children ended up losing their lives.
o Diphtheria is also coming back because of lack of vaccinations, even in many European
and developed countries.

OSCE-guide-III.doc

Page 202 of 255

Pediatrics

Tetanus is universal; it is in the soil everywhere, if a child is injured, the injury is


vulnerable to be contaminated with tetanus, which has serious fatal consequences.
Do you have any questions or any more concerns?
o

Candidacy for vaccination / history BINDE:


Before I proceed further, I would like to make sure that your child is a good candidate for vaccination,
and we usually start vaccination at 8 weeks.
- Does your child have any illness, sickness, fever, flu-like symptoms? If high fever, we wait. Did
he get recurrent infections before? (if not very young)
- Was he diagnosed with any neurological diseases?
- Was he scanned for HIV? Were you scanned for HIV? Is there any risk that you might have HIV?
- Does he have any allergy?
Based on what you told me, your child is a candidate for vaccination.
- Is it ok till now?
Mother vaccination:
- As a child, were you vaccinated? How do you feel about that?
- If it is ok with you, we can set up a follow-up meeting to discuss in details your vaccination status
and find what vaccination(s) you might need to take.
What are vaccines?
- Vaccines are one of the most significant preventive interventions in medicine. It helped to save the
lives of large number of children all over the world. As children are vulnerable to be infected with
many infections, even thought it might affect adults, but usually with no serious consequences as
compared to children!
- We have large number of diseases and infections, caused by large number of bugs; some of them
cause serious illnesses. We try to protect against many of it. We take the different bugs (bacteria
and viruses), and we either kill it or we make it so weak so it will not cause any harm, then we
inject it into our bodies by a needle. Usually the body reacts by forming anti-bodies which are
protective chemicals against these bugs. Later in life, if the child will get exposed to the real bugs,
these anti-bodies will protect him.
- Was I clear? Any questions?
How do we vaccine?
- Because we have large number of diseases, and each one needs more than one shot, we try to
decrease the number of shots to be given.
- For example, there is a shot called pentacel that stimulates the body to produce protective
antibodies against 5 diseases, it protects against diphtheria, tetanus, polio, pertussis and
hemophilus influenza B. It is to be given at 2, 4, 6, 12 and 18 months and another booster dose at
around 5 years.
- You do not need to worry about memorizing them; I will give you a schedule of the required
immunizations to follow with his family doctor. And these vaccines are covered by OHIP.
- Any questions or concerns do you have?
Side effects of vaccines
- Vaccines are generally safe, millions and millions of children were and are vaccinated and it saved
the lives of large number of them.
- However, like any other medical intervention, vaccines have some side effects.
- It may cause minor issues, like pain, redness and swelling, at the site of injection, it may cause
fever, which we deal with Tylenol for pediatrics.
- There are some few rare side effects that include prolonged crying, seizure, and some children
become floppy, again, this is rare, but if this happens, you need to seek medical attention
immediately.
- Lastly, very rarely, children experience a serious allergic reaction, and that is why we keep the
children under medical supervision for 20 minutes after vaccination.
I hope that I answered you concerns, and now you have a better idea about vaccinations. Any other ideas or
concerns you need to talk about?
Thanks for coming and see you next visit for the first vaccination shot.

OSCE-guide-III.doc

Page 203 of 255

Pediatrics

Child with DM counselling


Father of 6 years old boy comes to you as he has some concerns about his diabetic son
History

Counselling

Complications
Management

When was the child diagnosed? How?


What type of insulin?
Is DM controlled? Regular measure of blood glucose? Hb A1c!
Symptoms of DM
Symptoms of DKA
Symptoms of complications: nephropathy / retinopathy / neuropathy
(usually develops more than 5 years after diagnosis)
Do you have special concerns?
What do you know about diabetes? Did you read? Hear? Anything?
Type 1 DM
Insulin
DKA /+/ Hypoglycemia /+/ Prevention of complications
Macro and micro vascular complications
Meal plan, education, exercise, psychological support
Insulin injections with BS monitoring
Children are more prone to brain damage with hypoglycaemia, therefore
high target range in younger and tight control in older kids

Diabetic ketoacidosis (DKA):


- Diabetic ketoacidosis (DKA) is an acute, major, life-threatening complication of diabetes.
DKA mainly occurs in patients with type 1 diabetes, but it is not uncommon in some
patients with type 2 diabetes.
- DKA is a state of absolute or relative insulin deficiency aggravated by ensuing
hyperglycemia, dehydration, and acidosis The most common causes are underlying
infection, disruption of insulin treatment, and new onset of diabetes.
- DKA is defined clinically as an acute state of severe uncontrolled diabetes associated
with ketoacidosis that requires emergency treatment with insulin and intravenous fluids.
- Symptoms of hyperglycemia associated with diabetic ketoacidosis may include thirst,
polyuria, polydipsia, and nocturia.
- Signs of acidosis may include shallow rapid breathing or air hunger (Kussmaul or
sighing respiration), abdominal tenderness, and disturbance of consciousness.
- Signs of dehydration include a weak and rapid pulse, dry tongue and skin, hypotension,
and increased capillary refill time.
Hypoglycemia:
- Hypoglycemia is a syndrome characterized by a reduction in either plasma glucose
concentration or its tissue utilization to a level that may induce symptoms or signs such
as altered mental status and/or sympathetic nervous system stimulation.
- Symptoms of hypoglycemia may be categorized as neurogenic (adrenergic) or
neuroglycopenic. Sympathoadrenal activation symptoms include sweating, shakiness,
tachycardia, anxiety, and a sensation of hunger. Neuroglycopenic symptoms include
weakness, tiredness, or dizziness; inappropriate behavior (sometimes mistaken for
inebriation), difficulty with concentration; confusion; blurred vision; and, in extreme
cases, coma and death

OSCE-guide-III.doc

Page 204 of 255

Pediatrics

Bed wetting counselling / Nocturnal Enuresis


Introduction
1- Analyze the CC

3- Rule out infection 2- Impact

4- DD

5- BINDE

6- PMH
7- FH

OSCE-guide-III.doc

OS CF D:
- When did it start? How did it start? Sudden or gradual?
- Frequency
- Primary or secondary (dry period(s) of time)?
- Is it continuous or on and off? How often? Day and night? Every
day? Every night?
Factors: stress / drinking too much fluids before bedtime
How does Mom feel about it?
How does the child feel about it (impact of this on child)?
Constitutional symptoms!
Did you notice if your child has fever or skin rash? Odd smell or
colour of urine? Pulls his penis? Cries while peeing?
Rule out child
neglect / abuse
Medical conditions

BINDE screening
Parent attitude!
DM (drinking too much water / going more
often to pee / feeling tired / losing weight)
- Diabetes insipidus (history of meningitis /
brain infection / head trauma)
- UTI (detailed in No 3)
- Neurological: trauma or surgery to back /
bowel dysfunction / leg weakness or
numbness
- Seizure disorder
Stressors
- New sibling
- Home / school change
- School performance
Very briefly because the child is more than 6 years old
- Scan for risk factors for child abuse / neglect
How is his school performance?
Who is the primary care giver, who else does live with them at home,
is he the only child, any sisters or brothers?
Kidney disease
Kidney disease
Bed wetting
DM
Seizure disorder

Page 205 of 255

Pediatrics

Counselling

Management

OSCE-guide-III.doc

The condition is common;


- By the age of 4 years, 25% of children have this problem
- By the age of 7 years, 5-10% of children have this problem
- More common in boys than girls
May be it is due to regression of his development because of the
current stresses in his life
Condition is usually self limiting & you need to give the child some
time & he will adapt very well to the changes
Try behavioural modification for 3 months:
- Avoid drinking late before sleep,
- Try to wake him up to go to the washroom,
- Document dry nights and use reward system.
Alarm therapy
Pharmacological measures:
- Desmopressin (DDAVP): 1 hour before bedtime. Be careful to
water intake, the only serious side effect is seizures due to water
intoxication
- Oxybutynin: at bed time, it bladder contraction therefore it leads
to bladder capacity

Page 206 of 255

Pediatrics

Breast feeding counselling








A lot of enthusiasm
Congratulation for the news, how do you feel being a (prospective) mother? It is good to hear
that you plan to breast feed. Do you have any concerns?
I need to ask some questions

If still pregnant:

How the pregnancy is going so far?

Pregnancy questions of BINDE

Ante-natal care follow-up (from OB-GYN)

If already delivered:

Birth questions of BINDE


PMH: medications (lithium, antibiotics, anti-convulsions) & allergies
Social history: smoking, alcohol, drugs / work and home environments

1- Rule out contra-indications for breast feeding:


- The mother has active herpes simplex on breast
- The mother has untreated, active T.B.
- The mother has been infected with HIV / is taking antiretroviral medications
- The mother is using or is dependent upon an illicit drug
- The mother is taking prescribed cancer chemotherapy agents, such as antimetabolites
that interfere with DNA replication and cell division OR is undergoing radiotherapy;
however, such nuclear medicine therapies require only a temporary interruption in
breastfeeding
- An infant diagnosed with galactosemia, a rare genetic metabolic disorder
- N.B.: Hepatitis B is NOT a contraindication to breastfeeding
2- Counselling:
Advantages of breast feeding
- Highly nutritional, providing all elements baby needs (especially colostrum), breast milk
contains: more vitamin C, easily absorbable iron, less protein load on the baby
- Contains antibodies to help your child fight infections
- Ready, worm, clean, economic, sterile
- Less allergic
- Secures bonding between mom and baby, emotional satisfaction for the mother and
creates sense of security for the baby
- Help mom reduces weight, a method of contraception

Problems with breast feeding that might happen to the mother:


- Sore nipple (clean & apply soothing lotion)
- Engorgement (pump)
- Mastitis (treat & do not stop feeding)

OSCE-guide-III.doc

Page 207 of 255

Pediatrics

3- Advice:
- Mother should get enough nutrition, fluids, vitamins and rest.
- Give supplementations of:
o Vitamin D from day 1
o Iron from 4 6 months
o Start solid food from 4 6 months, I will give you a table with the recommended
time and types to start solid food
- Mother can use OCP but it will reduce amount of milk OR use an IUD
- Avoid using any medication without asking your Doctor
- Avoid smoking & alcohol
- Care of the breast: frequent cleaning with water and proper hygiene, warning signs:
engorgement, tenderness, redness, hotness
- I will give brochures & information about BF classes
- I will give you the immunization schedule so that you remember to bring him for followup and for vaccination
- Do you have any questions or concerns?
4- Frequently asked questions about breast feeding:
How often you should feed your child?
- On demand at the beginning
- Then the child will adapt to a schedule of every 3 4 hours

How long should the baby stay on each breast? (10 minutes)

How do you know that your baby is feeding well?


- Gaining weight
- Sleeping well
- Wetting 5 6 diapers daily
What do you do if you do not have enough milk? (You can supplement with formula)




How can you breast feed & work at the same time?
- Use pump & keep the milk in a bottle for 3 6 hours outside and 24 hours in a fridge,
you can keep it in the freezer

OSCE-guide-III.doc

Page 208 of 255

Pediatrics

Psychiatry

OSCE-guide-III.doc

Page 209 of 255

Pediatrics

Mental status exam the psychiatry interview


ABS MAP TCIJ
ALWAYS clarify! What do you mean by that?
Appearance Dressed
Dishevelled: dressed and groomed
poorly
Groomed
Given age matches his chronological age
Dressing matches the weather
Behaviour
Psychomotor retardation vs. Psychomotor agitation
Eye contact *
Cooperative *
Hostile *
Abnormal movements / jerks / tics / lip smacking (anti-psychotics)
Speech
Volume
(low / normal / loud)
(process of
Tone
(parkinsonism: monotonous)
talking)
Articulation
Fluent
Pressured speech
Mood
How is your mood? How do you feel?
Write it in patient own words
Affect
Quality (by pt observation)
Elevated
Euthymic
Depressed
Anxious
Congruency
Appropriateness
Others
Stable / range / labile / flat
Perception
Normal perception
Illusions
Hallucinations
Thought
Processing (how does the patient connect Goal directed
ideas); e.g. how did you come here today? Thought block
Flight of ideas
Loose association
Tangentiality
Circumstantiality
Contents
Obsessions
Delusions
Suicidal / homicidal ideation
Cognition
Mini mental exam
Insight
Judgement
Brief comment: (1) The patient is well dressed, well groomed; and his appearance matches his
chronological age. (2) He has (good / poor) eye contact, cooperative (not), with psychomotor
(retardation / agitation) (3) His speech is of normal volume, tone, fluent, not slurred, and not
pressured. (4) His mood is (5) His thoughts are organized (or disorganized). (6) There are no
delusions or hallucinations. (7) There is no suicidal ideation or homicidal thoughts. (8) Judgement
(good / poor), insight (intact / lost).

OSCE-guide-III.doc

Page 210 of 255

Pediatrics

Perception
Hallucinations:
- Visual:
o Usually organic (tumour / epilepsy / cocaine and amphetamine)
o Brain tumour /+/ alcohol intoxication / DT /+/ cocaine / hallucinogens
o Do you see objects / things that others do not see?
o Can you describe what do you see?
o Do they give you any messages?
o Are these messages asking you to harm yourself or anyone else?
- Auditory:
o Usually schizophrenia
o Do you hear voices / things that other people do not hear? When you are alone,
do you hear voices coming from your head?
o How many voices
o Are they familiar or not?
o Are they talking to you or about you? What are they telling you?
o Did they ever ask you to harm yourself or somebody else? What is preventing
you from doing this?
o How do you feel about these voices?
- Tactile:
o Cocaine chronic use (most probably) OR delirium tremens
o Do you feel ants / insects crawl on your body / skin?
- Smell: usually epilepsy
Though
Processing:
o

How did you come here today?

Content:
+ Obsessions:
- Repeated intrusive thoughts that the patient knows it is wrong, and he can not resist, if he
resists  anxiety  take actions to try to anxiety (compulsions)
- Mostly regarding: cleanliness, contamination / order / checking /
o Do you have any repeated thoughts or images that you find difficult to resist?
About what? What do you do?
+ Suicidal / homicidal ideation:
o Do you have any thoughts or ideas of harming yourself?
o Or harming other people?
o Any access to weapons?
- If the patient is suicidal / homicidal / can not take care of himself  admit, if he/she
refuses  form 1 (for involuntarily admission for 3 days for psychiatric assessment
by another physician). Form 1 has to be filled within 1 week from seeing the patient.
- I want to file form 1 for the patient and call the hospital security to bring the patient back.
I have concerns about safety of the patient and other people.

OSCE-guide-III.doc

Page 211 of 255

Pediatrics

+ Delusions:
- False fixed believes, that do not match with the patient cultural and religious background
- You can not convince the patient it is wrong, even with proof
- The ideas
o Believable (could be) non bizarre
o Unbelievable (could never be) bizarre
o Do you believe that other people would like to harm you? OR conspire against
you?
o Do you think that others would like to control you?
o Read your mind?
Thought broadcasting
o Put thoughts into your head?
Thought insertion
o Steal thoughts from your head?
Thought withdrawal
o If you are watching the TV or reading the newspaper, do you believe that they
are talking about you?
Delusion of reference
o Do you believe that you are a special person? With a special talents? Or special
power? Do you believe that you have a special mission to do in life? Do you
think you deserve to be treated specially?
Grandiosity
o Do you feel other people are falling in love with you?
Eromantic
o Do you believe any part of your body is rotten?
Cognition:
- Are you becoming forgetful? Are you losing your staff?
- Assess abstract vs. concrete thinking!
Insight:
- Do you think that you are doing well? Or do you need help?
Judgement:
- If there is a fire in the building, what are you going to do?
- If you find a stamped and addressed envelop on the ground, near the mail box, what
would you do?
General screening:
- Depression:
o What is your mood? How do you feel?
o Did you lose interest in things that were interesting to you before (e.g. certain
hobby, playing something)?
- Anxiety:
o Are you the kind of person who worries too much?
o Do you have excessive fears or worries?
- Psychosis:
o Do you hear voices or see things that others do not?
o Do you think that someone else would like to hurt you?

OSCE-guide-III.doc

Page 212 of 255

Pediatrics

DSM-IV-TR
Diagnostic and Statistical Manual of Mental Disorders 4th Ed/2000 Text Revision
Multi-axial system (5 axes)
The DSM-IV organizes each psychiatric diagnosis into five dimensions (axes) relating to
different aspects of disorder or disability:
- Axis I: Clinical disorders, including major mental / psychiatric disorders, and learning
disorders, Substance Use Disorders
- Axis II: Personality disorders and intellectual disabilities (although developmental
disorders, such as Autism, were coded on Axis II in the previous edition, these disorders
are now included on Axis I)
- Axis III: Acute medical conditions and physical disorders
- Axis IV: Recent stressors, i.e. psychosocial and environmental factors contributing to the
disorder
- Axis V: Global Assessment of Functioning or Children's Global Assessment Scale for
children and teens under the age of 18 (a questionnaire)
Example of a full proper psychiatric diagnosis:
- Bipolar I / Anti-social personality / DM+HTN / Divorce / global assessment was not
done because the patient was not cooperative
Diagnosis of diseases based on DSM-IV-TR is based on CRITERIA and TIME.
- Depression:
o MI PASS ECG or MIS GE CAPS
o You need to find at least 5 of the 9 for > 2 weeks, including at least one of the
Mode or Interest.
o If not fulfilling these criteria: non-specified mood disorder
o In teenagers: we do not need M or I, we can replace it with agitation OR drop in
school performance + other 4 criteria.
- Schizophrenia:
o 4 positive symptoms: hallucinations, delusions, disorganized speech,
disorganized behaviour.
o 1 other category; negative symptoms: mood, catatonia
o At least 1 month of active symptoms (2 of 5) + 6 months of deterioration in
functioning.
o 1 active symptom (not 2) is accepted in the following cases:
 If the hallucinations are > 2 voices (commanding or commenting)
 The delusions are bizarre
- Anxiety:
1- Panic attack vs. panic disorder:
a. In panic disorder, there is at least ONE panic attack with at least ONE month
of worries and fears of having it again
b. Panic attack might be one or more attacks
c. If patient is avoiding going outside  with agoraphobia
2- Phobias specific to certain objects
3- OCD
4- GAD: excessive unrealistic fears for more than 6 months PLUS other manifestations
5- PTSD (acute or chronic): Have you ever encountered a situation in which your
personal or mental safety and wellbeing were endangered? When? Do you have
flashbacks or nightmares?

OSCE-guide-III.doc

Page 213 of 255

Pediatrics

History taking Psychiatry


MOAPS: mood / organic / anxiety / psychosis / serious conditions (self care, suicide, homicide, support) / HEADSSS
Major psychiatric illness
Suicide Minor psychiatric illness
Personality disorder / Drinking
/ addiction / Eating / sleeping
S
disorders / Somatic disorders /
A
Mood
Anxiety
Psychosis
Cognitive (delirium /
D
dementia) ...
P
Criteria

Low: MI PASS ECG
1- Panic attack vs. panic
Criteria (1 month of 2-5 active
E

High: DIG FAST +
disorder
symptoms + 6 month of
R
MI PASS ECG
2- Specific phobias
function deterioration)
S
3- OCD
- 1st time or did you have it
- 1st time or did you have it
O
4- GAD
before?
before?
N
5- PTSD
- What about the opposite?
S
Dx: one of the mood disorders
Past psychiatric history
Past psychiatric history
MOAPS
Organic:
1- SAD if IV drug use: check for liver (hepatitis) / constitutional symptoms (HIV)
2- PMH, including constitutional symptoms
3- Rule out medical conditions as DD, e.g. medications and specific diseases
Thyroid disease
Mitral valve prolapse
Brain tumour / HIV
Anxiety / psychosis
Mood / psychosis
Mood / anxiety
Serious conditions (red flags):
- Self care (are you eating / sleeping well?)
- Suicidal / homicidal ideation
Social history:
- How do you support yourself financially?
- With whom do you live? Family support?
For teenagers, add: HEADSSS
Family history of psychiatric illness: suicide / depression / SAD / seen by psychiatrist

OSCE-guide-III.doc

Page 214 of 255

Psychiatry

Mood disorders:

OSCE-guide-III.doc

Page 215 of 255

Psychiatry

Psychosis
Pt comes to the clinic complaining of strange feelings in his right hand
Clarify the CC
OSCD

Criteria :

Hallucinations

Delusions
1st time or did you have it
before?
Past psychiatric history
Organic

Mood / Anxiety
Serious conditions
Social history
Family history

123456


Can you tell me what is going on!


What do you mean? Is it pain, numbness, tingling?
OS: When did it start? What were you doing?
C: Is it all the time, or on and off? Any specific setting?
In you opinion, why do you have this?
In addition to , do you have any other strange feelings?
Any hallucinations: What? For how long?
Any (tactile or visual) hallucinations  will be mostly organic
 cocaine (substance abuse) until proven otherwise

Have you seen a psychiatrist before?


Cover the following:

Head injury / trauma

Brain infection (fever, vomiting) / Brain tumour (N/V)

HIV

Thyrotoxicosis

Medications: thyroxin, anti-parkinsonism (L-dopa)

SAD (cocaine, marijuana, amphetamines)
Constitutional symptoms / self care / suicide

Differential diagnosis:
- Schizophrenia (a mental disorder that impairs the way you perceive reality. It could be
very disabling)
- Brief psychotic disorder
- Post-partum psychosis
- Drug-induced
- Brain tumour
- HIV
- Delirium / dementia
- Mood disorder
Investigations:
- CBC / toxicology screen
- HIV / syphilis test
- Septic workup
- CT / MRI brain
Pay attention to patient cues
- Poor hygiene
- Looking at wall or ceiling
- Paranoid
- Talking to some body.

OSCE-guide-III.doc

Page 216 of 255

Psychiatry

Management:

Will examine and do some tests


o
Because you have stopped your medications, it looks like your schizophrenia is
relapsing, that is why we need to admit you and refer you to psychiatrist to
reassess your condition, how do you feel about that?

Will start medication which is helpful in reducing the symptoms (Risperidone). Explain
about side effects: weight gain / blood glucose level / cholesterol / drowsiness

Arrange follow up visit

Information e.g.: support groups / brochures


Notes:

Whenever you suspect substance abuse: after you ask have you ever tried recreational
drugs? ask what about crack cocaine? Do you sniff? Do you inject? Did you share
needles
o
If shared needles  scan for hepatitis (liver symptoms), HIV (repeated
infections / repeated diarrhea)
If the patient came because his parents or roommate have concerns, you can ask the
patient: what kind of concerns does have?

Difficult situations:

If the patient with hallucinations tells you that he sees a radiation and gives you a photo and asks:
do you see it doctor?  For me it does not look like radiation, but I can understand that you see
this as radiation
At any time the patient starts to agitate and worries about special hallucinations!
o
You are safe here, no body will harm/hurt you
If the patient is away:
o
Do not chase him/her around the room, stand by your chair
o
I would like to assure you that you are safe here, no one will harm you
I do not like Egyptian people, by the way, are you Egyptian doctor?
o
Why are you concerned about that?
o
Whether I am Egyptian or not will make no difference in this situation
I do not like gays, by the way, did you see a gay patient today doctor?
o
Why are you concerned about that?
o
As a physician, I deal with all patients, regardless their race, religion, sex, sexual
orientation or anything else!
Do you think I am crazy doctor?
o
There is no medical term called crazy. However sometimes some people have
difficulties in the way they handle their thoughts and the way they interact with and
perceive reality, we call that schizophrenia. It is a mental illness like any other illness that
can affect the body, that we can treat with medications

Case: A young man can not move his neck, DD acute dystonia:

Trauma

Meningitis

Subarachnoid hemorrhage

Cervical disc

Muscle spasm

Anti-psychotic medication (e.g. haloperidol), treatment: lorazepam


Case: Patient is in the balcony, wants to fly, mother is calling you?! What do you tell her?
Talk to him to attract his attention

The doctor should get the phone number and address and ask the nurse to call 911

Ask her if you can speak with the patient  psychosis patient

OSCE-guide-III.doc

Page 217 of 255

Psychiatry

Schizotypal personality disorder


-

Delusions
Magical believes
Limited number of friends that share the same believes

Ethical challenges:
- Will you hook me to the cleaning machine that cleans the blood? I am glad you came
here today, I think you need help, but not with the machine.
- Will you admit me doctor? We need further psychiatrist assessment then we may need to
admit you.

OSCE-guide-III.doc

Page 218 of 255

Psychiatry

Panic attack
Patient comes to the clinic complaining of dizziness
Clarify the CC

Analysis of CC

HPI

Criteria
during
attack
(AS)

Between
attacks
Anxiety

MOAPS

When you say dizziness, do you feel:


- light headed
- Spinning
Os Cf D
Analyze the attack: How did it end? How many attacks? Are they similar?
What were you doing? When was your last one?

cardiac: heart racing17 / chest pain / tightness / excessive sweating

respiratory: SOB / wheezing

neurology: dizziness / numbness / tingling / weakness / shaky /
buzzing sounds / headache / vision changes / difficulty balance

GIT: nausea / vomiting / difficulty swallowing

Depersonalization: you feel that you are outside of your body

Derealisation: feel things around you are strange / not real

Excessive fears of: losing control / going crazy / dying

Do you have fears of having other attacks?

How does it affect you? Do you avoid going out? (Relation to
agoraphobia?)

Are you the kind of person who worries a lot? Excessive fear

Are you under any stress in your life? How can you cope with this?

Any special fears? High altitudes? Closed places? Talking in public?
Pets?

Have you ever encountered a situation in which your personal or mental
safety and wellbeing were endangered? When? Do you have flashbacks
or nightmares?







Past psychiatric
history
Serious conditions

Hypoglycemia
Thyroid disease
Pheochromocytoma
SAD (cocaine / amphetamine / alcohol withdrawal) / Caffeine
Arrhythmias / MVP
Anemia (fatigue / light headedness / heavy menses / PMH anemia)

Constitutional symptoms
Self care

Suicide
AMPLE
Heart diseases / thyroid / abdominal tumours



PMH
Family history
Social history

17

Any heart racing, ask the patient can you tap it for me, then comment to the examiner: it
looks regular / irregular for me

OSCE-guide-III.doc

Page 219 of 255

Psychiatry

COUNSELLING
-

With what I heard from you today, the most likely diagnosis to your symptoms is a
medical condition that we call panic attack. We still need to do physical examination,
some investigations like blood works, urine analysis, electrical tracing of your heart
(ECG), to exclude other medical conditions and to confirm our diagnosis.
o Now Mr what do you know about panic attacks?
o Do you want me to explain this in details over the next few minutes?
Inform the patient:
o Explain the pathophysiology: panic attack or panic disorder is a kind of severe
anxiety, it happens suddenly, in attacks. Usually it is related to stress.
o It is due sympathetic over-activity, imagine you are crossing the road, and a
speedy car is approaching you, normally, our body reacts to this by enhancing the
sympathetic nervous system, which leads to some changes: increase in the heart
rate, rise in blood pressure, and you feel alert. This is normal and useful reaction.
o The same reaction might happen suddenly without any external trigger, and this
would be stressful, and this is what we call a panic attack.
o Consequences: this might happen again / may cause significant limitations
Preventive measure:
o Life style modification (caffeine and alcohol / better sleep hygiene)
o Relaxation techniques (e.g. breathing techniques / meditation)
Treatment:
o Like many other conditions, it could be treated.
o Treatment varieties include:
 Talk therapy
 Medications: 2 types
Anti-anxiety: Lorazepam 0.5 mg qhs x 2 weeks (it is important
to use it on schedule, not irregularly)
SSRIs: Paroxetine 10 mg od x 4 weeks similar to what we
usually use with depression. Like any other medication, they
have their side effects; GIT disturbances, headache, some sexual
dysfunction. And this improves by time.
Follow-up 2-3 weeks
Offer more information: brochures / web sites
Whenever you suspect social problems  involve the social workers

Generalized Anxiety Disorder (GAD)



Excessive anxiety and worry (apprehensive expectation), occurring more days than not for at
least 6 months, about a number of events or activities (e.g. work / school)

The person finds it difficult to control the worry

Treatment:
o Lifestyle: caffeine and alcohol avoidance, sleep hygiene
o Psychological: psychotherapy, relaxation, mindfulness, and CBT
o Pharmacological:
 Benzodiazepines (short term, low dose, regular schedule, long half-life,
no PRN)
o Buspirone (tid dosing)
o Others: SSRIs/SNRI (paroxetine), TCAs (nortriptyline), beta-blockers
o Avoid Bupropion due to stimulating effects

OSCE-guide-III.doc

Page 220 of 255

Psychiatry

Tiredness OR weight loss


Introduction
CC
Clarify the CC

Os Cf D

Ask about sleep

Tiredness
- Is it weakness? Can not do?
- Lack of energy? Tiredness?
- Limitation of activity? How many
blocks are you able to walk?
- Not being refreshed after sleep? Do
you have any special concerns?
Timing:
- Morning or all day: ?depression
- End of the day: organic
- How many hours? And before?
- Find difficulty falling asleep?
- Do you wake up during night?
- When you wake up, do you feel
refreshed? Do you need naps?

sleep /or/ the same


? organic
- Constitutional symptoms
- Tender points in your body
All systems review (head to toe):
- Cardiovascular
- Lung
- GIT / liver
- Urinary
- MSK / skin / rheumatology / autoimmune
- Anemia / bleeding
LMP / mens. hx / pregnancy
Cancer colon for males
- Endocrine (thyroid / DM)

sleep / insomnia
? depression
Criteria : MI PASS ECG
1st time or did you have it before?
What about the opposite?
Past psychiatric history
Organic //
Anxiety / psychosis
Serious conditions
Social history
Family history

Counselling on depression

MI mood / interest
PMH of cancer
Social history SAD
Family history
Diabetes Mellitus:
- Hx of DM
Fluctuations (acute)

- Symptoms:
Complications (chronic) vascular
MICRO
MACRO
- Eat more
- Blurred vision
DKA
- Nephropathy
- CAD
- Drink more - Tired
Hypoglycemia
- Neuropathy
- CVS screen
- Pee more
- Weight loss
- Retinopathy
- PAD /
impotence
N.B. -blockers are contraindicated in DM: it causes hyperglycemia / and it masks hypoglycemia

OSCE-guide-III.doc

Blood sugar measured


Emergencies

Page 221 of 255

Psychiatry

Sleep / fatigue notes


1- Sleep hygiene questionnaire:
- How many hours? How about before?
- How does this affect your life? Do you work night shifts? empathy
Before
- When do you go to bed?
- Find difficulty falling asleep? How long does it take you?
- Do you sleep in dark room? Lit room?
- Do you eat before sleeping? Heavy meals? Late meals?
- Do you drink before sleep? Alcohols? Coffee?
- Do you exercise before sleep?
- Do you read in bed? Watch TV?
During
- Do you wake up during night? Any reason? Can you sleep again?
- Do you sleep alone? Or do you have sleep partner?
o Does he notice you are snoring? Do jerky movements?
o Does he snore? Does he do jerky movements?
- Do you have dreams? Nightmares?
After
- Do you wake up early?
- When you wake up, do you feel non-refreshed? Do you need naps?
- Do you work on shifts?
2- Depression:

Psychomotor question: do you think things take more time to do now? Compared
to before?

Pancreatic cancer  depression

Whenever you find alcoholic patient  check for complications:
i. Cancer pancreas
ii. Liver damage ( liver enzymes) / hepatitis / cirrhosis / carcinoma
iii. GIT: upper GIT bleeding / peptic ulcer perforation
iv. Depression (alcohol / depression / suicide) is common combination

Treatment for depression (or most of the psychiatric diseases):
i. Life style modification
ii. Talk therapy
iii. Medications
Usually in combination
3- Domestic abuse presentations
- Headache
- Abdominal pain
- Insomnia / sleeping pills
- Vaginal bleeding
- Fatigue

OSCE-guide-III.doc

4- Fatigue
- Depression / PTSD
- Domestic abuse
- Hypothyroidism
- Fibromyalgia
- Anemia:
- Old person: think cancer & occult blood
- Young female: think menorrhagia
- Diabetes Mellitus, polyuria
- Anorexia nervosa

Page 222 of 255

Psychiatry

Insomnia
A lady complaining of insomnia
Common presentation to: domestic abuse / depression / anxiety
Introduction
CC
Clarify the CC

Insomnia / Tiredness
- Difficulty falling sleep
- Waking up
Analysis CC: Os Cf D
- More at certain time of the week?
- Did you try anything to help? Did it work?
Ask about sleep Sleep hygiene questionnaire
Anxiety - Do you have too many worries?
- What comes in your mind before falling asleep?
- Any changes / stresses in your life?
- Do you wake up with nightmares?
Depression - Screen with MI; if positive  screen MI PASS ECG
PMH
Social
- With whom do you live? Support?
 Screen for domestic violence or spouse abuse
- Children?
- Financial support?
Notes

Did you ever think to hurt yourself? NO, my kids need me,
o What about if they are not around? Maybe!
o This means: implicit yes to suicidal ideation


Lady looking for renewal of benzodiazepines:


o Renew it and tell her that she needs to use it properly
o Tell her that she is in grief, she needs SSRIs, start SSRIs, taper benzodiazepines

OSCE-guide-III.doc

Page 223 of 255

Psychiatry

Domestic Violence Spouse Abuse


Screen for domestic violence or spouse abuse:
-

ASSURE confidentiality: I would like to assure you that our conversation is completely
confidential, whatever you will tell me here, I will not release any information, unless
otherwise required by the law!
With whom do you live? How do you describe this relationship? Supportive?
o How long have you been in this relation?
o Do you feel safe at home? In this relationship?
Do you or your partner go through stressful times?
o Do you sometimes have conflicts? Arguments?
Is there any chance that you partner drinks or uses drugs? How often? When he drinks,
does he become angry? Lose control? When was the last time?

Verbal / emotional:
- Does he start to shout at you? Swear at you?
- Does he call you names? How does this affect your self-esteem?
Physical:
- Did he ever get angry to the extent that he became physical?
- Did he try to put you down? Does he try to control you? How did this affect you?
- Did he try to push you? Hit you? How many times?
- Any visits to the ER? When was the last time?
Financial:
- Who is controlling the spending at home?
- Do you have access to financials? Do you take permission?
- Did he ever to try to take you money against your wishes?
Sexual:
- Did he ever force you to do sexual activity against your will? How do you feel?
-

Duration of abuse? Severity? Hospitalized!


How does this affect you? Are you pregnant now? Do you work now? How do you
support yourself financially?

Children involvement:
- Did he ever mistreat / abuse you in front of the children?
- Did he ever mistreat / abuse the children?
Fatality:
- Do you have access to weapons at home?
- Did you ever have thoughts to put an end to this all by ending your life or his life?
- Did you ever talk to anyone about this?
OUTCOME:
- The patient decides to end the relationship and leave  you must provide support and
shelter
- The patient decides to continue: either with OR without police involvement

OSCE-guide-III.doc

Page 224 of 255

Psychiatry

Wrap-up:
- Based on what you have told me, what you are experiencing (or have gone through) is
called domestic violence or spouse abuse, and it is common. It is an illegal crime, and it
is against the law.
- It is not your mistake, and you should not feel guilty about that. It is unacceptable, and
nobody deserves to be treated in this way.
- We know from studies that the situation will not improve, on the contrary, it will
deteriorate, and you do not need to accept this. The studies show that the longer you stay
in this relationship, the higher the chance of abuse.
- Consequence to the children (if any): psychological trauma
- It is important that you consider reporting the situation to the police for your safety. It is
difficult decision to leave or stay.
- The husband needs help, check willingness to get counsel.
-

If you like to end the relationship and leave:


o You can call the police, they will come and arrest him, then they will investigate
the case, and may put charges against him, and you do not need to worry that he
might hurt you, the police can give a restraining order
o I will connect you with the social services and support groups, you do not need to
go through all of this by yourself, they will be able to help with housing,
financial support for both of you and the kids
However, if you want to continue the relationship, you can still involve the police. We
know from the studies that if you involve the police, situation will improve, they will
come, speak with him, do some investigations, and then, by law, your husband will be
obliged to attend special training courses:
o Anger control
o Stress management and relaxation techniques
o Drinking problem rehabilitation
o Marital counselling
My concern is that if you go back without taking any measures, things might deteriorate
and get out of control and one of you might end up losing her/his life, at least:
o You need to prepare an escape plan:
 Where to go when you dont feel safe!
 A bag with essential belongings (e.g. IDs, bank documents, )
 Easily accessible (you can pick it in second and leave)
o We will schedule a follow-up visit within few days
Is it ok with you if I document this?
Support:
o Police is not the only option, in the city, there are community recourses: shelter,
hotline, legal aid, they are free, and confidential.
o Give social support group numbers.

Clues for domestic abuse:


- She is avoiding eye contact
- Not answering directly for questions about the relationship with partner
- I hate drinking
- He is a great father, likes his daughters, and works very hard.
- I wish I can be a better mom / I am a failure / do you think too I am clumsy.

OSCE-guide-III.doc

Page 225 of 255

Psychiatry

Child Abuse18
The child came to the ER with femur fracture, the skeletal survey showed multiple healing
fractures, counsel
Introduction

Analyze the event

Is it the first time?

BINDE

Other children
PMH of the child

I assure you that he is ok, and after we finish I would accompany


you to see him, is it ok with you. Before this I would like to ask you
some questions to know more about his condition / fracture
- Can you describe what happened? What he was doing?
- Who witnessed it? Anybody else?
- When did this happen? When did you come to the ER?
- Other injuries before or visits to ER?
- Analyze each event
- Did you take him to the same hospital?

Are you the biological mother?

Is your current partner the biological father?
- B: screen for the risk factors for child abuse:

Was this pregnancy planned? Regular f/u visits?

Was he a term baby? Did he need special attention?

Has he had congenital anomalies?

Do you think he is a difficult baby? Fussy baby?

SAD for both partners!
- I: Are his shots up-to-date? If no, any reason?
- N: What is his weight? Do you know about his growth charts?
Regular f/u visits?
- D: Is he hyperactive baby? Challenges you most of time?
- E:

How do you support yourself financially? Any support from
the biological father?

Anybody at home seeing a psychiatrist? Illness?

Tell me more about your childhood
- Do you have other children?
- Repeated visits to ER?
Chronic illness / bone or metabolic diseases

Screen for domestic violence or spouse abuse

18

Good TWO screening questions: immunization (not up-to-date) / weight (FTT or under
nutrition)

OSCE-guide-III.doc

Page 226 of 255

Psychiatry

Wrap-up:
-

How do you feel your child has so many fractures?


I know that you are concerned about your son, sometimes it is challenging to look after a
child, especially if you do not have enough support.
In the same time, children at that age have flexible bones, and it is difficult to explain the
nature of his fracture(s) only by jumping from a couch.
In these situations we usually involve the children aid society (CAS), this is a kind of
social services devoted to the safety and well being of children
o Please do not do this? Why?
o They will take my son! Why r u saying so? Any experience with them?
o Not necessarily that they take your child, this is not their first priority
Their first concern is the safety of your child, what will happen is that they will:
o Ask you some questions about what happened!
o Come to visit you at home
o Talk with your partner
Then they will take their next step based on the results of these meetings
I am sure you are sharing my concerns about (the child name) safety!

If there is spouse abuse / domestic violence:


- Also, based on what you have told me, what you are experiencing (or have gone through)
is called domestic violence or spouse abuse, and it is common. It is an illegal crime, and
it is against the law.
- It is not your mistake, and you should not feel guilty about that. It is unacceptable, and
nobody deserves to be treated in this way.
- We know from studies that the situation will not improve, on the contrary, it will
deteriorate, and you do not need to accept this. The studies show that the longer you stay
in this relationship, the higher the chance of abuse.
- It is important that you consider reporting the situation to the police for your safety.

OSCE-guide-III.doc

Page 227 of 255

Psychiatry

Domestic abuser
You are bout to see a 55/60 years old gentleman, whose wife is recovering in the ER, she has
bruises, and he asked to speak with you. In the next 10 minutes counsel him
Introduction
Analysis
SH / Safety
Counsel

Domestic violence

Anger control
Stress management and relaxation techniques
Drinking problem rehabilitation
Marital counselling

Offer social support if there is a need




Introduction:

If the patient asked to see you: I understand that you are here because you are accompanying
your wife, she has bruises and my colleagues are taking care of her right now. How can I help
you today?

If the patient is inquiring about her status: I can assure you that she is stable and in safe hands
now.

If the patient asks to see her: After we will finish, I will ask her, if that is ok with her, I can
take you there.
Analysis:

Do you have any idea how did she end up having all these bruises?

Was there any argument / disagreement / shouting? Did you lose control? Did it end up that
you physically hurt her?

Is this the first time or happened before? Any repeated visits to the ER before?
Social history:
How long have you been together? What is the nature of your relationship? Stable? Was there
and significant conflicts before?

Was there any recent change or stressor in your life? How do you support yourselves
financially? Do you have enough resources?

Do you have anybody else at home? Any family support? Do you have children? How is the
relation with them?

SAD


Safety:

Criminal record / access to weapons at home

If you go home now and face the same situation, how would you react?

Any chance that you might hurt yourself or any other one?

OSCE-guide-III.doc

Page 228 of 255

Psychiatry

Counselling:




I can see that you are going through stressful period of time. It must be difficult for you and
your wife. Sometimes this stress might present by changes in behaviour and/or personality.
If you do not have enough support at home, things might get out of control.
What happened is what we call domestic violence; it is a kind of physical abuse. It is not
acceptable, and it is considered illegal crime. However, this is your wife decision. If she
chooses to report you, that is her right, and nobody can prevent her. She can press charges
against you, and they will take you to the court, in this case you might need legal help, this
might have serious consequences.
On the other hand, if she decides not to take any measure, may be you should try to improve
the situation by taking steps to decrease the stress in your life, and you can consider reducing
your alcohol drinking. Drinking alcohol might leads to what we call disinhibition in which
one might lose control on his reactions and usually this leads to violent and serious
consequences.
I can help you by referring you to attend:
o Alcohol rehabilitation programs
o Stress management and anger control programs

I recommend also that you consider attending family marital counselling; they have good
experience in dealing with couples going through difficult times.

Finally, I can help you to contact the social services. They might be able to help; you can
speak with them and see what they might be able to do! Is that ok with you?

OSCE-guide-III.doc

Page 229 of 255

Psychiatry

Depression
Screen: MI PASS ECG
Organic:
Illness: hypothyroid, anemia / pernicious anemia, M.S, cancer / cancer pancreas

Medication B Blockers, Anti-parkinsonian

SAD


Depression management / counselling


7. Inform the patient
a. Based on what you have told me, the most likely explanation for your condition
is a medical condition called depression, what do you know about depression?
Did you read anything about it? It is the most common mood disorder, in which
you feel low, upset and lack of energy. It is a common problem, and it is
treatable.
b. Explain the pathophysiology: it is related to imbalance of the chemicals in our
brain, most likely related to decreased serotonin
c. Consequences / complications of the condition: it affects functionality, leads to
decreased concentration and ability to work, and in severe cases in susceptible
persons, it might lead them to suicide
8. Treatment (outpatient):
a. Talk therapy (psychologist / psychiatrist)
b. Medications: SSRIs (which are very effective medications) take effect after few
weeks  Cipralex 10 mg PO od x 3 weeks (side effects include: 1. weight gain,
2. GIT symptoms, 3. sexual dysfunction). These side effects improve by time and
to reduce it, we start increasing the dose gradually (start low, go slow).

Because depression has serious (fatal) consequences, if you do not take this
medication, you will be compromising your safety.

DO NOT stop it on your own; we can start to gradually decrease the dose
after the proper period of treatment. You need to continue on it for at least 6
months after symptoms improve.
c. Follow up visit after 2-3 weeks
d. Contract: sometimes when the anti-depressant starts to work, the energy level
improves while the mood is still low, that is why sometimes there is increase in
suicidal ideation. Usually happens 2-3 weeks, you need to promise me that if this
happens with you, call 911 or call me immediately and come to see me
9. Offer more info: brochures / web sites
10. Break every 30-60 seconds and ask the patient: does that make sense? Is this acceptable?
Reasonable? Is it clear?
11. Treatment (hospitalization): for suicidal patients
a. From what you have told me, you are meeting the criteria of what we call
and I have concerns about your safety, because you have more than THREE risk
factors for suicide as per the screening test. Do you mind to stay with us in the
hospital for few days, so we can do the required investigations and start the
medications, until you feel ok, what do you think about that?

No doctor, I am not staying in the hospital!!!

Actually, Mr as I told you, I have concerns about your safety, we can not
compromise your safety. And by allowing you to leave today, we will be
compromising your safety.

OSCE-guide-III.doc

Page 230 of 255

Psychiatry

Dysthymia
COMMON CASE IN THE EXAM


Depression presentations:
o Sad (low mode), weight loss, insomnia, tired
 Scale the sadness 0 10
o Indecisiveness: difficulty making decisions
o Low self esteem  how do you feel about yourself?
o If good days: ask for periods (check for gaps 2 months)

How does it affect your life?


o Then assess functionality; what do you do?!

Screen MI:
o If positive  MI PASS ECG
 If positive  assess SAD PERSONS

Any relation to menstrual periods  pre-menstrual dysphoric disorder

Counselling: similar to depression, but mention that Dysthymia is a milder form of


depression, with longer duration, and does not interfere with life functionality.

OSCE-guide-III.doc

Page 231 of 255

Psychiatry

Premenstrual Dysphoric Disorder (PMDD)


DSM-IV-TR Diagnostic Criteria for Premenstrual Dysphoric Disorder
A. In most menstrual cycles during the past year, five (or more) of the following symptoms were
present for most of the time during the last week of the luteal phase, began to remit within a
few days after the onset of the follicular phase, and were absent in the week post-menses,
with at least one of the symptoms being one of the first four listed
1. Markedly depressed mood. Feelings of hopelessness, or self-deprecating thoughts
2. Marked anxiety, tension, feeling of being "keyed up" or "on edge
3. Marked affective lability
4. Persistent and marked anger, irritability, or increased interpersonal conflicts
5. Decreased interest in usual activities
6. Difficulty concentrating
7. Lethargy, easily fatigued, lack of energy
8. Change in appetite overeating or specific food cravings
9. Hypersomnia or insomnia
10. A sense of being overwhelmed or out of control
11. Physical symptoms breast tenderness or swelling, headaches, joint or muscle pain,
sensation of bloating or weight gain
B. The disturbance markedly interferes with work, school, social activities or relationships with
others
C. The disturbance is not merely an exacerbation of the symptoms of another disorder such as
Major Depressive Disorder, Panic Disorder, Dysthymic Disorder or Personality Disorder
D. Criteria A, B and C must be confirmed by prospective daily recordings and/or ratings during
at least two consecutive symptomatic cycles (how to diagnose)
Treatment

1st line: SSRIs highly effective in treating PMDD
o Fluoxetine (20 mg od) and sertraline (50 mg od) most studied
o Can be used intermittently in luteal phase (mid cycle  onset of menstruation
pre-menstrual) for 14 days

2nd line
o Alpraxolam (Xanax) for anxiety symptoms

3rd line
o OCP containing progesterone drospirenone (e.g. Yasmin)
o GnRH agonists (e.g. leuprolide)
o If GnRH agonist completely relieves symptoms, may consider definitive surgery
(i.e. Total abdominal hysterectomy+ bilateral salpingo-oophorectomy)

OSCE-guide-III.doc

Page 232 of 255

Psychiatry

Abdominal Pain / Headache


Abdominal pain for few weeks, and was seen by a surgeon last week, comes to your office ( to
have MRI OR to renew medication).
Headache for 7 months, young man, comes to renew Tylenol 3
Chief complain a request (investigations OR medication renewal)
HPI

Analyze
the CC






AS





Impact
Red flags








Os Cf D / PQRST / / 1st time


When did the headache (pain) start?
Did you seek medical attention? What was the diagnosis? Did you take
any medication? When did you start Tylenol 3? Why?
Analyze previous visits: is the pain different from before? How?
Other pains / headache
GIT / liver
Genito / urinary
How does this affect your life? How are you coping?
Constitutional symptoms
Screen red flags for headache:

Trauma

Worse at night

Nausea / vomiting

Bothered by light /+/ Neck pain / stiffness

Weakness / numbness / tingling in body / seizures
Are you under stress?
Support systems

DD
Medical problem
PMH
FH
SH

Somatisation
MOAPS screening (screen for depression)
PMH: HEAD SSS
FH of psychiatric disease
SH
Counselling

Physical examination
Notes:
- Somatisation disorder: (4 pains / 2 GIT / 1 neuro / 1 sexual) complains
- If the pain is only during the day, and not nights  mostly non-organic

+ If requesting MRI  NO MRI


+ Actions for Tylenol 3:
- If using it for few weeks  stop it / do not worry about withdrawal symptoms
- If using it for long time  counsel / renew / promise to cut down gradually
- If using it for depression  start SSRIs / taper Tylenol 3 (decrease gradually) / then brief
counselling for BOTH Tylenol 3 & depression
- If drug seeker  DO NOT give any narcotics / rehabilitation
+ Always renew the medication, except for drug seeker (very anxious to renew the narcotic / will
not accept another alternative / making stories to rash you to prescribe it).
+ If not drug seeker: reassure the patient that you will prescribe her pain medication before the
end of the session, but in order to prescribe the proper medication, you need to ask some
questions, can you bear with me for few minutes?

OSCE-guide-III.doc

Page 233 of 255

Psychiatry

Somatoform disorders DD
General Characteristics:
 Physical signs and symptoms lacking a known medical basis in the presence of psychological factors
 Cause significant distress or impairment in functioning
 Symptoms are produced unconsciously
 Symptoms are not the result of malingering or factitious disorder which are under conscious control
 Primary gain: somatic symptom represents a symbolic resolution of an unconscious psychological
conflict; serves to reduce anxiety and conflict; no external incentive
 Secondary gain: the sick role; external benefits obtained or unpleasant duties avoided (e.g. work)
Management of Somatoform Disorders:
 Brief frequent visits
 Limit number of physicians involved in care
 Focus on psychosocial not physical symptoms
 Minimize medical investigations; co-ordinate necessary investigations
 Biofeedback
 Psychotherapy: conflict resolution
 Minimize psychotropic drugs: anxiolytics in short term only, antidepressants for depressive symptoms

Somatization disorder

Conversion disorder

Pain disorder

Hypochondriasis

Fibromyalgia

Lots of symptoms: 8 physical symptoms that have no organic


pathology: 4 pain + 2 GIT + 1 sexual + 1 pseudo-neurology
One or more symptoms or deficits affecting voluntary motor or
sensory function that mimic a neurological or general medical
condition (e.g. impaired co-ordination, local paralysis, double vision,
seizures or convulsions
La belle indifference
Pain is primary symptom and is of sufficient severity to warrant
medical attention
Post-traumatic / post-surgical
Exacerbated by psychic factors
Preoccupation with fear of having, or the idea that one has, a serious
disease (e.g. brain tumour) based on a misinterpretation of one or
more bodily signs or symptoms
Onset often after car accident
Wake from sleep feeling un-refreshed
Wide spread pain, above and below waist, both sides of the body
Characteristic reproducible tender points: occiput, low cervical C5C7, lateral border of the sternum, post neck, lateral epicondyle 2cm
below that point, lat thigh, med knee
Treatment:






Chronic fatigue
syndrome
Factitious disorder /
malingering

OSCE-guide-III.doc

Patient education
Exercise program (walking, aquatic exercises), physical therapy
(good posture, stretching, muscle strengthening, massage)
Stress reduction, CBT
Amitriptyline 10 25 mg qhs
Gabapentin 300 mg tid

Similar to fibromyalgia but FATIGUE is the predominant feature


Associated with sleep apnea / irritable bowel syndrome

Page 234 of 255

Psychiatry

Counselling for somatisation disorder:


- I understand that you are here because of , and to (renew medication / do MRI / )
and we will discuss that, but before discussing this, I would like to explain the findings in
your case.
- Based on the symptoms ( and the surgeries you had) the most likely explanation to your
pain (headache) is a medical condition called somatisation disorder.
- What do you know about somatisation disorder? Would like me to explain?
- It is not uncommon condition, and we do not know the exact explanation for it, but we
believe that because some patients are more sensitive to pain than others, or may be due
to patients difficulty in handling stresses in their lives, these stresses may manifest as
painful experiences (symptoms).
- Do you have a family doctor?
 YES
o I will explain some points for you now, and then you can arrange a meeting with
your family physician and discuss the follow up with him, in these situations, it is
important to have only one physician dealing with all the investigations so that he
can get better understanding of the whole situation.
 NO, I do not have a family physician!
o I can be your family physician, if you would like to. That means we will set a
follow-up visits every 4 weeks, during which we will review underlying
symptoms, to make sure we are not missing any serious condition.
o We will review the stress in your life and see how we can help you with it:
 I can refer you to psychiatrist to help you deal better with any stress /
conflict in your life
 And we can consider some medications (Amitriptyline 25 mg PO qhs),
it belongs to a family of medications called TCA (tri-cyclic antidepressants) but we use it for pain control
Counselling if the patient is depressed:
- I understand that you are here because of , and to renew medication and we will
discuss that, but before that, I would like to explain the findings in your case.
- Based on what you have told me, the most likely explanation to your symptoms is a
medical condition called depression.
- We need to treat the depression with [talk therapy, behavioural modifications, and
medications (SSRIs, TCA)]  depression counselling
- For Tylenol 3 we will not stop it suddenly, I will renew it for you, but we will agree that
you will gradually cut it down, over the next few days, till the other medication (SSRIs)
kicks in.

If there was a suicidal attempt:


However, because of the suicidal attempt 2 days ago, we would like you to stay with us
in the hospital for few days so that we can start the treatment

OSCE-guide-III.doc

Page 235 of 255

Psychiatry

Drug seeker
If you find a man searching in the drawers of the hospital, firmly ask him to stop, tell him this is private
property and he is not allowed to go through this medical stuff
I wish it could be that simple, but I need more information and physical exam before I can write any
prescriptions to you, as I am a little bit concerned about the amount you have been taking, which might
have been harmful to you

Introduction
HPI
Analyze
the CC







AS





Impact







Red flags







Analyze Tylenol 3
Other medications

Why are you taking it? What was the diagnosis?


Os Cf D / PQRST / / 1st time
When did the headache (pain) start?
Did you seek medical attention? What was the diagnosis? Did you take
any medication? When did you start Tylenol 3? Why?
Analyze previous visits: is the pain different from before? How?
Other pains / headache
GIT / liver
Genito / urinary
How does this headache affect your life? How are you coping?
Have the medications been impacting your life?
Relationship with family
Education, Employment
Legal problems, police involvement?
Constitutional symptoms
Screen red flags for headache:

Trauma

Worse at night

Nausea / vomiting

Bothered by light /+/ Neck pain / stiffness

Weakness / numbness / tingling in body / seizures
Are you under stress?
Support systems

In addition to Tylenol 3, do you take any other meds? Sleeping pills?


MOAPS screening
PMH: HEAD SSS
FH of psychiatric disease
SH

Counselling
Analyze Tylenol 3
-

So you told me you are taking it for


Who prescribed it to you?
Who renewed it to you? Why?
When was the last renewal? Can you show me your last bottle?
How many tablets do you use now? And before? When did you start to the use?
When you take it, beside for the headache relief, how do you feel? What if you do not take it, how do
you feel? Shaking? Heart racing? You feel you are on the edge?
Do you renew it from the same doctor or different doctors? Why you did not go to him this time? Is it
ok that I contact him?
Do you renew it from the same pharmacy or different pharmacies? Is it ok that I contact the pharmacy?
Did you ever obtain the medication from the street?

OSCE-guide-III.doc

Page 236 of 255

Psychiatry

Given the benign history with no suspicion of ICP or focal deficits, and description of headache
consistent with the common tension headache, full neurological examination is not indicated, I
would like to perform a brief neuro screening exam  move on.
Counselling:
- I understand that you are here to renew your Tylenol 3; we will discuss that, but before
that let me ask you: what is your understanding of Tylenol 3?
- Tylenol 3 is a good medication when it is used for particular indication. Do you know
what does it contain? It contains 2 medications:
o One of them is the regular Tylenol as you buy it from the pharmacy
o The other one is codeine
- Tylenol itself is a safe and effective drug, and can be used for long time, however, if there
is no strong indication to use it, it is better to it as it might cause liver and kidney injury.
- On the other hand, the other medication codeine it is a drug belongs to the family we
call narcotics which is similar to morphine. It is an excellent pain killer if used for short
term, but, if it is used for long term, this is concerning for us, do you know why?
o First of all, people need to keep increasing the dose in order to obtain the same
effect; we call that tolerance.
o Also, if you stop using it suddenly, you will have withdrawal symptoms,
similar to that you have now; running nose, tearing, N/V, diarrhoea, drowsiness,
muscle aches, sweats, shaking, and heart racing.
- For these reasons, people get easily hooked on Tylenol 3, and can not stop it. Not only
that, they will need to keep increasing the dose. We call that a habit forming
medication.
- If I renew your medication, I will not be helping you, it will be like a vicious circle, and
the more I renew your medication; the more dependent you will be on it; the more you
will need it. For that reason it is not the right step to renew it.
o Can you give me just few pills; I have a very important interview?
o Even if I give you few pills, this is not the solution, this will be temporarily, and
the problem will keep increasing. We must stop the drug
o I can help you with sick note
o I can give you another non-narcotic medication that can help you with your pain
- I appreciate your trust to give me all the information, but based on what you described,
you are having dependence on narcotics.
- It sounds like you have been going through a lot of stress in your life. I am wondering
that if you would be interested in talking to one of our social works here, who is expert to
find out the community resources for you.
There are also some numbers you can call; they are professionals to help people deal with
medications or drugs. Or if you like, I can refer you to a detoxification center, where they
will help you to quit.
The seeker may be seeking Fiorinal
-

Fiorinal is a combination preparation of (barbiturate / caffeine / ASA) properly used only for the
relief of occasional tension headaches.
It is a habit-forming medication; that can precipitate withdrawal symptoms: agitation, delirium
and seizures.
The fact that patient consumes a lot suggests overuse due to dependence.
The patient may also develop analgesic headache syndrome, in which inappropriately used
analgesics actually cause headache.
Suggest a drug holiday, with weaning from caffeine and alcohol, proper sleep hygiene, diet
control, exercise, and stress management.

OSCE-guide-III.doc

Page 237 of 255

Psychiatry

Lithium discontinuity
Introduction






Mania

History

Have been diagnosed with bipolar 3 years ago, and would like to
discontinue your medication.
What is the medication you want to stop?
Why would you like to stop your medication?
I am glad you came here to discuss it, any other concerns

When were you diagnosed? How?


Any serious consequences? Were you hospitalized? For how
long?

Were you seen by a psychiatrist? Regular f/u?
Do you feel: DIG FAST (distractibility, impulsiveness, grandiosity,
flight of ideas, activity, sleep, talkative)
MI PASS ECG



Today

Lithium

Scan for
depression
History

Do you renew your medications on regular basis?


How much lithium do you take? From the beginning?

Are you taking it regularly?

Do you measure lithium level? On regular basis? When was the
last time? What was it? What is your target?

Are you still taking it? Did you stop?

How do you feel about lithium?

Hypothyroidism: do you have your thyroid hormone measured?
Do you feel cold? Dry skin? Constipation?  give thyroxin

Diabetes insipidus: do you feel thirsty? Drink more? Pee more?
Got your urine checked? ttt: thiazides

Abdominal pain? Nausea / vomiting?

Neuro shaking/tremors: -blockers

Neuro ataxia/balance/seizure: stop it
I know that you have been asked all these questions before, let me ask
it for another time!



Side effects

MOAPS

Do you feel: DIG FAST (distractibility, impulsiveness (with painful consequences), grandiosity,
flight of ideas, activity, sleep, talkative)
D Do you have a lot of projects? Were you able to finish it to the end? Can you focus on
multiple projects?

Are you spending more money than before? Are you borrowing money that you can not
I
pay back? Are you over-using your credit cards?

With whom do you live? Many sexual partners?

SAD: what started 1st; feeling high or talking drugs?

Have you had problems with the law? Fighting? Arrest? Speeding tickets?
G Do you feel very special? Have special mission?
Do you feel a lot of thoughts? Ideas?
F
A How much time do you spend on your projects?
How many hours do you sleep? Any changes?
S
T Did anybody mention that you are talking fast?

OSCE-guide-III.doc

Page 238 of 255

Psychiatry

Side effects of any medication:


- Liver toxicity
- Nausea, vomiting, diarrhea
- Headache
- Insomnia, irritability
- Agranulocytosis (Carbamazepine): check CBC every week, then every 2 weeks, then
every month
Counselling:
- I understand you are here because you would like to discontinue the lithium, however
before we discuss that; I would like to know your understanding about mania and mood
disorders!
- Mood disorders are common, and the most common of them is depression where people
feel low and do not concentrate and its treatment include the talk therapy and medications
that could be used for 6-12 months and could be stopped if the condition improved and in
some times we need to give the treatment for longer periods of time.
- This is not the case for mania/bipolar. We can treat and control it, but we can not cure it,
may be one day in the future we will be able to do this.
- Your chance of relapse if you stop it is 60% and after the second time this goes up to
80%, and after the third time it goes higher to 90%. You can see it is increasing.
- Based on your lithium level, which is within therapeutic target (0.5 1.2), we can
measure it today and we can try to decrease it gradually to check if you are feeling good
and closely monitor you. But you have to promise me that at anytime you feel high mood,
start to spend too much, talk fast or start not to sleep well, you have to come to see me or
go to the nearest ER and inform them.
- Regarding your inability to write, this is not related to lithium, thought block is not a side
effect of lithium. You may try some relaxation techniques to help you concentrate more.

OSCE-guide-III.doc

Page 239 of 255

Psychiatry

Manic patient
If the patient is psychotic:
First step is to detect early what is his mood?
- If high mood:
manic attack, with psychotic feature
- If no high mood:
brief psychotic disorder / schizophreniform
Usually patient brought by police or family member or asked to come by family members

Patient is talking fast and a lot, laughing, moving around

Ask whether the patient has been on medication before or not, e.g.: Lithium

Ask about any side effects of lithium medication N/V / Diarrhea / tremors / polyuria

Obtain history in the usual format
Introduction
Ask about the Mood
Assure the patient
Red flags
HPI

MOAPS

PMH / FH
SH

Assure the patient: you are safe here, you are in the hospital and no
one will hurt you
Fever / headache / nausea & vomiting / head injury

OCD

Mania (DIG FAST)

Depression (MI PASS ECG)

Suicide (SAD PERSONS)

If you leave what will happen? What would you like to do?
Screen

SAD: alcohol / substance abuse / amphetamine

Medical conditions; hyperthyroidism: history of thyroid
problems, symptoms (heart racing, sweating, heat intolerance,
neck swelling, visual field changes)
Psychiatric disease

N.B. if any patient has mood disorder; go through DIG FAST and MI PASS ECG
Management:

Explain that the patient has recurrence of his mania or bipolar. This is because he stopped
taking the Lithium.

Will examine and do some tests.

Will start medication. If Lithium is causing some troubles, we can start another medication.

Usually you need to admit the patient to control the symptoms of mania (from what you have
told me, you are meeting the criteria of what we call manic episode and I have concerns
about your safety).

OSCE-guide-III.doc

Page 240 of 255

Psychiatry

Suicidal attempt
Introduction

Analyze the event

Before

After

Psychiatric assessment
Risk
MOAPS

LOTS OF EMPATHY
And to see what should be the next step, first, I would like
to know how you feel about being saved.
o If happy, I am glad for that
o No!
Assure confidentiality
Can you tell me more about what happened?
What is the name of the medication? How many tablets? Any
alcohol with it?
Why did you do that?
Is it the first time?
Who saw you and brought you to the hospital?
Assess the plan here, was it organized? Or it was an impulse?
Did you leave a note? Recently, have you been giving your
belongings away?
What is going in your mind now?
If you leave the hospital, what are your plans? Where do you
want to go? What do you want to do?
If another crisis may happen, are you going to hurt yourself?
Were you seen by a psychiatrist? Were you given a diagnosis?
Do you see your psychiatrist? Take meds?
Assess the risk factors: Analyze SAD PERSONS
Screen for anxiety
Screen for psychosis
Screen for suicidal / homicidal ideation / self care
Past medical history / allergy / medications /

Decision
Conclusion / Counselling
SAD PERSONS
S
A
D
Sex Ag Depressio
Mal e
n
e
>
65

P
E
Previou Ethano
s
l
attempts
SAD

3-4
>5
E
R
N

R
Rationa
l
thinkin
g lost

S
Suicid
e in
the
family

O
Organize
d plan

N
NO
suppor
t

S
Seriou
s
illness

HEAD PMH
SSS

Release if enough support


Hospitalize
-

SAD
What did you think will achieve by ending your life?
Sometimes people hear voices asking them to end their life, did you hear this?
HEAD SSS
H: With whom do you live? Anybody else? Anybody else? If there is a stepparent in the image, ask about the relations with him and with other parents. Do you
feel safe at home? Then ask gradually, if there is a chance that this parent might get
angry when he drinks? May shout, may swear at, may push, and may hurt?
- Past medical history

OSCE-guide-III.doc

Page 241 of 255

Psychiatry

Decision:
- If still depressed and/or SAD PERSONS (>3-5)  admit
- It she is ok, regrets the accidents, no SAD PERSONS  release
Conclusion / Counselling:
HOSPITALIZE
- Based on our interview, I have concerns about your safety, because you have more than
THREE risk factors for suicide as per the screening test. Do you mind to stay with us in
the hospital for few days, so we can do the required investigations and start the
medications, until you feel ok, what do you think about that?
RELEASE
- Based on our interview, it is ok if you would like to leave, but you have to arrange a
follow up meeting with your family doctor within 3 days.
- However, I would like you to know that life sometimes could be challenging, and you
may face challenges in the future. It is important that you learn how to deal with
challenges. If you feel over whelmed, talk to somebody, and ask for help
- I can arrange a meeting with a social worker, a psychiatrist!
- I would also like you to promise me that if at any time you want to harm yourself or end
your life, you will seek medical help immediately; you can come to my office or call 911.
If multiple suicidal attempts  borderline personality disorder  do NOT admit
Notes:
- If no eye contact, wasting time, no pt interaction  assure confidentiality
- Whenever you hear car accident  show empathy / did you hurt yourself / ask about
who was in the car / was any one injured?
- If the person driving was < 18 and was driving alone  be curious  this must be an
important meeting / person that you really did not want to miss!
- The girl asks you to tell her mother that she crashed mothers new car! She does not want
to directly (herself) inform the mother!
o I can not do this.
o Why do you think this would help? She will not be angry
I see, however, life is full of challenges, it is better that you try to learn how to
deal with challenges yourself.
o We can help you to tell your mother by yourself, we can arrange a meeting with
your mother, I can be present, or we can ask a nurse or a social worker to be
there.
- The girl does not want to inform her parents that she did attempt suicide!
o You assess her and if she is to be released, e.g. she regrets what happened, she is
happy to be saved, no SAD PERSONS risk factors  she is competent 
respect her wishes.

OSCE-guide-III.doc

Page 242 of 255

Psychiatry

Eating disorder
Young female, her parents brought her because they have concerns about her weight
Anorexia nervosa
Restrictive

Bulimia nervosa
Binge-purge
Under weight
Distorted self image
Amenorrhea

Disturbed perception  loss of insight 


incompetence  inform parents and admit
involuntarily
Introduction

Average low weight

Binge-purge (> 3 times / week) > 3 months


Lose control  over-eat  react (purge)

Your parents brought you . How do you feel about that?


I am glad that you came:
- To figure this out
(if she is ok)
- To assure your parents
(if she is not ok)

Weight analysis
Diet
Exercise
Extra measures
Impact
MOAPS
- Mood:
scan for depression
- Organic: DM / hyperthyroidism / constitutional symptoms (cancer)
- AP:
screen for anxiety / psychosis
- S:
HEAD SSS
FH
Eating disorder / psychiatric illness / suicide
Weight analysis:
- What is your weight today?
- When did you start to lose weight? What was your weight at that time? How much did
you lose? What was your highest weight? What is your target weight?
- Why are you losing weight?
- Are you losing weight alone? Or someone else is encouraging you?
- When do you look at yourself in the mirror, how do you perceive yourself? How do you
perceive your weight?
- Do you like to dress in baggie?
- It looks like you lost a lot of weight in short period of time; I would like to know how did
you achieve that?
Diet:
Let us talk about your diet;
- How many meals do you eat per day? How about snacks?
o What do you eat in breakfast? How about the amount?
- Do you calculate calories? How much calories do you eat per day?
- Do you eat alone or with other people?
- Do you like to collect recipes? To cook?

OSCE-guide-III.doc

Page 243 of 255

Psychiatry

Exercise
- How about exercise? Do you exercise?
- How many times a week?
- Do you dance? Practice any sports?
Extra measures:
- Do you take anything else to help you to lose weight?
- Do you take stool softeners? Do you take water pills?
- Did you try before to induce vomiting?
- Do you sometimes exceed the amount of food you intended to eat? How many times a
week?
- How do you feel after that? How do you compensate?
Impact / consequences:
Because you have lost a lot of weight, I would like to know the impact of this on you!
- Do you have amenorrhea? When was the LMP? Regular?
- Do you feel cold / tired / swelling in your legs?
- Pigmentation on your skin? Fine hair growth? Skin changes?
- Any bony pains? Fractures?
- Muscle cramps? Calf pain?
- Heart racing? Light headedness, dizziness, fainting?
Conclusion:
- I am concerned that you have a condition called Anorexia Nervosa (explain)
- It is affecting your body, without treatment it could be fatal
- The treatment is to start eating and to gain weight. It is a tough task but I will refer you to
a multi disciplinary team to start treatment
- Would you like to discuss that with your parents
Management of anorexia nervosa:
- Anorexia patient is to be admitted to hospital if:
o <65% of standard body weight (<85% of standard body weight for adolescents),
o Hypovolemia requiring intravenous fluid,
o Heart rate <40 bpm
o Abnormal serum chemistry or if
o Actively suicidal
- Agree on target weight on admission and reassure this weight will not be surpassed
- Psychotherapy (individual/group/family): addressing food and body perception, coping
mechanisms, health effects
- Monitor for complications of AN
- Monitor for re-feeding syndrome: a potentially life-threatening metabolic response to refeeding in severely malnourished patients resulting in severe shifts in fluid and electrolyte
Bulimia nervosa:
- Criteria for admission: significant electrolyte abnormalities
- Treatment: biological (treatment of starvation effects, SSRIs), psychological (cognitive
behavioural therapy, family therapy, recognition of health risks)
Notes: So doctor do you agree with me that I am overweight? Or do you see me like my parents I am not
good?
- I will share your parents concern, it looks like you lost significant weight in short period of time,
and this is concerning.
- If the patient lost interest  slow down  summarize and start again slowly

OSCE-guide-III.doc

Page 244 of 255

Psychiatry

Mini-mental status exam:


Introduction: Mr Now, we will do a mental exercise, in which I am going to ask you some
questions. Some of these questions are easy, and some questions are difficult, please do as much
as you can!

Prepare this list before you go to the room in cases of delirium / dementia / post-concussion.
Then you can mark the correct or the wrong ones
1
2
3
4
5
5 Orientation to place
6
7
8
9
10
5 Orientation to time
11
14
19
22
25
27
28
29
30

12
15
20

13
16
21

23
24
26
Close your eyes!

No ifs, ands or buts

17

18

3 3 words recall immediate


5 world backwards
3 3 words recall delayed
3
2
1
1
1
1

3 steps command
Aphasia (pen / watch)
Read / execute
Write
Copy
Repeat

1-5 / Orientation to place: do you know which country we are in? Province? City? Hospital (or
street) name? Which floor (or suit number)?
6-10 / Orientation to time: do you know which year we are in? Season? Month? Day of the
month? Day of the week?
11-13 / 3 words recall immediate: I am going to tell you 3 objects, and I would like you to
repeat after me and memorize it, and I will ask you about it later! (penny/ tree/ car)
14-18 / Concentration: can you spell the word world backwards? He gets -1 for each nonmatching letter (first check if he can spell it correctly forward)
19-21 / 3 words recall delayed: can you tell me the 3 words that I told you before
22-24 / 3 steps command: give all the instructions at once; are you left or right handed? Can you
please take this paper by the hand / fold it into halves / give it back to me?
25-26 / Aphasia (pen / watch): what is the name of this? What is this?
27 / Read and execute: can you read this sentence and do what is written in it!
28 / Write: can you write a sentence for me!
29 / Copy: can you copy these two shapes!
30 / Repeat: can you repeat after me; no ifs, ands, or buts!
MMS score < 24  incompetent

OSCE-guide-III.doc

Page 245 of 255

Psychiatry

Dementia
Difficulty with memory for 6 months
Introduction
Analysis of CC
Behavioural
changes

I would like to ask some questions; then we will do a mental exercise


Memory assessment

Did anybody tell you that you have changes in your personality? Being short
temper? More arguments?

If there is a fire in this building; what are you going to do?

How about your sleep? (dementia: fragmented sleep /+/ delirium: reversed
sleep cycle; sleep at day, awake at night)

MMS
DEATH
SHAFT
MOAPS

Let us take a day of your life; I would like to see how did it affect your life?
Activities of daily living (ADL)
Instrumental Activities of Daily Living (IADL)
Organic in details and screen the rest (especially mood for pseudo-dementia)

Memory assessment: Can you tell me more about this difficulty! OCD +
- Any fluctuations in memory level?
- This deterioration is gradual slowly progressive, or is it you feel ok for a while then you
have attack then you are fine then you have another attack? (step ladder)
- Are you having difficulty memorizing numbers?
- Do you have difficulty finding words?
- Do you have difficulty reading? Writing? Calculating?
- Do you lose your stuff?
- Do you make lists to remind you to do things you used to do on regular basis? Do you
have difficulty organizing your schedule?
- Do you have difficulty doing tasks you used to do before; like tying a tie?
- Do you feel difficulty for new events, or old events?
o Recent: What did you have for breakfast? Confirm from partner!
o Remote: Who was the USA president during WWII? (Roosevelt)
ADL DEATH:
- Dressing:
- Eating:
- Ambulatory:
- Toileting:
- Hygiene:

difficulty dressing and undressing yourself?


do you remember to get all your meals? Or do you skip meals?
do you have difficulty moving around?
how about urination? Have you ever lost control or wet yourself?
any difficulty having showers?

IADL SHAFT:
- Shopping:
who is responsible for shopping? You or your wife?
- House keeping: how about house keeping, are you able to help your wife?
- Accounting:
who is responsible for banking at home?
Did you ever give cheque without balance?
- Food:
do you cook? Did you ever forget the stove on?
- Traffic: do you drive? Difficulty driving? Have you ever lost your way?

OSCE-guide-III.doc

Page 246 of 255

Psychiatry

MOAPS screening:
Mood:
- Depression pseudo-dementia?
Organic:
- Do you have nay long term disease? Kidney? Lung? Heart?
- SAD
 History of stroke? Difficult with vision / hearing? Weakness / numbness? Loss of
balance? Urinary incontinence?
 Head trauma? Injury?
 Brain tumour / infection
- Medications? OTC? Sleeping pills?
- Any history of thyroid disease? Symptoms of hypothyroidism?
 Hx of surgeries? In stomach?
 Are you vegetarian? For how long? Do you take supplements?  pernicious anemia
Anxiety
Psychosis
Self care / suicide

Dementia cases:
- 69 years old man comes to your clinic because he is keeping forgetting for the last few
months. In the next 5 minutes; take history and assess (this is too long for 5 minutes, but
during taking history, and if you mention: I would like to do the MMS exam, the
examiner will give you the score)  Alzheimer.
- 55 years old patient comes to your clinic because he has difficulty in memory. His MMS
score is 21. In the next 5 minutes, take history  thyroid.
- 67 years old man, comes to your clinic complaining of difficulty with memory. In the
next 10 minutes take history and assess (make MMS exam)  Dementia.
The cases could be:
- Thyroid disease (especially if pt is younger than 60 years)
- Alzheimer disease
- Dementia
- Depression pseudo-dementia
- HIV
- Pernicious anemia
- NPH (normal pressure hydrocephalus): if the patient has difficulty in AT of the
DEATH; i.e. falls due to ataxia and urinary incontinence

OSCE-guide-III.doc

Page 247 of 255

Psychiatry

Delirium
Delirium cases:
- A middle aged gentleman comes to your clinic because his dad is not himself for the last
3 days. Take history by proxy
- A middle aged gentleman comes to your clinic because his mom is in seniors home; they
gave her 15 units of insulin instead of 5 units, and she is not herself. Counsel him!
(insulin induced hypoglycemia  stressful event  decompensate a border line
delirium)
- Patient has surgery 3 days ago, not feeling himself. Patient will be aggressive.
- Patient has surgery 3 days ago, not feeling himself. Patient will keep repeating: I do not
know!  mini-mental status exam
Case 1: Dad has not been himself / not sleeping well
Introduction
I will ask some questions in order to reach a working plan
Analysis of the CC

How old is he? What are your concerns?
 Tell me more! Any recent stress? OCD
 Did you notice if your dad is angry / aggressive?

Does it look like your dad is seeing things do not exist? Hearing
voices? Complaining of insects crawling on his skin?
 Does he sleep during night? What a bout during the day?
 Is he eating? Taking care of himself?

With whom does he live? How is he capable of keeping life? How
does this affect his / their life?

Is it first time?

Constitutional symptoms
Causes
DD  Any headache / vomiting / neck pain / skin rash / red eyes / any ear
discharge / runny nose / teeth pain / diff swallowing / SOB / cough /
Infection
urine changes / abd pain / calf pain / swelling
Trauma  Head trauma? Injury?
Surgery  Recent surgeries? Pain at site of injection? Dressing change?
SAD  SAD
Medications  What about medications, do you have a list with medications? Go one
by one!
- Is he hypertensive? Controlled? Regular measurements?
- I can see that he is diabetic; for how long? Controlled? Regular
f/u and measurements? HbA1c?
- Cholesterol / Water pills / Anti-depressants
- Sleeping pills; if more than 1; ask if it was prescribed by the
same doctor
- Erythromycin!!! Why was he taking it? Pneumonia!
Conclusion
It looks like your dad has a medical condition called delirium it is a
serious condition. Your dad needs to be seen by a doctor ASAP, can you
bring him to see me. If he is too far, he needs to be taken to the nearest
ER; we will need to decrease or stop some of his medications, and restart
them gradually.
Theophylline (for asthma): stop and take beta 2 agonist instead
Erythromycin (for pneumonia): change the antibiotic
Lorazepam: discontinue

OSCE-guide-III.doc

Page 248 of 255

Psychiatry

Case 2: DT
Patient is agitated, delirious and uncooperative
I can assure you that are safe here, you are in the hospital and no one will
Introduction
hurt you, we would like to help you

I can see that you are looking to the wall, do you see anything? Do
you see anything else? Do you hear voices?

Doctor, do you see the spiders I see? For me, it does not look like
spiders, however, I understand that you can see them at the moment,
but I can assure you that nothing will hurt you!
Analysis of CC

I can see you are scratching; do you feel anything? Do you hear / see
anything?

Do you think any one would like to hurt you? Assure safety!

When did that start? OCD?

How was your sleep?
Full MMS exam

Constitutional symptoms
Causes
DD  Any headache / vomiting / neck pain / skin rash / red eyes / any ear
discharge / runny nose / teeth pain / diff swallowing / SOB / cough /
Infection
urine changes / abd pain / calf pain / swelling
Trauma  Head trauma? Injury?
Surgery  Recent surgeries? Pain at site of injection? Dressing change?
SAD  SAD: any shaking / sweating
Medications  What about medications, do you have a list with medications? Any
sleeping pills?

Do you have nay long term disease? Kidney? Lung? Heart?
Conclusion
It looks like you have a medical condition called delirium it is a serious
condition. It is reversible, fluctuating, impairment of LOC. It affects 25%
of Hospitalized people.

Will give medication to help you calm down

Will have a nurse close by if you need any thing

Will keep the room quiet and well lit

Will come back again to see you
Notes:
- It the patient is not cooperative, keeps repeating I do not know; start to ask the questions of the
MMS exam, they will go with you. After you finish, you can continue the rest of your exam
- If the patient is starring at the wall; ask him: I can see that you are looking to the wall, do you see
anything there?
- Mental status exam = psychiatric interview
- For delirium; we do the MMS exam daily until he improves
- For dementia; we do the MMS exam every 3-6 months; for follow-up
If confused patient (long case examination)
 GCS: only if the patient is poorly responsive
 MMS
 Cranial nerves
 Body:
- Pronator drift
- Hoffmans reflex thumb flexion  UMNL
- Cerebellar tests: finger to nose, rapid alternating movements
- Power / sensation / reflexes
 Patient standing: gait, Romberg test, planter flexion power
 Patient supine: tone

OSCE-guide-III.doc

Page 249 of 255

Psychiatry

Smoking Cessation counselling


1- Congratulations, We will speak in details about how we can work together to achieve this
healthy goal, but first let me ask you some questions, I need to have the bigger picture about your
smoking, and this will help us to figure out the best plan to achieve our goal

2- Smoking history:

When did you start smoking? For how many years?

How many cigarettes per day?
3- Reasons (motivations): to seek smoking cessation
4- Previous attempts: How many times? Why did you fail? When was the last time?
EMPATHY: failure is a normal part of trying to stop
5- Is there any other smoker in your home? Is she/he willing to quit? It will be a great idea if
both of you tried to quit at the same time, this will increase the success rate of your trial.
If she/he would like to know more information or need help, I will be more than happy to
meet her/him, we can arrange a meeting
6- Impact (complications of smoking):

Cancer (lung hemoptysis, tongue, nasopharynx, urinary bladder, other cancers)

Cardio vascular hazards (myocardial ischemia)
7- Red flags:

Constitutional symptoms

Risk factors (personal history or family history) of:

Heart disease / attack / HTN

Diabetes mellitus / hyper-cholesterolemia
8- Plan:

STAR:
i. Set a quit date, print papers with this date and stick it under your vision
so that you see it frequently during the day
ii. Tell your family, friends, they will be your support
iii. Anticipate the challenges you will face (nicotine-withdrawal effects:
headache, nausea and a craving for tobacco, insomnia, irritability,
anxiety, and weight gain)
iv. Remove cigarettes and other tobacco products (e.g. ashtrays) from your
home, car, and work

Nicotine Replacement Therapy:
i. Nicotine patch [21 mg (if smoking > 25 cig/day), 14 mg, 7 mg]
ii. Nicotine gums
iii. Nicotine inhaler

Psychological support for smoking cessation (to the craving):
i. Zyban (Bupropion):
+ used with tapering smoking for 2 weeks, then stop smoking
+ 150 mg qAM x 3 days then 150 mg bid x 3 months
+ Contra-indications: epilepsy, seizure disorder, eating disorders, patients
undergoing abrupt discontinuation of ethanol or sedatives
ii. Champix (Varenicline): urge to smoke and withdrawal symptoms
+ 0.5 mg qAM x 3 d then 0.5 mg bid x 4 d then 1 mg bid x 3 months

Investigations:
i. CBC / urinalysis / lipid profile
ii. If there is risk factors for heart diseases: stress ECG test
iii. If patient is worried, or if there is hemoptysis: chest x-ray

OSCE-guide-III.doc

Page 250 of 255

Psychiatry

Refusal to treatment counselling


Mrs 56 yrs old, was recently diagnosed with lung cancer, counsel her.
Introduction - I understand you were diagnosed recently with lung cancer
- How are you coping?
History
Brief history
- SAD
- Family history of lung cancers
Support
- With whom do you live?
- Any family support?
- How do you support yourself financially?
Lung cancer - What is your understanding of (lung cancer)?
- Do you know which type you have?
- Available treatment
- Now, I would like to explain the treatment options we have,
- Once diagnosed, usually surgery is late to be done, so we have radio / chemo
- Based on your condition and stage of cancer, the surgeon thinks that chemo
and/or radio therapy are the best line of treatment for you, this is based on
the many clinical trials and evidence-based medicine.
Treatment
- No doctor, I do not want to contaminate my body with chemicals, I am going
refusal
for spiritual therapy! OR whatever cause!
Ask for reason: Why? How long have you been thinking that way? Did you talk
about this with your family?
Assess competency; rule out depression:
- When we talk about decision-making, we need to rule out certain conditions
and to know more about your health. Can you please tell me how has your
mood been lately? Do you find yourself cry easily? Interests? Suicidal
ideation? Any major event, death, accident in the family recently? MI PASS
ECG
Counsel
If depressed  assess suicide and psychiatric consult
- You sound depressed to me, I would love to respect your wish, but I would be
more comfortable to follow that after you talk to psychiatrist, do you want to
do arrange that?
- Give it more thoughts; I will arrange the family meeting, social worker, and
psychiatrist. I will come back to talk to you again tomorrow
In not depressed  she is competent, she can refuse treatment
- After all this is your decision; I just want to make sure you know the available
treatments that were proved to be beneficial in treating lung cancer.
- Why not to try both? We start the medical therapy that we are sure it works,
and you go for spiritual therapy!
- Explain the condition, the available treatments
- The side effects of treatments and the complications of not getting treatment
(terminal illness)
- What about arranging a meeting with some one who has had the same medical
condition, and speak with him/her. You will get better insight into the disease
and you will see the results of treatment.
- How about arranging a meeting for you and your family members (if you wish)
with the surgeon, so that he can explain condition in more details?
-

A 71 years old with cancer colon, patient concerns: afraid of living with colostomy / concerned about
being a burden to the family / afraid of complications of the surgery (a friend died in similar surgery)
A 70 years old lady with lung cancer, refusing the surgery because her husband died 30 years ago in a
surgery (she does not know why) / she thinks that people die from the anesthesia

OSCE-guide-III.doc

Page 251 of 255

Psychiatry

Truth telling
Usually a son or daughter asking you not to inform the patient (parent / grandparent) about his
terminal illness or advanced condition
Introduction:
- Well, it is not unusual for families to have that request!
- Why you do not what her/him to know? (cant handle the bad news, fragile personality,
depression, )
- Does the patient have advanced directive? Will? Have discussed this before?
Explain patient must be told:
- Well, you know your loved one the best, given her/his previous reaction, it is reasonable
to have that concern.
- On the other hand, the patient has the right to know, we can not hold this right, besides,
usually; people will go through different stages when they react to bad news, we have
specialists here who can really help the patient and family to go through these stages.
Explain the reasons to tell:
- Patient has the right to know
- Patient will have suspicion about his own condition
- We need to discuss the treatment options, and it is the decision on the patient
- Patient may need to start some arrangements
Explain the implications not to tell:
- It is difficult to hide, it is a team work, will eventually know or find out
- Patient will lose the trust to doctors in general

Decision:
- Will talk to the patient to see if she/he wants to know all the details or not?!!
o If yes, we have to tell her/him
o If no, we will ask if she/he would like us to inform someone else
- In all cases, if the patient asks, we have to tell her/him
Conclusion:
- I can tell that the patient has a very caring family, it must be very hard on the family as
well, if they need someone to talk to cope, I can arrange that if they want
- I can give the family a little bit more time to think and we will talk again, patient will
eventually need to know the truth.

OSCE-guide-III.doc

Page 252 of 255

Psychiatry

Organ Donation
Explain resuscitation effort
- What has been done to the patient resuscitation
- When he came in, he was not breathing, we put down a tube, his heart was not beating,
we did the compression
- What is the outcome, patient is not responding to treatment due to the severe trauma to
the head.
Explain the condition: brain death
- Patient is in a state called the brain death, (check with the relatives if they know this
terminology)
o Irreversible brain damage, no functioning at all
o Not responding to light, to pain
o No spontaneous breathing, will never gain consciousness, when we stop the
machine, he does not breathe
o Legal term for death.
- Confirmed by two nerve specialists: neurologist / neurosurgeon:
- Show empathy: sorry for the loss, patient was young / healthy / family needs him /
Give time to family to recall how active and how nice he was
Bring the issue of organ donation:
- He was healthy, good candidate for organ donation
- If he had advanced will, driver license
- What familys view about this
- It is familys decision now, the decision is a life gift
Explain how to do it
- We have a team to do that, they will respond very quickly
- Many organs can be used
- There is time limit; decision should be made within the next 24 hrs
- You will be notified which organ used and where to go, but you wont get the
individuals name
Explain funeral
- It wont affect the arrangement for funeral
- Still can have the open casket, wont affect the face
Address any questions or concerns

OSCE-guide-III.doc

Page 253 of 255

Psychiatry

OCD
-

Address body language


Why do you wear gloves and hat? And refuse hand shaking?
Start with an open ended question; OCD / trigger /

Obsessions:
- Type of obsession: dirt and contamination, orderliness, religious, checking and
rechecking?
- Do you feel that these obsessions are not real?
- Do you want to get rid of them?
- What do you do to overcome the stress created by these ideas?
- How many times do you wash your hands? How long do you take in a shower?
- Impact on life, work,

MOAPS:
- Screen for mood disorders
- Screen for organic causes
- Screen for other types of anxiety disorder,
- Screen for psychosis
- Screen for suicide, homicide, self care

Past medical history / medication & allergies


Family history
Social Hx: smoking, alcohol, drugs, work, home, support




OSCE-guide-III.doc

Page 254 of 255

Psychiatry

NOTES

OSCE-guide-III.doc

Page 255 of 255

Das könnte Ihnen auch gefallen